全部 1- 101- 201- 301- 401- 501- 601- 701- 801- 901- 1001- 最新50


面白い問題おしえて〜な 十二問目
1 名前:132人目の素数さん:2006/09/07(木) 07:00:00
面白い問題、教えてください


2 名前:132人目の素数さん:2006/09/07(木) 07:03:00
過去ログ
http://www3.tokai.or.jp/meta/gokudo-/omoshi-log/
まとめwiki
http://www6.atwiki.jp/omoshiro2ch/

1 http://cheese.2ch.net/test/read.cgi/math/970737952/
2 http://natto.2ch.net/test/read.cgi/math/1004839697/
3 http://science.2ch.net/test/read.cgi/math/1026218280/
4 http://science.2ch.net/test/read.cgi/math/1044116042/
5 http://science.2ch.net/test/read.cgi/math/1049561373/
6 http://science.2ch.net/test/read.cgi/math/1057551605/
7 http://science2.2ch.net/test/read.cgi/math/1064941085/
8 http://science3.2ch.net/test/read.cgi/math/1074751156/
9 http://science3.2ch.net/test/read.cgi/math/1093676103/
10 http://science4.2ch.net/test/read.cgi/math/1117474512/
11 http://science4.2ch.net/test/read.cgi/math/1134352879/


3 名前:132人目の素数さん:2006/09/07(木) 07:06:07
面白い悶題めこすじ〜な 69悶目

4 名前:132人目の素数さん:2006/09/07(木) 07:23:48
>>3
死ね

5 名前:132人目の素数さん:2006/09/07(木) 11:16:04
IMO面白い

 ↓ ↓ ↓

IMO 1971の3
IMO 1990の6
IMO 1992の5
IMO 1993の3
IMO 1995の6
IMO 1999の3
IMO 2000の3
IMO 2001の3
IMO 2002の6
IMO 2003の3
IMO 2004の3
IMO 2006の6

6 名前:132人目の素数さん:2006/09/07(木) 18:29:19
0.9mmのシャープの芯から0.3mmの芯を削りだす。
最高で何本削りだせるか?


7本でいい?

7 名前:132人目の素数さん:2006/09/07(木) 21:30:44
短くてよけりゃ、いくらでも輪切りにしますが。

8 名前:132人目の素数さん:2006/09/08(金) 02:38:35
>>6
削って見せろや

9 名前:132人目の素数さん:2006/09/08(金) 10:20:14
>>6 7本でいんじゃね?

10 名前:132人目の素数さん:2006/09/08(金) 13:01:19
7本可能な事と、8本以上が不可能な事を示す必要があるとか無いとか。

11 名前:132人目の素数さん:2006/09/08(金) 16:13:43
問題は直径3の円Aの中に直径1の円が重ならないように8個入るかって事か

プログラムさえ使っていいなら有限種類の配置方法と
その際の円の重なる部分の面積を求めれば不可能かどうか判定出来るな

12 名前:132人目の素数さん:2006/09/08(金) 18:33:02
直径3の円Aの中に直径1の円が重ならないように8個入るか
⇔直径2の円の中に8個の点を配置して、どの2点間の距離も1以上となるようにできるか

下図から、領域Sには高々1点しか入らない。よって、残りの円環領域に7個の点が入ることに
なる。ところで、この円環領域は6個のパイン形領域Pに分かれるので、これら6個のうちある
パイン形領域には2つ以上の点が入ることになる。ところが、パイン形領域には高々1個の
点しか入れることができないので、矛盾。
ttp://tamago.donburi.org/src/up3069.png

こんな感じか?

13 名前:132人目の素数さん:2006/09/09(土) 05:12:35
>>12
図がよめねー。

14 名前:132人目の素数さん:2006/09/09(土) 20:40:00
ケプラーはよそう。

15 名前:13:2006/09/10(日) 08:36:56
おお!図が見れた。
それでよさげ

16 名前:132人目の素数さん:2006/09/10(日) 12:13:09
幾何20051210122540  幾何10-15
他にもっとスマートな解き方もあるだろうがwikiに解答無かったんで解いた
かなり汚いが今は反省している
ちなみに角度の「゜」は省略で表記もよくわからんのでご了承ください

∠BAC=X,BDの中点をM,ACとBDの交点をE,CD=1とする(CDは計算しやすいように便宜上ね もちろんCD≠1でもできる)
△ECD∽△CBDから @AB^2=BD×ED
△BCDについて正弦定理から BD/sin74=CD/sin30 つまり BD=2sin74
@から CD^2=2sin74×ED つまり ED=1/(2sin74)
EM=DM-ED=BD/2-ED=sin74-{1/(2sin74)}
∠MAE=16 なので △AEMについて正弦定理から AE/(sin90)=EM/(sin16)
つまり AE=〔sin74-{1/(2sin74)}〕/(sin16)={2(sin74)^2-1}/(2sin74cos74)=(-cos148)/(sin148)=(cos32)/(sin32)
AM⊥BDより AE^2-EM^2=AB^2-BM^2
{(cos32)^2}/{(sin32)^2}-(sin74)^2+1-1/{4(sin74)^2}=AB^2-(sin74)^2
AB^2={(cos32)^2+(sin32)^2}/{(sin32)^2}-1/{4(sin74)^2}=1/{(sin32)^2}-1/{4(sin74)^2}={4(sin74)^2-(sin32)^2}/{4(sin32sin74)^2}
ここで 4(sin74)^2-(sin32)^2=-2{1-2(sin74)^2}+2-{1-(cos32)^2}=-2cos148+(cos32)^2+1=(cos32+1)^2
よって AB=(cos32+1)/2sin32sin74
△ABEについて正弦定理から
AB/(sin74)=BE/(sinX)
(cos32+1)/{2sin32(sin74)^2}=(BM+ME)/(sinX)
ここで BM+ME=2sin74-1/(2sin74)={4(sin74)^2-1}/2sin74 から
(cos32+1)/{2sin32(sin74)^2}={4(sin74)^2-1}/2sin74(sinX)
(cos32+1)/(sin32sin74)={4(sin74)^2-1}/(sinX)
{2(cos16)^2}/{(2sin16cos16)(cos16)}=(2cos32+1)/(sinX)
1/(sin16)=〔2{1-2(sin16)^2}+1〕/(sinX)
sinX=sin16{3-4(sin16)^2}=3sin16-4(sin16)^3=sin(16・3)=sin48
0<X<180 は明らかなので X=48 または 132
X=132 のとき ∠ABD=106-132<0 となってしまうため不適
以上から ∠BAC=X=48

17 名前:132人目の素数さん:2006/09/10(日) 12:19:41
>>12
これは鳩ノ巣原理か

18 名前:132人目の素数さん:2006/09/10(日) 16:21:45
太陽から地球まで、光は8分20秒かかって届きます。
さて、光速が秒速30万kmとすると、地球は太陽の周りを秒速どれくらいで公転しているでしょう?

19 名前:132人目の素数さん:2006/09/10(日) 17:24:24
約2π(AU/年)

20 名前:KingOfUniverse ◆667la1PjK2 :2006/09/11(月) 07:45:38
20ならジュースでも飲むか。
廿という字は20だ。
ところで、辺を共有している面同士を同じ色で塗らずに正20面体の面に4色のうちのいずれか一色を塗る方法は何通りあるか?

21 名前:132人目の素数さん:2006/09/12(火) 18:54:00
【有名問題(鉄板少女アカネ問題)】

鉄板少女アカネ問題の解答例を参考に
アカネから堀北真希までを最短で変換せよ

解答例
http://tv7.2ch.net/test/read.cgi/actress/1157434545/212

22 名前:132人目の素数さん:2006/09/13(水) 13:10:44
>>21
aho

23 名前:132人目の素数さん:2006/09/13(水) 22:42:09
数学の問題ではないような気もするが、、、
「直径10cmの球は直径10cmの円形の穴を通過できるか?」
↑↑↑
物理板で散々もめた問題。
けっきょくどういう結論に達したかは忘れた。

24 名前:132人目の素数さん:2006/09/13(水) 23:04:03
>>23
球の表面と穴の円周がぶつかるので通過できない。
直径10cmの球及びその内部をBとすると、Bの内部(つまりB^i)は
直径10cmの円形の穴を通過できる。

25 名前:24:2006/09/13(水) 23:33:55
しまった…穴の方もまた円周が含まれるものと思い込んでた。完全な解答はこうなるな。

Rの部分集合の”長さ”(ルベーグ測度)は、その集合から零集合を除いても不変なので、長さ
だけ与えても集合は一意に定まらない。「直径10cmの球」という表現だけでは、その球は表面を
含んでいるのか分からず、同じく「直径10cmの穴」という表現だけでは、その穴は円周を含んで
いるのか分からないので、通過できるか否か判断できない。問題に不備がある。

26 名前:132人目の素数さん:2006/09/13(水) 23:50:09
>>25
まずは、印象でものを言う事を許してください。
その場合、球または穴のどちらか一方が周(円周・球表面)を含んでいなければ
ぴったりと接しながら通過できるということでよろしいですか?

その理解の場合、次のような疑問が生じます。(こちらが本題)
両方がその周を含んでいないときには、どちらか片方が周を含んでいる時にくらべ
すこし余裕があって通過できるんでしょうか?

27 名前:132人目の素数さん:2006/09/14(木) 00:08:14
>>26
>すこし余裕があって通過できるんでしょうか?
(印象では)円周分の余裕があると見てよいはず。

28 名前:132人目の素数さん:2006/09/14(木) 00:19:28
「通過できる」「すき間がある」等の定義による、としか答えられんだろ。数学的には。

実数を、ある点を境に左右に分けるとき、
「すき間がある ⇔ どちらにも含まれない要素がある」
と決めれば、(-∞,0) (0,∞) はすき間があることになるし、
「すき間がある ⇔ どちらにも含まれない、測度が0でない集合がある」
と決めればすき間はないことになる。

29 名前:132人目の素数さん:2006/09/14(木) 00:34:55
>実数を、ある点を境に左右に分けるとき、
数学的には、”ある点を境に左右に分ける”の定義が不明。
>「すき間がある ⇔ どちらにも含まれない、測度が0でない集合がある」
数学的には、何の測度か不明。A⊂Rに対してμ(A)=1 (0∈A),0 (0∈R−A)
と定義すれば、μはRのベキ集合上の測度となり、一点集合{0}は測度μに関して0でない。

30 名前:132人目の素数さん:2006/09/14(木) 00:48:00
0でないならすきまがあると決めたら0でないならすきまがある。


31 名前:132人目の素数さん:2006/09/14(木) 00:50:18
そういうことだ

32 名前:132人目の素数さん:2006/09/14(木) 17:42:39
ここに4cm, 5cm, 6cmの長さのひもがある。
これを使ってA,Bの2人がゲームをすることにした。
ひもを1本選び任意の場所を切る、ということを交互に繰り返す。
ひもの本数はそのたびに増えていく。
最終的に1cm未満のひもを作ったほうが負けというルールである。
Aが先手となったのだが、最初にどのひものどの部分を切るべきだろうか?

33 名前:132人目の素数さん:2006/09/14(木) 17:50:06
どっちも負けるんじゃないか?

34 名前:132人目の素数さん:2006/09/14(木) 17:52:49
1cmのひもを先に作ったほうが負け。
と訂正します。

35 名前:132人目の素数さん:2006/09/14(木) 18:04:00
>>34
1cm未満の、でした。

36 名前:132人目の素数さん:2006/09/14(木) 21:41:34
問題のできが悪いのでいいかげんにしか答えないが

解1
Aは初手で脂肪しました。そうです太さ0.9cmのひもにしてしまったからです。

解2
Aは降参しました。そうです太さ5kmのひもだったので
裂けるチーズの裂け方のように切っていたら
時間がかかり過ぎて決着がつきませんでした。

解3
Aは最初に5cmのひもを1cmと4cmに切ろうとしましたが作戦失敗でした。
ひもに太さがあったのでBがひもを斜めに切ってしまったからです。

37 名前:132人目の素数さん:2006/09/14(木) 21:44:48
なんだこいつ。

38 名前:132人目の素数さん:2006/09/14(木) 21:52:21
条件を追加させてください。
ひもの太さは考えないものとします。
また、誤差無しで切ることができるものとします。
つまり、例えば4cmのひもをちょうど1cmと3cmに切り分けることができます。
もちろん半端な長さに切ることも自由です。
切ることによってひもの長さの合計が増減することは無いものとします。

39 名前:132人目の素数さん:2006/09/14(木) 22:03:09
真面目だなw間抜けとも言うw

40 名前:132人目の素数さん:2006/09/14(木) 22:41:21
>>39
いや、余計なツッコミを防ぎたかったもので。

もうひとつ条件を追加します。
制限時間は考えないものとします。

41 名前:132人目の素数さん:2006/09/15(金) 06:07:21
>>32

5cmの紐を真ん中で切る。
奇数cmの紐1本を
相手に渡すと主導権を握られるから。

例えば3cmの紐が1本あると真ん中で切れば
1.5cmと1.5cmで勝てる。
これを1cmと2cmに切れば
2cmを切って返されて負ける。

つまり常に奇数cmの紐を
こちらに1本回させればいい。

まぁ完全情報零和ゲームと呼ばれる類の問題だね。
オセロや将棋と同じ。
逆算して詰める訳だ。

42 名前:132人目の素数さん:2006/09/15(金) 06:39:23
g(x) = x - (x-1)^(-1) - (x-2)^(-1) - (x-3)^(-1)
とおく。f(x)が−∞〜∞で積分可能ならば
∫_[−∞〜∞] f(g(x))dx = ∫_[−∞〜∞] f(x)dx
が成り立つことを示せ。(ネタ元はポリヤ&ゼゲーの1巻)

43 名前:132人目の素数さん:2006/09/15(金) 10:26:28
>>41
Aが5cmのひもを真ん中で切ると、ひもの長さは 2.5, 2.5, 4, 6 になります。
次にBが6cmのひもを1.5cmと4.5cmに切ると、1.5, 2.5, 2.5, 4, 4.5 になります。
これでAがどうやってもBが勝てる状態になっています。

44 名前:132人目の素数さん:2006/09/15(金) 10:33:23
>>41
5cm のひもを真ん中で切ったあと、
6cm のひもを 4.5cm と 1.5cm に切られたらどうする?

>>32
6cm を 2cm と 4cm に分ける

45 名前:132人目の素数さん:2006/09/15(金) 10:38:04
>>44
正解です。どう解きましたか?

46 名前:132人目の素数さん:2006/09/15(金) 10:53:06
>>45
昔、計算したことがあるので…
一松信の「石とりゲームの数理」にも載ってます

47 名前:132人目の素数さん:2006/09/15(金) 11:57:04
>>46
そうでしたか。
では解説します。

まず、ひもの長さについてはその整数部分だけに着目すればいいことに注意します。
例えば、6cmのひもを1.5cmと4.5cmのひもに切ったとき、これを1cmと4cmに置き換えても問題ありません。
なぜかというと、1.5cmも1cmもそれ以上切れないし、
4.5cmと4cmのひもは整数部分に着目する限り同じようにしか切り分けられない
(例えば4.5=2.2+2.3に対して4=2+2で整数部分は2と2で同じ)からです。
つまりa(aは正整数)をa=b+cなるb,cに分ける代わりに、それをa=b+cまたはa-1=b+cを満たす
正整数b,cに置き換えることで、同じゲームが成立します。
これであとは全パターンを調べれば正解がわかります。
数学的に解くには、2進数を利用する方法があるので調べてみてください。
(書くのが面倒。本当はここが重要だったりしますが。)
この問題は、選択肢が無限にあるのに、解いてみると正解はただ1つというところが面白いかと思います。

48 名前:132人目の素数さん:2006/09/15(金) 13:43:08
>>32
常に切れる紐を対称にのこせるように切る。

49 名前:132人目の素数さん:2006/09/15(金) 14:47:19
>>43
それができればいいんですが、この場合はできなくないですか?

50 名前:132人目の素数さん:2006/09/15(金) 14:48:19
>>49>>43>>48でした。

51 名前:132人目の素数さん:2006/09/15(金) 19:42:42
△ABCにおいて辺AB,AC上にそれぞれ点D,Eをとり、BEとCDの交点をFとする。
△BDF=4, △BCF=5, △CEF=6 のとき、四角形ADFEの面積を求めよ。

52 名前:132人目の素数さん:2006/09/16(土) 13:43:57
問題:無限階常微分方程式

(I + h d/dx + 1/2! h^2 (d/dx)^2 + 1/3! h^3 (d/dx)^3 + ...) u(x) = f(x)

を解け。ここで h は定数であり、微分作用素については Iu = u および
(d/dx)^k u は u の k 階微分の意味である。また、解 u を解析関数とする。


53 名前:132人目の素数さん:2006/09/16(土) 18:51:25
u(x)=f(x+h)?

54 名前:132人目の素数さん:2006/09/16(土) 20:30:14
>>53
惜しい!Taylor 展開より u(x + h) = f(x) だから x
の代わりに x - h を入れて u(x) = f(x - h) が答え。
微分方程式と言っておきながら、実はただの平行移動
という問題。

55 名前:132人目の素数さん:2006/09/16(土) 20:46:02
無理やり微分作用素

e^(h d/dx):= I + h d/dx + 1/2! h^2 (d/dx)^2 + 1/3! h^3 (d/dx)^3 + ...

を定義すれば、平行移動作用素になってしまう。不思議じゃない?

56 名前:132人目の素数さん:2006/09/17(日) 20:38:13
>>51
意外な答え

57 名前:132人目の素数さん:2006/09/17(日) 20:49:20
>>51
B = (0, 0), C = (c, 0) と置くと △BCF = 5 より
F = (f, 10/c) と置けるので二直線

BF: y = 10/(cf) x
CF: y = -10/(c(c - f)) (x - c)

を得る。△DBC = △BDF + △BCF = 9 より D = (d, 18/c) と
置けて、D が直線 CF 上にあることから d = 1/5 (9f - 4c)
であり D = (1/5 (9f - 4c), 18/c) となる。

同様に △ECB = △CEF + △BCF = 11 より E = (e, 22/c) と
置けて、E が直線 BF 上にあることから e = 11/5 f であり
E = (11/5 f, 22/c) となる。

以上から二直線

BD: y = 90/(c(9f - 4c)) x
CE: y = -110/(c(5c - 11f)) (x - c)

の交点 A = (99f - 44c, 990/c) を得る。よって、△ABC = 495
であり、四角形 ADFE = 480 となる。

58 名前:132人目の素数さん:2006/09/17(日) 20:54:44
なるほど。
もっとエレガントな解法もあるよ。

59 名前:132人目の素数さん:2006/09/17(日) 22:01:52
四角形ADFE=xとすると、メネラウスの定理から
CE/EA*AB/BD*DF/FC=1
11/(x+4)*(x+15)/9*4/5=1
45(x+4)=44(x+15)
x=480

60 名前:132人目の素数さん:2006/09/17(日) 22:22:39
>>59
おー、まさしくそれが>>58で言った解法。
図形問題に馴れてる人なら難しくなかったかな。
ちなみに、意図的に>>32で使われている数で問題を作った。

61 名前:132人目の素数さん:2006/09/17(日) 22:57:05
面白いか?

62 名前:132人目の素数さん:2006/09/18(月) 01:19:08
3284^158を11で割った剰余を求めよ。

63 名前:132人目の素数さん:2006/09/18(月) 01:22:52
>>62
4

64 名前:62:2006/09/18(月) 01:24:40
>>63
正解。

65 名前:132人目の素数さん:2006/09/18(月) 05:08:18
3^79 mod 11 で行き詰まってしまったorz

66 名前:132人目の素数さん:2006/09/18(月) 06:41:49
3^10≡1 mod11

67 名前:132人目の素数さん:2006/09/18(月) 12:45:42
>>62
3284≡6 (mod11)
∴3284^158≡6^158 (mod11)
また、
6^1≡6 (mod11)
6^2≡36≡3 (mod11)
6^3≡6^1*6^2≡6*3≡18≡71 (mod11)
6^4≡(6^2)^2≡3^2≡9 (mod11)
6^5≡6^2*6^3≡3*7≡21≡-1 (mod11)
なので、
3284^158≡6^158
    ≡(6^5)^31*6^3
≡(-1)^31*7
≡-7    
≡11-7≡4 (mod11)
∴4

68 名前:132人目の素数さん:2006/09/18(月) 12:48:10
合同式って高校で教えなくなったよね・・・。

69 名前:132人目の素数さん:2006/09/18(月) 12:48:38
なんか初等整数論で合同式を勉強したばっかりの高校生の解答って感じね

70 名前:132人目の素数さん:2006/09/18(月) 12:49:09
合同式って以前は教えてたっけ
もとから指導要領には無かったような

71 名前:132人目の素数さん:2006/09/18(月) 12:54:37
>>69
別にいいじゃないか。おまえもチンコに毛が生えてオギャーと出てきたわけではあるまい。

72 名前:132人目の素数さん:2006/09/18(月) 13:26:24
いや丁寧なのは良いことだと思うよ、うん

73 名前:132人目の素数さん:2006/09/18(月) 13:28:22
別にいいんだけど合同式なら合同式でなんかこうグッとくるような面白い解法が
あるのかなぁと勝手に期待してて拍子抜け

74 名前:132人目の素数さん:2006/09/18(月) 13:29:38
むしろ問題にいえよ。

75 名前:132人目の素数さん:2006/09/18(月) 13:30:05
tasikani!!!

76 名前:132人目の素数さん:2006/09/18(月) 13:47:32
>>67の方針なら、6^r≡−1 (mod 11)を満たす(最小の)自然数rは、もし
存在するとしたらr=5しか有り得ないことがフェルマーの小定理から
分かるので、6^1から順に計算する必要はなく、6^5だけ計算すればよい。

77 名前:132人目の素数さん:2006/09/18(月) 13:48:31
そんなことせずに6^(11-1)≡1でいいじゃん
gcm(6,11)=1なんだから

78 名前:132人目の素数さん:2006/09/18(月) 13:50:46
でもフェルマーの小定理位は使ってもばちは当たらないのでは?あと
3284=3289-5くらいは・・・

79 名前:132人目の素数さん:2006/09/18(月) 13:58:59
3284=3289-5≡-5≡6だろ
つうかこの問題はどうでも良い工夫の話しか出ようがないような

80 名前:132人目の素数さん:2006/09/18(月) 17:08:58
じゃあ終わり 誰か次の問題ヨロ

81 名前:132人目の素数さん:2006/09/18(月) 19:02:48
>>42

82 名前:132人目の素数さん:2006/09/19(火) 08:50:40
無限に伸びるゴムひもの上を蟻が歩く.
いまゴムひもの長さは2mあり、蟻が端から毎秒1pの速さで歩き始めるのと同時に、ゴムひもを毎秒1m伸ばす.
蟻の体力や寿命及びゴムの幅は十分あるものとして考えると、計算上蟻は反対側に辿り着けることが解る.
そこで、蟻はおよそ何年後に辿り着けかを有効数字2桁で答えよ.

83 名前:132人目の素数さん:2006/09/19(火) 13:08:03
1.7×10^36 年後

84 名前:132人目の素数さん:2006/09/19(火) 15:30:26
どなたか>>82の解法を教えて下さい


85 名前:132人目の素数さん:2006/09/19(火) 15:55:15
>>84
ゴムひもの最初の長さをL、伸びる速度をE、蟻の速度をV、
時刻tにおける蟻の位置をゴムひもの長さに対する相対値で表してx(0≦x≦1)とすると、
(d/dt)(L+Et)x=V+Ex
Ex+(L+Et)(dx/dt)=V+Ex
dx/dt=V/(L+Et)
x(0)=0より
x=(V/E)log{1+(E/L)t}
x=1を解くと
t=(L/E){e^(E/V)-1}

86 名前:132人目の素数さん:2006/09/19(火) 16:08:57
え、それで合ってる?

87 名前:132人目の素数さん:2006/09/19(火) 16:16:07
ん? どこか間違ってる?

88 名前:132人目の素数さん:2006/09/19(火) 16:18:20
いや数字が合わんかっただけ・・・。
どっか間違えたんだろう。わざわざレスにすることなかったすまん。

89 名前:132人目の素数さん:2006/09/19(火) 18:14:18
独りで神経衰弱をするとき最大何ターンで終了するか。

ルール:
1組52枚のトランプを使用。
同色同数字のカードをペアとする。
全てのカードをよく混ぜて裏向きに並べた状態で開始。
1ターン毎に2枚のカードをめくり、ペアならばそれを取り除き、ペアでなければ元に戻す。
全てのカードが取り除かれた時点で終了。

プレイヤーは完全な記憶力を持ち、既にめくったカードの色と数字は分かるものとする。
プレイヤーはできるだけ少ないターンで終了することを目指すものとする。

90 名前:132人目の素数さん:2006/09/19(火) 21:56:36
俺のターン!ドロー!俺は手札から(ry

91 名前:132人目の素数さん:2006/09/19(火) 22:44:07
>>89
51

92 名前:132人目の素数さん:2006/09/19(火) 23:01:08
26回

93 名前:132人目の素数さん:2006/09/19(火) 23:12:42
>>91
正解。
では同色でなくても同数字ならばペアと見なすとすると?

94 名前:132人目の素数さん:2006/09/19(火) 23:13:17
39ターン

95 名前:132人目の素数さん:2006/09/19(火) 23:31:36
どう解いた?

96 名前:132人目の素数さん:2006/09/20(水) 00:12:00
>>93
>>94があるので自信がないが42ターン

97 名前:132人目の素数さん:2006/09/20(水) 04:17:14
2枚をめくるというのが 一枚めくった時点で2枚目を選べるのか
同時に2枚めくるのかで変わってきそうだが
同色のみがペアの最大値が51ということなので前者で考える。

01ターン A 2 ← 最初の2枚は揃わない
02ターン 3 A ← 未知のものと既知のものが出る
03ターン A A ← 既知の組み合わせを取る(これより後でとってもいいが消費ターン数はかわらない)
04ターン 4 2 ← 未知のものと既知のものが出る
05ターン 2 2 ← 既知のものをとる
06ターン 5 3 ← 未知のものと既知のものが出る
07ターン 3 3 ← 既知のものをとる

: この時点で JとQが既知
22ターン K J ← 未知のものと既知のものが出る
23ターン J J ← 既知のものをとる
24ターン A Q ← 未知のものと既知のものが出る
25ターン Q Q ← 既知のものをとる

98 名前:132人目の素数さん:2006/09/20(水) 04:18:14
続き

26ターン 2 K ← 未知のものと既知のものが出る
27ターン K K ← 既知のものをとる
28ターン 3 A ← 未知のものと既知のものが出る
29ターン A A ← 既知のものをとる

: この時点で JとQが既知
48ターン K J ← 未知のものと既知のものが出る
49ターン J J ← 既知のものをとる
50ターン Q Q ← 一枚目で必ず既知のものが出るのでとる
51ターン K K ← 最後の2枚をとる

同時に2枚めくるルールなら50ターン以降が以下のように変わるかな

50ターン Q K ← 既知のもの1枚と未知のもの一枚をめくるが揃わなかった
51ターン Q Q ← 既知のもの2枚をとる
52ターン K K ← 最後の2枚をとる


99 名前:132人目の素数さん:2006/09/20(水) 05:03:48
続き

このゲームでターン数が少なくなるということは未知(同数字の1枚目または3枚目)のカードを
できるだけ少ないターンであけてしまう(すなわち既知のカードにしてしまう)ことである。

1→A 10→T 11→J 12→Q 13→K と書く。
初めてめくるカードが以下の並びの時
{A23A425364758697T8J9QTKJAQ2K3A425364758697T8J9QTKJQK}

・この並びでは、未知のカードが2枚連続で出てくることは最初の12と次の23だけしかない。
・最から数えて2枚めは未知のカードではない。
・ゆえに未知のカードを全てめくるためには少なくとも25ターンが必要
・未知のカードが出たターンではカードをとる(ペアにする)ことはできない。
・ガードを全て取るためには26ターンが必要
・つまり全てをとるためには51ターンが必要

以下、52ターンにはならないことの説明。

未知のカードを全てめくるために少なくとも26ターン必要な並びは
最後の2枚が同数のカードの時だけしかない。
しかしこのような並びでは、最後の未知のカードをめくった同ターンに
ペアにすることができてしまうので52ターンにはならない。



100 名前:132人目の素数さん:2006/09/20(水) 05:09:13
99の訂正

× ・最から数えて2枚めは未知のカードではない。
○ ・最後から数えて2枚めは未知のカードではない。

これは52ターンにならないことの説明に使ったような
最後の2枚が同数の並びではない
つまり未知のカードをめくった同ターンにペアに
できてしまうような並びではないことを説明している。


101 名前:132人目の素数さん:2006/09/20(水) 05:43:10
25ターンまですべて異なるカードをめくっていけば
26ターン目の1枚目をめくった時点で
すべてのカードの位置を特定できる。
26ターン目から全部取り続けられるんだから
51ターン目で終わるのはほとんど明らか。

102 名前:132人目の素数さん:2006/09/20(水) 11:46:43
>>97-100
既に1枚目のAをめくったことがあって、あるターンで2枚目のAがめくられたとき、
次のターンですぐに2枚のAをめくるのは損じゃないか?
温存しておけば、3枚目のAが出たときに4枚目をめくることなくペアを作ることができる。

103 名前:132人目の素数さん:2006/09/20(水) 16:08:14
>>102
それでめくる回数が節約できるとは思えないのだが
詳しく説明してくれ

104 名前:132人目の素数さん:2006/09/20(水) 17:47:53
ガウス平面上に重心が0となるような異なる4点a(1),a(2), a(3), a(4)を取る。
これらに回転 exp(it) を施し
b(k) = a(k) exp(it)
を定義する。

任意の t に対して、実部の平方和
S(t) = Re(b(1))^2 + Re(b(2))^2 + Re(b(3))^2 + Re(b(4))^2
は定数か?

定数でなければ、定数となるためのa(1),a(2), a(3), a(4)の満たす条件を言え。

105 名前:132人目の素数さん:2006/09/20(水) 18:52:01
>>93
47

106 名前:132人目の素数さん:2006/09/20(水) 19:35:41
>>103
あるターンで最初にめくったカードが3枚目のAのとき、
既出のAをめくればそのターンはハズレを回避できる。
カードの順序によっては、運が悪い場合、
このような回避が1度もできないような場合もありそうだが、
それが無いとしたらこれは有効な作戦といえる。

107 名前:132人目の素数さん:2006/09/20(水) 21:18:26
>>93
40

108 名前:91,105:2006/09/20(水) 21:24:44
>>106
1枚のカードは最大で2回しかめくられない。
2枚目のAを温存するした場合、
既出のAの3枚目、4枚目ともにあるターンの2回目にめくられると
4枚のAが2回づつめくられ、Aは最悪8回めくられる。
2枚目のAをすぐ取れば、Aをめくる回数は最悪でも7回。
最悪の場合を想定するなら取ったほうがよいことになる。

>プレイヤーはできるだけ少ないターンで終了することを目指すものとする。

これの解釈によっては温存しなければいけない場合もあるのかも。


109 名前:132人目の素数さん:2006/09/20(水) 21:47:07
>>108
温存するのは、あるターンの2つめのカードが2枚目のAの場合だよ。
次のターンで1枚目と2枚目のAをめくってしまうと、
あるターンの1つめが3枚目のAの場合にハズレを回避するチャンスが無くなる。
そのチャンスが来なくても、1枚目と2枚目のペアはいつでも取れるから残しておいて損は無いはず。

110 名前:91,105:2006/09/20(水) 21:59:06
>>109
なるほど、誤解していた。

>>97-100は「同色同数字のカードをペアとする」場合じゃないのか?


111 名前:132人目の素数さん:2006/09/20(水) 22:53:30
なんかいろいろ答えが出てるけど、みんな根拠あるの?

112 名前:91,105:2006/09/20(水) 23:57:32
>>111
47ターンの場合
1[A,2] 2[3,A] 3[4,A] 4[5,A] 5[6,2] 6[7,2] 7[8,2]
8[9,3] 9[10,3] 10[J,3] 11[Q,4] 12[K,4]
以上が12ターン目までの最悪のパターン。
4が3枚めくられているが4枚目の4があるターンの2枚目めくられるとするとA〜4は8回めくる
5-Kは最悪でも7回しかめくらない。また、最後のカードはめくらなくてもわかる。
よってカードをめくる回数は4*8+9*7-1=94
94/2=47ターン




113 名前:97:2006/09/21(木) 02:33:03
混乱させてスマン。
>>97-100 は 同色のペアの場合。51が正解らしいので 書いてみた。
色について何にも書かなかったからわけわからんことになってる。

異色でもペアを認める場合は温存法が使えるようなのでも少し減りそうだね。

114 名前:132人目の素数さん:2006/09/21(木) 09:30:21
>>112
>5-Kは最悪でも7回しかめくらない。
ここがわからぬ。

115 名前:132人目の素数さん:2006/09/21(木) 21:47:49
温存ありだったら46ターンになった。
47ターンとかあるからまだターン稼ぎできそう。

116 名前:91,105:2006/09/21(木) 21:51:53
>>114
>5-Kは最悪でも7回しかめくらない。

ごめん、確かにこれでは、わからないよね。説明不足でした。

13ターン以降の最悪のパターンは
13[5,5] 14[5,4] 15[6,6] 16[6,5] 17[7,7] 18[7,6] 19[8,8] 20[8,7]
21[9,9] 22[9,8] 23[10,10] 24[10,9] 25[J,J] 26[J,10] 27[Q,Q]
28[Q,J] 29[K,K] 30[K,Q]
めくられていないカード1枚は K
この後、既知のA-4を8ターン、5-Kを9ターンかけて取とって47ターン。

5も8回めくるとした場合
13[6,6] 14[6,4] 15[7,7] 16[7,5] 17[8,8] 18[8,5] 19[9,9] 20[9,5]
21[10,10] 22[10,6] 23[J,J] 24[J,7] 25[Q,Q] 26[Q,8] 27[K,K]
28[K.10]
めくられていないカード3枚は J Q K
この後、既知のA-5を10ターン、6-Kを8ターンかけて取とって46ターン
で1ターン短くなってしまう。


117 名前:132人目の素数さん:2006/09/21(木) 22:34:16
>>116
なんかあってそうな気がするが、それが最悪のパターンだというのは数学的に証明できるのかな。
考えてるうちに混乱してきたわ…。

118 名前:115:2006/09/21(木) 23:54:54
01[1,2] 14[7,8]   26ターン以降はカードを取り除くのみ。
02[1,2] 15[8,7]   全てのカード(52枚)を取り除くには26ターン必要。
03[1,2] 16[8,9]   25+26=51(ターン)
04[3,1] 17[9,8]   同色でなくても同数字ならばペアと見なすと最大51ターン。
05[3,2] 18[9,10]   同色同数字のカードをペアとするときと同じターン数になった。
06[3,4] 19[10,9]
07[4,3] 20[10,J]
08[4,5] 21[J,10]
09[5,4] 22[J,Q]
10[5,6] 23[Q,J]
11[6,5] 24[Q,K]
12[6,7] 25[K,Q]
13[7,6] 26[K,K]めくらなくても[K,K]はわかっている。

119 名前:132人目の素数さん:2006/09/22(金) 00:08:36
とりあえずベストの行動は決まってるのか?

120 名前:132人目の素数さん:2006/09/22(金) 01:40:01
>>118
03ターンで1が出たときに既にわかっている1とペアでとってしまえば
ターンが減ると思うのだが‥


121 名前:132人目の素数さん:2006/09/22(金) 10:51:40
>>119
ターンの1枚目は、まだめくっていないカードを優先してめくる。
ターンの2枚目は、ペアを作ることを優先する。1枚目が初めての数字の場合、まだめくっていないカードをめくる。
これでよさそうな気がする。

122 名前:91,105:2006/09/23(土) 19:52:15
>>117

先ず、あるターンの1枚目にめくったカードが既知のカードとペアにしてとれるときは、必ず取るものとする。

8回めくる数字は、2-4枚目のカードがあるターンの2枚目にめくられる。
1枚目のカードがあるターンの1枚目だとしても、ターンの2枚目になるカードが2枚多い。
全体としてターンの1枚目にめくられるカードと2枚目にめくられるカードの枚数は同じなので、
この分は他の数字のカードがターンの1枚目にめくられて相殺されなくてはならない。

最悪のパターンを作るには、8回めくる数字が多い方がいいので
7回めくる数字でターンの1枚目にめくられるカードを多くしなければならない。

7回めくる数字でターンの1枚目にめくられるカードが最も多いのは
 1枚目=ターンの1枚目
 2枚目=ターンの1枚目(※1枚目とペアにしてとるので、1枚目がターンの2枚目にもなっている)
 3枚目=ターンの1枚目
 4枚目=ターンの2枚目
となるパターンでターンの1枚目になるカードがターンの2枚目になるものより1枚多い。

6回めくる数字の場合は、ターンの1枚目になるカードがターンの2枚目になるものより
2枚多いパターンが最大になるが、最悪のパターンを作るには、8枚めくる数字1つを
6回めくる数字1つで相殺するよりも、8枚めくる数字1つを7枚めくる数字2つで相殺
したほうがよい。

8回めくる数字は、最大で4つ、残りの数字は最悪でも7回しかめくられない。
したがってカードをめくる回数は8*4+7*9=95回を超えることはない。


123 名前:132人目の素数さん:2006/09/24(日) 01:30:18
>>122
合ってるみたいだね。
プログラムで計算したら確かに47回になった。

124 名前:115,118:2006/09/24(日) 02:58:09
>>120
条件にあてはまらないから>>118はだめです。

125 名前:115,118,124:2006/09/24(日) 03:01:08
1枚ずつめくる(同時に2枚めくらない)

01[A,K] 21[6,6] 41[10,10]
02[2,A] 22[6,5] 42[Q,J]
03[A,A] 23[5,5] 43[J,J]
04[A,2] 24[7,6] 44[J,Q]
05[2,2] 25[6,6] 45[Q,Q]
06[2,A] 26[8,7] 46[Q,J]
07[A,A] 27[7,7] 47[J,J]残りQ,Q,K,K,K,KでQ,Kは既出(01ターン、46ターンで既出)
08[3,2] 28[7,8] 48[K,K]残りQ,Q,K,KでQは既出(48[Q,Q]残りK,K,K,Kのときは50ターンで終了)
09[2,2] 29[8,8] 49[K,Q]残りQ,Q,K,KでQ,Q,Kは既出
10[4,3] 30[8,7] 50[Q,Q]
11[3,3] 31[7,7] 51[K,K]又は50[K,K] 51[Q,Q]
12[3,4] 32[9,8]
13[4,4] 33[8,8]  AからQまでの各数を8回めくり、Kを6回めくる例である。
14[4,3] 34[10,9]  同じ数字のカードをめくる回数は4から8。
15[3,3] 35[9,9]  当然、めくる回数は多い方がターンを稼げる。
16[5,4] 36[9,10]  例より多くめくるためには
17[4,4] 37[10,10] (1)AからKまでの各数を8回めくる。(不可能)
18[6,5] 38[10,9]  (2)AからQまでの各数を8回めくり、Kを7回めくる。(不可能)
19[5,5] 39[9,9]  J,Kを7回めくるとき(46[Q,K] 47[K,K] 48[K,Q] 49[Q,Q] 50[J,J] 51[K,K])
20[5,6] 40[J,10]  (1)、(2)が不可能なので例のめくる回数が最も多い。答え51ターン。

126 名前:132人目の素数さん:2006/09/24(日) 03:17:05
>>123
> プログラムで計算したら
なにをどう計算したのかkwsk

127 名前:132人目の素数さん:2006/09/24(日) 10:36:07
>>126
カードを
a…場に残っていてまだめくっていない
b…場に残っていてめくったことがある
c…場から取り除かれた
の3つに分類すると、各数字の4枚についてA,B,Cの枚数は
(a,b,c)=(4,0,0),(3,1,0),(2,2,0),(2,0,2),(1,1,2),(0,0,4)
の6通りが考えられる。
(1,3,0),(0,4,0),(0,2,2)のパターンもあるが、これらのパターンがあれば
すぐに次のターンでその数字を取ることにすればよいので、実際は無いものと考えてよい。
13個の数字のうちこれらのパターンがそれぞれいくつあるかによってゲームの局面が分類される。
各局面における、ターンの1枚目と2枚目でのプレイヤーの選択と、
まだめくっていないカードをめくるときどのカードが出るかに対して、
ミニマックス法を適用することにより、ゲーム開始局面からのターン数を求めた。

ちなみに、数字がn個で各数字が4枚ずつの場合について調べてみた。
2, 6, 10, 14, 17, 21, 25, 28, 32, 36, 39, 43, 47, 50, 54,
58, 61, 65, 69, 72, 76, 80, 83, 87, 91, 94, 98, 102, 105, 109(n=30まで)
どうやらn=1の場合を除いて [(11n-2)/3] と表されるらしい。

128 名前:125:2006/09/24(日) 16:28:01
>>125の方法だと>>121の条件は満たしているが温存していないからだめ。
だめな例で混乱させてしまってスマン。
きりのいいところで次の問題どうぞ。

129 名前:132人目の素数さん:2006/09/24(日) 23:37:59
軽い問題を。

将棋盤があり、最初は真ん中のマスに駒が置かれている。
置かれている駒を縦か横に挟む2マスに新たな駒を置き、間の駒を取り除く。
これを繰り返して1マスを除く80マスに駒が置かれた状態にすることができるか?

130 名前:132人目の素数さん:2006/09/24(日) 23:42:52
端にある場合はどうする?片方だけに置かれる?操作禁止?

131 名前:132人目の素数さん:2006/09/24(日) 23:45:12
>>130
えーと、禁止。

132 名前:132人目の素数さん:2006/09/24(日) 23:47:09
あと、駒を挟む2マスのどちらかに駒がある場合も、もちろん禁止。

133 名前:132人目の素数さん:2006/09/25(月) 10:38:05
>>129出来ない

134 名前:132人目の素数さん:2006/09/25(月) 10:40:22
>>129
盤を市松模様に塗りわけ、中央のマスの属する側に 1,
そうでない側に -1 を割り当てる。
駒のあるマスに割り当てられた数の和は mod 3 で 1 と合同。
ところが、1マスを除く80マスに駒が置かれた状態は
mod 3 で 0 または 2 と合同なので、実現不可能。

135 名前:132人目の素数さん:2006/09/25(月) 11:38:11
>>134
正解!

最初に真ん中から1つずれたマスに駒が置かれている場合はどうなんだろう。
答えは知らない。

136 名前:132人目の素数さん:2006/09/25(月) 17:13:15
>最初に真ん中から1つずれたマスに
1つだけなら同じだね。
「最初に真ん中から1つずれたマスにももう1つ駒が」ってことかな。

137 名前:132人目の素数さん:2006/09/25(月) 18:54:00
-1=2.


138 名前:132人目の素数さん:2006/09/25(月) 20:16:00
>>136
>1つだけなら同じだね。

なぜ?

139 名前:132人目の素数さん:2006/09/25(月) 21:05:10
市松模様に塗り分ければ
かならず黒升におかれた駒の数と白升に置かれた駒の数は
片方が偶数、片方が奇数になるじゃん

140 名前:132人目の素数さん:2006/09/25(月) 22:47:15
>>139
ならないってば

141 名前:132人目の素数さん:2006/09/25(月) 22:56:55
市松模様に塗り分ければ
かならず黒升におかれた駒の数と白升に置かれた駒の数は
片方が偶数、片方が奇数になるじゃん

142 名前:132人目の素数さん:2006/09/25(月) 22:59:04
あ、そうか
失礼

143 名前:132人目の素数さん:2006/09/26(火) 00:22:58
age

144 名前:132人目の素数さん:2006/09/26(火) 00:35:39
ageとくか

145 名前:132人目の素数さん:2006/09/26(火) 00:36:56
1!+4!+5!=145

146 名前:132人目の素数さん:2006/09/26(火) 00:40:58
>>141
なんで?

147 名前:132人目の素数さん:2006/09/26(火) 00:50:27
T OFOFNTSFTFEN○N

○に入る数字は何か?

148 名前:132人目の素数さん:2006/09/26(火) 00:58:46
>>140だけど、>>142は俺じゃないからね
>>142=>>141と思われ

149 名前:132人目の素数さん:2006/09/26(火) 01:03:22
>>147
せめて数学らしい問題を持ってきてくれ。

150 名前:132人目の素数さん:2006/09/26(火) 01:04:42
>>148
>>141>>142なのかどうかわからんが、漏れもならンと思うよ。

151 名前:132人目の素数さん:2006/09/26(火) 01:09:00
確かに、よく考えたら>>141=>>142である根拠は無いね
違ってたら失礼

152 名前:132人目の素数さん:2006/09/26(火) 16:23:27
>>147
S

153 名前:132人目の素数さん:2006/09/27(水) 23:55:42
>>147
7


154 名前:132人目の素数さん:2006/09/28(木) 07:39:39
こんな確率求めてみたい その1/4
http://science4.2ch.net/test/read.cgi/math/1154790000/l50
での未解決問題。

161 名前: 132人目の素数さん 投稿日: 2006/09/19(火) 16:59:46
どうやっても分かりません。どなたか教えて下さい。

1,2,3,4・・・nと1からnまでの数字が書かれたカードが1枚ずつ計n枚入っている箱がk個ある。
このk個の箱のそれぞれからカードを1枚、計k枚取り出す。
取り出されたカードの数字の和がm以下である確率を求めよ。


155 名前:132人目の素数さん:2006/09/29(金) 00:25:58
n=6で固定しても十分難しいね。

156 名前:132人目の素数さん:2006/09/29(金) 14:10:27
>>154
f(x)=( Σ_[j=0,n-1](x^j) )^k =Σ_[j=0,k(n-1)](a(j)x^i) とすれば
a(m-k)はカードの和がmになる組み合わせの数になる。
カードの数字の和がm以下である確率は、(Σ_[j=0,m-k]a(j))/(n^k) 。
a(j)の漸化式は、
a(j)=-Σ_[i=1,k;(i=<j)]{(-1)^(i)C[k,i]a(j-i)           (jがnの倍数でないとき)
  =-Σ_[i=1,k;(i=<j)]{(-1)^(i)C[k,i]a(j-i)+((-1)^m)C[n,m] (j=mnのとき;mは整数)
たぶん簡単な式ではあらわせないと思う。


157 名前:132人目の素数さん:2006/10/02(月) 02:08:12
(1)下図のように、一辺に3個の○が並び正三角形を形成している。
これらの○のうち2つだけを移動し、逆三角形にせよ。

  ○
 ○ ○
○ ○ ○

(2)下図のように一辺に4個並べた場合は、3個の○を移動すれば
逆三角形になることを示せ。

   ○
  ○ ○
 ○ ○ ○
○ ○ ○ ○

(3)下図のように一辺に5個並べた場合は、5個の○を移動すれば
逆三角形になることを示せ。

    ○
   ○ ○
  ○ ○ ○
 ○ ○ ○ ○
○ ○ ○ ○ ○

(4)一般に、一辺にn個の○を並べて正三角形を作ったとき、
それを逆三角形にするには最低何個の○を移動させればよいか?

158 名前:132人目の素数さん:2006/10/02(月) 10:58:01
n は三角数という条件付?

○ ○ ○ ○ ○ ○ ○ ○
         ○
         ↓
         ○
○ ○ ○ ○ ○ ○ ○ ○

で逆三角形ってのもなし?

159 名前:132人目の素数さん:2006/10/02(月) 11:44:09
問題文をよく読みましょう。

160 名前:132人目の素数さん:2006/10/02(月) 16:04:46
>>157
[x] を x を越えない最大の整数とする。n 段のとき、
移動すべき ○ の最小数は

T(n) = 1/2 (6[n/3]^2 - 4(n - 1)[n/3] + n^2 - n)

で与えられる。よって T(3) = 2, T(4) = 3, T(5) = 5。

161 名前:出題者:2006/10/02(月) 23:24:52
出題者として、これからいくつか問題を出していきますから、解いてください。

中学生でも解ける問題です。

『問1』

まず、下の図を見てください。

http://photos.yahoo.co.jp/ph/nen_kingkong/vwp?.dir=/811c&.src=ph&.dnm=d8de.jpg&.view=t&.done=http%3a//photos.yahoo.co.jp/ph/nen_kingkong/lst%3f%26.dir=/811c%26.src=ph%26.view=t

図のように、・横    3 
      ・高さ(縦)5
      ・奥行き  6
の直方体があります。この直方体のA点からB点までの最短距離を求めて下さい。
 
但し、最短距離は、内部を通らず、この直方体の表面を通って下さい。

答えは数値のみでよいです。(解き方、解答方法はまだ提示しない下さい。)

では、皆様!お願い致します。

162 名前:132人目の素数さん:2006/10/02(月) 23:26:05
>>161
死ね

163 名前:132人目の素数さん:2006/10/02(月) 23:27:09
>>161
うぜぇよお前二度と来んな

164 名前:132人目の素数さん:2006/10/02(月) 23:28:12
>>161は死ね、氏ねじゃなくて死ね

165 名前:132人目の素数さん:2006/10/02(月) 23:32:56
>>161は死ねばいいと思うよっていうか死ね

166 名前:出題者:2006/10/02(月) 23:33:01
おまえらな〜、こっち来いといっておいて、なんじゃらほい。

167 名前:132人目の素数さん:2006/10/02(月) 23:53:41
>>161
ものさしで測ったら 10 cm ありました。

168 名前:132人目の素数さん:2006/10/03(火) 00:50:58
漏れは11.5cmだった。


169 名前:132人目の素数さん:2006/10/03(火) 01:36:16
>>166
問題が面白くないのが最大の欠点だな。

170 名前:132人目の素数さん:2006/10/03(火) 07:02:27
>>161
さっき帰宅して、見ようと思ったのでクリックしたらみれんかた
だれか図、教えてくださいやさしぃ人

171 名前:132人目の素数さん:2006/10/03(火) 08:51:34
】【 解けるかな? 】【
http://science4.2ch.net/test/read.cgi/math/1159797214/l50

ここにある

172 名前:132人目の素数さん:2006/10/04(水) 00:47:59
関数f(x)=([x]+a)(bx−[x])がx=1とx=2で連続となるように定数a,bの値を求めよ。

よろしくお願いします。。([x]はガウス記号です)

173 名前:132人目の素数さん:2006/10/04(水) 00:50:19
>>172
マルチすんな

174 名前:KingOfUniverse ◆667la1PjK2 :2006/10/04(水) 00:54:58
talk:>>172 これは右連続だから、左連続になるようにすればいい。

175 名前:132人目の素数さん:2006/10/07(土) 19:00:53
>>42
#1 ある閉区間[a,b]で1、それ以外で0をとるようなf(x)で
  >>42の等号が成り立つなら全てのfで成り立つ事を示す
#2 b-a→0の極限を調べて∀y(Σ[g(x)=y]1/g'(x) = 1)が成り立つなら
  #1のfに対して>>42の等号が成り立つ事を示す
#3 g(x)=x-Σ[k](x-a_k)^(-1)という形の関数に大して#2が成り立つ事を示す

証明のアウトラインはこんな感じでいいのか?

176 名前:132人目の素数さん:2006/10/07(土) 22:28:24
>>170 図はここだよ。
http://cocoa.gazo-ch.net/bbs/18/img/200610/1012774.jpg


177 名前:132人目の素数さん:2006/10/09(月) 11:57:50
>>166 で、君は中学生?恐らく日本の小学、中学教育をきちんと受けてきたのなら
似たような問題に出会っているはずだ、とここまで書いて自信がなくなった。
この十数年はやばいのかも。

178 名前:132人目の素数さん:2006/10/09(月) 14:43:03
>>177
この問題の元ネタは数蝉のエレ解で、
さらなる元ネタはくぬーす先生の出題らしいぞ。


まぁ、だからどうしたという訳でもないわけだが

179 名前:177:2006/10/09(月) 22:01:37
>>178 そんなすごい来歴があるような問題だったんだ、なるへそ、長方形の頂点
の存在が味噌なんですね。うっかりしてました。エレガントに解くのは難しそうですね。

180 名前:132人目の素数さん:2006/10/12(木) 15:10:39
問題
ttp://image.itmedia.co.jp/games/articles/0610/11/l_wk_0601011LV09.jpg
ttp://image.itmedia.co.jp/l/im/games/articles/0610/11/l_wk_0601011LV09.jpg

解答2
ttp://image.itmedia.co.jp/games/articles/0610/11/l_wk_0601011LV10.jpg
ttp://image.itmedia.co.jp/l/im/games/articles/0610/11/l_wk_0601011LV10.jpg
解答3
ttp://image.itmedia.co.jp/games/articles/0610/11/l_wk_0601011LV11.jpg
ttp://image.itmedia.co.jp/l/im/games/articles/0610/11/l_wk_0601011LV11.jpg
解答4
ttp://image.itmedia.co.jp/games/articles/0610/11/l_wk_0601011LV12.jpg
ttp://image.itmedia.co.jp/l/im/games/articles/0610/11/l_wk_0601011LV12.jpg

点に太さがあるとは・・・
曲線でも直線に見えればOKとは・・・
意見をきかせて


DS用ソフト「レイトン教授と不思議な町」
ゲーム史上最大のナゾに挑む――レベルファイブ新作ソフト発表会で新たな事業展開も
ttp://plusd.itmedia.co.jp/games/articles/0610/11/news103.html
>『頭の体操』で有名な問題以外にも、本作のために新たに30問ほど問題を製作し収録している。
有名な問題と新たな30問が良問なら買っても良いけど例題が糞すぎ。

181 名前:132人目の素数さん:2006/10/12(木) 16:17:32
その手の問題で、幾何の問題の暗黙の了解事を
疑わなければならないとしたら
そうでない問題の全てに「ユークリッド平面において」とかの
注意書きを付けなければならない。

逆に、そのような条件が付いてない問題は
どのような空間を仮定してもよい事になってしまう。

182 名前:132人目の素数さん:2006/10/12(木) 16:31:38
>>180
リファラエラーが出て画像見れねーぞこの野郎!!

183 名前:132人目の素数さん:2006/10/12(木) 21:47:51
>>180
リファラエラーが出て画像見れねーぞ、このケツ毛野郎!!

184 名前:132人目の素数さん:2006/10/12(木) 22:06:55
ケツ毛バーガーwww

185 名前:132人目の素数さん:2006/10/12(木) 23:28:29
>>182-183
下図の9つの点を、なるべく少ない直線の一筆書きで結ぶことができるか。

・・・
・・・
・・・

解答
ttp://plusd.itmedia.co.jp/games/articles/0610/11/news103.html

186 名前:132人目の素数さん:2006/10/12(木) 23:29:46
それで正解ならあとの正解を囲めってのも
全部の選択肢いっぺんに囲んで○くれそうだな。

187 名前:132人目の素数さん:2006/10/12(木) 23:53:45
1分に一回分裂して増殖する細胞がある。1個から始めて直径1センチの
球になるまで30分かかった。では、直径1センチの球になるまで、2個から
始めたら何分かかるか?

188 名前:132人目の素数さん:2006/10/13(金) 00:01:18
29分と言いたいところだが、
最初の2個の配置の仕方によっては、うまく球にならないかもしれない。

189 名前:132人目の素数さん:2006/10/13(金) 00:16:50
一光年ぐらい離れてる

190 名前:132人目の素数さん:2006/10/13(金) 09:07:23
>>185
3本で出来る奴は点に勝手に面積を与えてるので
直線の方に勝手に面積を与えても許されるなら
太い一本の線でできるね。

191 名前:132人目の素数さん:2006/10/13(金) 21:36:55
許されるのは点と同じだけまでw

192 名前:132人目の素数さん:2006/10/13(金) 21:48:35
4本の線でしょ

193 名前:132人目の素数さん:2006/10/13(金) 22:01:53
┏━━┳━━┳━━┓
┃            ┃
┣   ╋   ╋   ┫
┃            ┃
┣   ╋   ╋   ┫
┃            ┃
┗━━┻━━┻━━┛
こういう9個の部屋を全て通るには直線は3本で良いって問題も昔見た。

194 名前:132人目の素数さん:2006/10/13(金) 22:29:21
容易。
平行な直線を三本引く。

195 名前:132人目の素数さん:2006/10/13(金) 22:30:32
それに、直線をちょっと傾ければ交わる。

196 名前:132人目の素数さん:2006/10/17(火) 12:56:18
(1,2,...,n)の置換(a_1,a_2,...,a_n)に対してmax(Σ|i-a_i|)を求めよ

197 名前:132人目の素数さん:2006/10/18(水) 12:55:00
floor(n^2/2).


198 名前:132人目の素数さん:2006/10/18(水) 19:00:22
整数の問題

【1】「nを整数値とする、2^n +1 は15で割り切れないことを証明せよ」
【2】「2000^2000を12で割ったときの余りを求めよ」 



199 名前:132人目の素数さん:2006/10/18(水) 19:22:28
(1) 2^n (n>0) の下一桁は 2,4,8,6,2,4,・・・
だから 15 で割りきれるとすれば n=4m-2
このとき 2^n+1=4^(2m-1)+1=(3+1)^(2m-1)+1=納k=1,2m-1]C[n,k]*3^k+2
右辺第一項は 3 の倍数なので 2^n+1 は 3 の倍数にならない。
したがって 2^n+1 は 15 の倍数にならない。
n≦0 のときは明らか。

200 名前:132人目の素数さん:2006/10/18(水) 19:26:33
マルチかよ。

201 名前:132人目の素数さん:2006/10/18(水) 19:43:16
【1】15を法として、2^1≡2,2^2≡4,2^3≡8,2^4≡1となるので、自然数nに
対して 2^n≡1,2,4,8 のいずれか となり、2^n≡−1には成り得ない。

【2】2000^2000=12k+r とおく。rを求めればよい。rは明らかに4で割り切れるので、
r=4Lとおける。よって4L≡2000^2000≡8^2000=2^6000 (mod 12)となり、4で割って
L≡2^5998≡1 (mod 3)となる。0≦L<3に注意して、L=1を得るので、r=4となる。

202 名前:132人目の素数さん:2006/10/19(木) 03:23:36
【問題】
(7/3)^1000 の一の位の数字を求めよ。

203 名前:132人目の素数さん:2006/10/19(木) 04:14:35
ネタ切れか?
もっと…。もっと面白くて、斬新で、解けたと感じた瞬間ゾクゾクするような問題キボーンヌキボーンヌキボー…

204 名前:132人目の素数さん:2006/10/19(木) 07:45:54
自然数a1,a2,…,an及び自然数k1,k2,…,knがあって、各iに対して1≦ai≦kiを
満たしているとする。a1〜anの値をそれぞれ変化させるとき、Σ[i=1〜n]aiが
偶数になるのは何通りあるか。

205 名前:132人目の素数さん:2006/10/19(木) 09:14:30
>>203
その台詞は、問題を解いてからにしてくれw

206 名前:132人目の素数さん:2006/10/19(木) 16:46:45
>205 
まったくだ

>203
とはいうものの数学に興味を持ってるのは良い事だと思うから
数学オリンピックの問題もやってみれば?

207 名前:132人目の素数さん:2006/10/19(木) 22:32:45
>>202
こんなの出来るのか?

208 名前:132人目の素数さん:2006/10/19(木) 23:25:41
198より簡単そうな気がする・・

209 名前:132人目の素数さん:2006/10/19(木) 23:33:13
>>203
んじゃ、この問題といてみてくれ
x^2 + y^2 = z^2 、 xyz>60 、 gcd(x,y)=1
を満たす正の整数x,y,zがある。
この時、次は正しいと言えるか?

7以上の素数pが存在して、 p|xyzを満たす。

210 名前:132人目の素数さん:2006/10/20(金) 08:56:51
>>209
解いた

211 名前:132人目の素数さん:2006/10/20(金) 13:31:35
>>210
詳しく教えてもらおう。

212 名前:132人目の素数さん:2006/10/20(金) 14:23:15
>209

p|xyz
 ↑
ちょい質問
この|はなに?


213 名前:132人目の素数さん:2006/10/20(金) 14:37:30
pはxyzを割り切る

214 名前:132人目の素数さん:2006/10/20(金) 15:23:23
>213
サンキュ
考えてみるわん

215 名前:132人目の素数さん:2006/10/21(土) 00:06:28
>>209
デキタ。結構手こずった。

216 名前:132人目の素数さん:2006/10/21(土) 01:17:30
>>215
さぁ、解答頼むぜ

217 名前:132人目の素数さん:2006/10/21(土) 04:19:13
(1) 高々可算個の元からなる全順序集合は、
  Qのある部分集合と順序同型になる、と言えるか?
(2) 高々実数濃度の元からなる全順序集合は、
  Rのある部分集合と順序同型になる、と言えるか?

218 名前:132人目の素数さん:2006/10/21(土) 23:41:47
>>217
(1)Xが条件を満たすとすると、NとXの間に集合としての全単射g:N→Xが存在する
f(n)∈Qを{g(1),...,g(n)}と{f(1),...,f(n)}が順序同型になるよう定義すれば{f(1),f(2),...}はXと順序同型

(2)R^2に辞書式順序を入れる(a>cかa=c,b>dなら(a,b)>(c,d)となる順序)
これは実数濃度だけどRのある部分集合と順序同型にならない
(同型f:R^2→RがあるならRは[f(x,0),f(x,1)]という形をした
 互いに共通部分を持たない非可算個の閉区間を含む事になるから)

219 名前:132人目の素数さん:2006/10/22(日) 00:54:58
(2)整列可能定理により、実数Rにある順序≦'を入れて整列集合とすることが
出来る。このとき明らかに(R,≦')と(R,≦)(←普通の順序)は順序同型でない。

220 名前:132人目の素数さん:2006/10/22(日) 03:10:33
(・ω・)質問

>集合としての全単射g:N→Xが存在する

個々の要素に対して同じ定義が存在するっていうことですか?
それか(1,2,3,4,5)と(1,2,3)は(1,2,3)で同値だろ!チクショーー!!っていうことですか?

221 名前:132人目の素数さん:2006/10/22(日) 14:15:06
>>220
可算集合だからX={a_1,a_2,a_3,a_4,...}とラベリング出来る、ってだけです

222 名前:132人目の素数さん:2006/10/22(日) 14:35:48
というか可算の定義そのまま

223 名前:132人目の素数さん:2006/10/24(火) 04:14:01
>>219
これはいくらなんでも無茶だろう。

224 名前:132人目の素数さん:2006/10/24(火) 07:26:16
>>209
答え:正しい。
証明: p|xyzを満たす7以上の素数pが存在しないとすると、x,y,zの素因数は2,3,5に限られる
ことになる。x=1またはy=1のときは解が存在しないので、x>1,y>1としてよい。一般に、
x^2+y^2=z^2,gcd(x,y)=1を満たす自然数x,y,zに対して
・zは3の倍数でない。x,yのうち片方は3の倍数で、もう片方は3の倍数ではない(mod 3で考えると分かる)
・zは2の倍数でない。x,yのうち片方は2の倍数で、もう片方は2の倍数ではない(mod 4で考えると分かる)
・zは5の倍数である。x,yの両方とも5の倍数でない(mod 5で考えると分かる)
が成り立つので、これとx>1,y>1より、a,b,cを自然数として(x,y,z)=(2^a,3^b,5^c),(3^a,2^b,5^c)
と表せることが分かる。(x,y,z)=(2^a,3^b,5^c)としてよい。このとき、x^2=(z−y)(z+y)より
2^(2a)=(5^c−3^b)(5^c+3^b)が成り立つ。よって、r+s=2aを満たすある非負整数r,sに対して
5^c−3^b=2^r,5^c+3^b=2^sと表せる。左辺は両方とも偶数だから、r,sは自然数となる。また、
明らかにr<sとなる。5^cを消去すると3^b=(2^s−2^r)/2となるから、もしr≧2だとすると、
右辺は偶数、左辺は奇数となって矛盾。よってr=1となり、3^b=2^(s−1)−1=4^(a−1)−1となる。
つまり4^(a−1)−3^b=1となる。mod 8で考えることにより、これを満たすa,bは(a,b)=(1,1)に
限られる。このときx=4,y=3となり、z=5を得る。しかしxyz=3×4×5=60となり、xyz>60に矛盾。

225 名前:132人目の素数さん:2006/10/24(火) 18:34:56
>・zは5の倍数である。x,yの両方とも5の倍数でない(mod 5で考えると分かる)

ここがよく分からん。
5^2 + 12^2 = 13^2
は駄目なのか?

226 名前:132人目の素数さん:2006/10/24(火) 18:46:43
>>225
本当だ…なんか計算ミスしてたみたい。訂正します。

・zは3の倍数でない。x,yのうち片方は3の倍数で、もう片方は3の倍数ではない(mod 3で考えると分かる)
・zは2の倍数でない。x,yのうち片方は2の倍数で、もう片方は2の倍数ではない(mod 4で考えると分かる)
・zは5の倍数である。x,yの両方とも5の倍数でない(mod 5で考えると分かる)



・zは3の倍数でない。x,yのうち片方は3の倍数で、もう片方は3の倍数ではない(mod 3で考えると分かる)
・zは2の倍数でない。x,yのうち片方は2の倍数で、もう片方は2の倍数ではない(mod 4で考えると分かる)
・上の2つ及び、x,y,zの素因数が2,3,5に限られていること、そしてz>1から、zは5の倍数となる。

227 名前:132人目の素数さん:2006/10/24(火) 19:32:46
thx そして、 乙。
今から読もう。

228 名前:132人目の素数さん:2006/10/24(火) 19:52:29
自然数 n に対してf(n)を次の形で定義する。

f(n)は3^nを十進数で表現したときの、各桁の総和である。
すなわち、f(1)=3、f(2)=9、f(3)=9、f(4)=9、f(5)=9、f(6)=18……

この時、lim[n->∞] f(n)を求めよ。

229 名前:132人目の素数さん:2006/10/24(火) 20:13:20
>>228
任意の自然数mに対して、ある自然数nが存在して、

3^n=999…99a1a2a3… (←右辺は十進法表示。上からm桁が全て9になっている)

となっている(これの証明には、log[10]3が無理数であること、そして、
無理数の稠密性を使う)ので、lim[n->∞] f(n)=∞となる。

230 名前:132人目の素数さん:2006/10/24(火) 20:15:43
しまった。これではlimsup[n→∞]f(n)=∞が示せただけか。f(n)が小さい値を取る
ようなnが無限にあったら、f(n)は振動するから、lim[n→∞] f(n)は存在しなくなるな…

231 名前:132人目の素数さん:2006/10/26(木) 06:25:24
ΘをRの通常の位相とし、Cを閉集合系とし、Bをボレル集合系する。
(1)各F∈C−{φ}についてf(F)∈Fが成り立つ写像f:C−{φ] → R を1つ構成せよ。
(2)各O∈Θ−{φ}についてf(O)∈Oが成り立つ写像f:Θ−{φ} → R を1つ構成せよ。
(3)各A∈B−{φ}についてf(A)∈Aが成り立つ写像f:B−{φ} → R を1つ構成せよ。(ゴメン。俺には解けない。)

232 名前:KingOfUniverse ◆667la1PjK2 :2006/10/26(木) 06:37:35
talk:>>231
Fに正の数が含まれるならばそのうちの最小、それ以外の場合はFの最大。
0,-1,0,1,-2,-3/2,-1,-1/2,0,1/2,1,3/2,2,-3,-8/3,-7/3,-2,-5/3,-4/3,-1,-2/3,-1/3,0,1/3,2/3,1,4/3,5/3,2,7/3,8/3,3…の中にOに含まれるものが存在するのでその最初のもの。
ボレル集合に対しては選択公理を使うか?

233 名前:132人目の素数さん:2006/10/26(木) 07:02:54
しまった… (3)は選択公理が無いとダメかも。(1),(2)は選択公理無しで構成可能。

234 名前:KingOfUniverse ◆667la1PjK2 :2006/10/26(木) 07:08:42
閉集合関係は、正の数のうちの最小ではなくて、0以上の数のうちの最小とするべきだったか。
ボレル集合族は閉区間族を含む完全加法族であるといった情報はあるが、逆に言うとそれしか分からないのだ。

235 名前:132人目の素数さん:2006/10/26(木) 10:31:33
5++5=== 

236 名前:132人目の素数さん:2006/10/26(木) 15:43:46
ボレルでなくても、Fσ, Gδ で十分むづかしい。

237 名前:132人目の素数さん:2006/10/27(金) 02:36:48
確率1/2で当たるくじがあり、当たると1点、外れると−1点もらえる。正の実数εに対して、
Pε(n)=「n回くじを引いたとき、n回とも、獲得した点数の合計がε未満である確率」
とおく。lim[n→∞]Pε(n)=0となることを示せ。(例えば、100点以上の点数を獲得
したいとすると、くじをずっと引き続けていれば、ツキがまわって来て めでたく
100点以上の得点を獲得できる。)

238 名前:132人目の素数さん:2006/10/27(金) 08:20:36
「小数展開に 1 が現れない実数全体は測度 0」 の証明と同じようにできる。

239 名前:132人目の素数さん:2006/10/28(土) 05:01:18
「小数展開に 1 が現れない実数全体は測度 0」の証明を知らない俺に教えてくれ。

240 名前:132人目の素数さん:2006/10/28(土) 11:38:46
>>231
(1)F∈C−{φ}を任意にとる。F=∪[x∈Z]([x,x+1]∩F)と表せるから、F≠φであることより、[x,x+1]∩F≠φ
なるx∈Zが存在する。そのようなxのうち|x|が最小のものをとる(x,−xの2つがとれるときは、正の方でも選んで
おこう)。[x,x+1]∩Fは有界な閉区間だからコンパクトであり、よって最小値αが存在する。α∈[x,x+1]∩F⊂F
となっているから、f(F)=αとおけばよい。
(2)B={(a,b)|a,b∈Q,a<b}とおくと、BはΘの開基であり、Bは可算集合である。つまり、位相空間(R,Θ)は
第2可算公理を満足する。B={(an,bn)|n∈N}と表記しておく。O∈Θ−{φ}を任意にとる。このとき、
(an,bn)⊂Oを満たす(an,bn)が少なくとも1つ存在するから、そのような(an,bn)のうち、nが最小のものを
とる。そして、x=(an+bn)/2とおく。このときx∈(an,bn)⊂Oだからx∈Oである。よって、f(O)=xとおけばよい。

241 名前:132人目の素数さん:2006/10/28(土) 14:02:55
>>240
>232 と本質的に同じ。清書ごくろうさん。

242 名前:132人目の素数さん:2006/10/29(日) 02:07:49
一辺の長さが1の正n角形に含まれる正2n角形のうち、面積が
最大であるものを求めよ。また、その理由も示せ。

243 名前:132人目の素数さん:2006/10/29(日) 02:23:49
π^e    と e^π    の大小関係を述べよ。
(πのe乗)   (eのπ乗)

*関数電卓禁止

244 名前:132人目の素数さん:2006/10/29(日) 02:25:21
んな、高校レベルの問題……

245 名前:132人目の素数さん:2006/10/29(日) 03:08:20
>>242
それってホントに最大値があるのか?
いくらでも大きくできそうな気がするンだが

246 名前:132人目の素数さん:2006/10/29(日) 03:43:02
nを固定して正2n角形のとり方を考えるってことなのかな?

247 名前:132人目の素数さん:2006/10/29(日) 03:53:07
まぁ、最大値をn使って表現するんだろうしなぁ……

248 名前:132人目の素数さん:2006/10/29(日) 05:24:49
>>243
確かに、スレタイの趣旨には合っているが、そんなの中学高校で既出なんだよ! アホか!
次に期待しておるぞ、下がってよい!

249 名前:132人目の素数さん:2006/10/29(日) 09:43:07
>>248
中学レベルで解いて。いや、問題の意味を説明して。

250 名前:132人目の素数さん:2006/10/29(日) 10:53:49
pailoge-elogpai=pai-elogpai

251 名前:132人目の素数さん:2006/10/29(日) 11:07:08
>>249
問題の意味が分からないって、マジで小学生なのか?
このスレはいつからオムツの取れないガキの溜まり場になったんだ?


('A` ) プウ
ノヽノ) =3'A`)ノ ヒャー
  くく へヘノ

252 名前:132人目の素数さん:2006/10/29(日) 12:59:25
>>251
いや、単に中学レベルって言われたからイヤミで切り替えしただけだろ。
中学だとe習ってないから、問題の意味が説明できないって言いたいみたいだな。

どっちにしろ、スレ違いだが

253 名前:132人目の素数さん:2006/10/29(日) 13:34:04
a[n+1]=√{1+(Σ[1,n]ak)^2}を解け。

254 名前:132人目の素数さん:2006/10/29(日) 16:19:38
(z-a)^-bdz、bは実数はどうやって複素積分するの?

255 名前:132人目の素数さん:2006/10/29(日) 16:35:17
このスレでは面白くもない問題は容赦なくスルーされます

256 名前:132人目の素数さん:2006/10/29(日) 17:45:59
State Koenig's Theorem. Use it to prove that 2^aleph0<>alephw.

257 名前:132人目の素数さん:2006/10/29(日) 20:17:54
まだ解けていないのか。。。プッ、

258 名前:132人目の素数さん:2006/10/29(日) 22:05:31
>>253を熔けないのが数学板の低さをものがたっている。
定時限な奴らの集まりである。こいつらを積分してやっても意味なく定Level。

259 名前:132人目の素数さん:2006/10/29(日) 22:25:30
>>258
>定時限

日本語から勉強しなおしてくださいね。

260 名前:132人目の素数さん:2006/10/29(日) 22:49:14
>>259は2chに華々しくデビューしたばっかの亜歩だお

261 名前:132人目の素数さん:2006/10/29(日) 23:20:08
おこちゃまは寝る時間ですよ。 荒らさないでね。 プケラ!

262 名前:132人目の素数さん:2006/10/29(日) 23:34:21
>>258
低レベルだな。せめて「こいつらのレベル全体の集合はルベーグ零集合」とか言ってくれ。

263 名前:132人目の素数さん:2006/10/29(日) 23:47:44
それだと全員のレベルが100でもルベーグ零集合だお( ^ω^)

264 名前:132人目の素数さん:2006/10/30(月) 00:38:33
そうでつね〜君達はグラスマン数にしとこうかなテラワロス

265 名前:132人目の素数さん:2006/10/31(火) 21:15:37
Rを実数全体の集合、≦をRの普通の順序とする。Rの非可算な
部分集合Aのうち、(A,≦)が整列集合となるものは存在するか。

266 名前:132人目の素数さん:2006/10/31(火) 21:32:30
>>265
存在しない。
Aが整列集合だとすると、Aの任意の元aとaより大きい最小の元bに対してa<c<bなる有理数cを対応させれば
AとQの部分集合が1対1に対応するのでAは高々可算。
(aがAの最大の元であればa<cなるcをとる)

267 名前:132人目の素数さん:2006/10/31(火) 23:24:26
>>250
続きをつづけて〜な(;´д`)ハァハァ

268 名前:132人目の素数さん:2006/11/01(水) 16:36:36
(x-a)(x-b)(x-c)……(x-y)(x-z)

を展開するとどうなる?

269 名前:132人目の素数さん:2006/11/01(水) 16:38:36
0って言いたいんか

270 名前:132人目の素数さん:2006/11/01(水) 16:45:43
>>269
正解。
つまらん問題だったか…

271 名前:132人目の素数さん:2006/11/01(水) 21:26:12
2つの円が、異なる2点A,Bで交わっている。
双方の円に共通な接線を1本引き、その接点をS,Tとする。
このとき、直線ABは線分STを二等分することを示せ。

座標と三角比でガリガリやったら一応証明できたが、
ちっとも勝った気がしないので、初等的解法を募集。

272 名前:132人目の素数さん:2006/11/01(水) 21:32:55
中心を結ぶ線とAbは直角だから、と、半径が同じだから、あとは見れば
わかるだろうぐらい書きなぐっておけばいい。

273 名前:132人目の素数さん:2006/11/01(水) 21:34:13
>半径が同じだから、


274 名前:132人目の素数さん:2006/11/01(水) 21:38:51
>>270
散々既出!
100万年ROMってから来い!
(゚Д゚)≡゚д゚)、カァー ペッ!!

('A` ) プウ
ノヽノ) =3'A`)ノ ヒャー
  くく へヘノ

275 名前:132人目の素数さん:2006/11/01(水) 22:04:28
はんけいがことなるのなら、接線は両側にあるから、2等分同時に出来たら、
abは折れ線になるじゃないか。。。

276 名前:132人目の素数さん:2006/11/01(水) 22:07:35
>>275
なんか、勘違いしてないか?

277 名前:132人目の素数さん:2006/11/02(木) 05:31:46
>>271
初等的なものになるかは知らんがこんなのはどうだろう。
2つの円P、Qの半径が同じとき題意は明らかに成り立つ。
次に3次元でP、Qを下の様に置く
P: x^2+y^2=1, z=0
Q: (x-x_0)^2+y^2=1, z=h>0, 0<x_0<2
そして直線AB,STをz方向に広がる平面にしておく。

このとき+zから見るとQのほうが半径が大きく見える…(*)
するとSTが二等分されるのは明らか…(**)

(*)でしかも(**)な写像が見つかるといいねって話。

278 名前:KingOfUniverse ◆667la1PjK2 :2006/11/02(木) 09:28:24
Iを実数空間の区間とし、f:I->Rを凸関数とすると、{(x,y)|x∈I,y∈R,y>=f(x)}はR^2の凸集合であることを証明せよ。
しかし、すぐにできるかもしれない。

279 名前:132人目の素数さん:2006/11/02(木) 13:02:12
>>271
つ[方べきの定理]

280 名前:132人目の素数さん:2006/11/02(木) 16:51:19
>>253をいまだに誰も熔けてない阿保の集まり。
では>>253をKing氏!この問題を幾何的に溶いてくれ。

281 名前:KingOfUniverse ◆667la1PjK2 :2006/11/02(木) 17:11:32
talk:>>280 何やってんだよ?

282 名前:132人目の素数さん:2006/11/02(木) 18:35:28
>>281溶けないのか?

283 名前:132人目の素数さん:2006/11/02(木) 18:46:24
そりゃあ溶けないだろうよ。氷じゃあるまいし

284 名前:132人目の素数さん:2006/11/02(木) 18:55:02
宿題か

285 名前:132人目の素数さん:2006/11/02(木) 21:37:13
>>253
は無視
>>253
は無視
>>253
は無視
>>253
は無視

286 名前:132人目の素数さん:2006/11/02(木) 21:56:51
球を平面で切断したら切り口が円になるけど
n個の平面でランダムに切断した時にできるn個の円の交点の数の期待値は?

確率幾何とかいう分野の問題らしい

287 名前:132人目の素数さん:2006/11/02(木) 22:02:26
>>286
そのn個のランダムな平面つーのが、どういう条件なのか言ってくれないと……

288 名前:132人目の素数さん:2006/11/02(木) 22:16:42
>>251 = >>274

はしゃぎ過ぎw

289 名前:132人目の素数さん:2006/11/02(木) 22:55:01
はやく>>253を溶けはやく>>253を溶けはやく>>253を溶け
はやく溶けはやく溶けはやく溶け
はやく溶けはやく溶けはやく溶け
はやく溶けはやく溶けはやく溶け

290 名前:132人目の素数さん:2006/11/02(木) 22:56:04
>>253はやく溶け溶け溶け溶け溶け溶け溶け溶け溶け溶け溶け溶け
はやく溶け溶け溶け溶け溶け溶け溶け溶け溶け溶け溶け溶け
はやく溶け溶け溶け溶け溶け溶け溶け溶け溶け溶け溶け溶け
はやく溶け溶け溶け溶け溶け溶け溶け溶け溶け溶け溶け溶け
はやく溶け溶け溶け溶け溶け溶け溶け溶け溶け溶け溶け溶け
はやく溶け溶け溶け溶け溶け溶け溶け溶け溶け溶け溶け溶け
はやく溶け溶け溶け溶け溶け溶け溶け溶け溶け溶け溶け溶け

291 名前:132人目の素数さん:2006/11/02(木) 22:57:05
>>253はやく溶け溶け溶け溶け溶け溶け溶け溶け溶け溶け溶け溶け
はやく溶け溶け溶け溶け溶け溶け溶け溶け溶け溶け溶け溶け
はやく溶け溶け溶け溶け溶け溶け溶け溶け溶け溶け溶け溶け
はやく溶け溶け溶け溶け溶け溶け溶け溶け溶け溶け溶け溶け
はやく溶け溶け溶け溶け溶け溶け溶け溶け溶け溶け溶け溶け
はやく溶け溶け溶け溶け溶け溶け溶け溶け溶け溶け溶け溶け

292 名前:132人目の素数さん:2006/11/02(木) 22:58:53
>>253答だけでもいいから早く解いてみろ!無視とかいって
学力がないのに数学板うろついてるニートたちよ!>>253はやく溶け溶け溶け溶け溶け溶け溶け溶け溶け溶け溶け溶け
はやく溶け溶け溶け溶け溶け溶け溶け溶け溶け溶け溶け溶け
はやく溶け溶け溶け溶け溶け溶け溶け溶け溶け溶け溶け溶け
はやく溶け溶け溶け溶け溶け溶け溶け溶け溶け溶け溶け溶け
はやく溶け溶け溶け溶け溶け溶け溶け溶け溶け溶け溶け溶け
はやく溶け溶け溶け溶け溶け溶け溶け溶け溶け溶け溶け溶け

293 名前:132人目の素数さん:2006/11/02(木) 23:07:49
>>253はやくとけ
(@>▽<@)ノφ(^∀^*)♪
(★嬉+O∀o*)??(゜Q。)??
(ノд<。`)ノ♪(ーεー*)
ヾ(*≧▽≦)〃(≧∇★)
ヾ(@^∂^@)¶キタ――(゜∀゜)――!!(((゜Д゜)))ガクガクブルブル(o^_^o)(^-^)v(*⌒▽⌒*)
ヾ(^▽^)ノわーいヽ(^^ )p(^-^)q(ー'`ー;)なぬ?(-.-")凸 チッチッチヽ(*`Д´)ノ(ノ-"-)ノ~┻━┻o-_-)=○☆(x_x;)
(;_;)>>253早く解け(・∀・)
(=゚ω゚)ノm(_ _)m(^3^)/チュッ(?_?)φ(._.)メモメモ(゚Д゚;≡;゚д゚)O(><;)(;><)O

294 名前:132人目の素数さん:2006/11/02(木) 23:56:32
>>285
出題者、必死だな

295 名前:132人目の素数さん:2006/11/03(金) 00:12:38
じゅーななの女子高生でっす^-^
>>253を解いてくれたステキな人と付き合っちゃいます
痩せ型、童顔、大きめの目がチャームポイントだよ♪

296 名前:271:2006/11/03(金) 02:33:02
>>279をヒントに考えてみた。
ABとSTの交点をPとすると、方べきの定理より
SP^2 = PA*AB = TP^2 より SP=TP である。■
こんなにあっさり決まるとは‥‥まさに瞬殺。
気づかないと泥沼だぁ。

>>277
発想が、俺の某友人に似ている。

297 名前:132人目の素数さん:2006/11/03(金) 02:36:43
>>296
方べきの定理を知っていますか?

298 名前:132人目の素数さん:2006/11/03(金) 02:54:59
なぜ誰も>>253を解かない・・・?
なぜ無視する


解け!解け!解け!解け!解け!解け!
解け!解け!解け!解け!解け!解け!
解け!解け!解け!解け!解け!解け!
解け!解け!解け!解け!解け!解け!
解け!解け!解け!解け!解け!解け!
解け!解け!解け!解け!解け!解け!


299 名前:132人目の素数さん:2006/11/03(金) 02:55:10
253初項は?

300 名前:271:2006/11/03(金) 03:09:02
>>297
訂正。真ん中はPA*ABではなく、PA*PBだな。
正直、弦の交点が円の外に出ているタイプは知らなかった。
昔どっかで見た記憶もあるが、こうやって実際の問題に
自力で適用できなかったわけだから知らないも同然。
証明が、交点が内部にある場合とほぼ同様にできることは確かめた。

301 名前:132人目の素数さん:2006/11/03(金) 03:09:52
>>300
うむ、それでいい。

302 名前:132人目の素数さん:2006/11/03(金) 04:34:38
>>286
確率幾何の問題なら、任意の測度に対して期待値を求めろって事なのかね

303 名前:132人目の素数さん:2006/11/03(金) 04:42:56
>>287ランダムはランダムや。無作為に条件付いたら無作為ちゃうやん。

304 名前:132人目の素数さん:2006/11/03(金) 06:23:50
平面で円をn個書くとき、交点の最大数はいくつ?

305 名前:132人目の素数さん:2006/11/03(金) 06:24:38
それを球面にマッピングすれば。。。

306 名前:132人目の素数さん:2006/11/03(金) 06:41:48
球面上でnこの円を書くとき、交点の数の最大は?

307 名前:132人目の素数さん:2006/11/03(金) 06:55:15
平面では直線は1回しか交わらない。球面では2回、直線が3回交わる空間はどんな空間?


308 名前:132人目の素数さん:2006/11/03(金) 09:43:26
座席が40列、最前列が10席、各列は前の列より2席おおくなっている。
全部で何席あるか? ヒント 台形の面積の公式を使う

309 名前:132人目の素数さん:2006/11/03(金) 11:21:13
an+1=√{1+(Σ[1,n]ak)^2}これを出題する!!解けるかな?

310 名前:132人目の素数さん:2006/11/03(金) 12:29:49
早く解いて

311 名前:132人目の素数さん:2006/11/03(金) 13:30:12
初項を言い当てたらいいだけ

312 名前:132人目の素数さん:2006/11/03(金) 13:50:23
最初だけiで後全部0でいいのか?

313 名前:132人目の素数さん:2006/11/03(金) 14:05:29
ok

314 名前:132人目の素数さん:2006/11/03(金) 15:11:40
n個の球を交差させて出来る交差面の最大数は?

315 名前:132人目の素数さん:2006/11/03(金) 15:15:39
n個のおっぱいを交互に愛撫する組み合わせの数は?

316 名前:β ◆aelgVCJ1hU :2006/11/03(金) 15:16:23
球をだんだんと大きくしていけばいい。
2+3+4+…n

317 名前:132人目の素数さん:2006/11/03(金) 17:38:51
球面上において直線は存在しない

318 名前:132人目の素数さん:2006/11/03(金) 18:43:03
>>317
一般に、2点間の道のりの最小値を与える曲線を、
その曲面上の「直線」と定義できる。
球面上では、それは球と直径を共有する円になり、大円と呼ばれる。

319 名前:132人目の素数さん:2006/11/03(金) 18:52:12
>球面上において直線は存在しない
みんな、幼少の頃から平面幾何しか教えられてないから、こういうふうに
洗脳されちゃうんだよな。小学校から非ユークリッド幾何を教えればいいのに。
もちろん、突っ込んだ内容ではなく、「直線」の何たるかが理解できる程度に。

320 名前:132人目の素数さん:2006/11/03(金) 19:08:18
問題は定義のなんたるかだな
もし直線が無いと定義すると同じ論議で円も球もどんなな図形も定義上有り得ないことにならんか?
こんなこと言い出せば切り無い気がする
全ては定義なんだよ


321 名前:132人目の素数さん:2006/11/03(金) 19:18:35
ならん。

322 名前:132人目の素数さん:2006/11/04(土) 03:30:48
直線がないと円をかけない。。。

323 名前:132人目の素数さん:2006/11/04(土) 03:35:52
直線の定義は接ベクトルが平行

324 名前:132人目の素数さん:2006/11/04(土) 03:49:47
>>309初校は1だ!さぁとけ

325 名前:132人目の素数さん:2006/11/04(土) 03:51:41
>n個のおっぱいを交互に愛撫する組み合わせの数は?
365

326 名前:132人目の素数さん:2006/11/04(土) 09:24:44
あとは順番に入れればいいだけ。お前はもう解けている。。。。

327 名前:132人目の素数さん:2006/11/04(土) 10:08:07
お前はもう溶けている。。。。

328 名前:132人目の素数さん:2006/11/04(土) 10:09:49
ヒントをくれてやる
連分数に直せば収束値がでるから、あとは逆算するだけ。

329 名前:132人目の素数さん:2006/11/04(土) 13:00:34
>>309をさあ溶くんだ!これが数学板の実態なねか?
さぁさぁ早く溶くんだ。
今すぐ溶きなさい。

330 名前:132人目の素数さん:2006/11/04(土) 14:17:16
ほとんどの漸化式は解析的に解けないのが数学界の常識だって
ことすら知らないのですか?
パソコンで計算してグラフにして見な?

331 名前:132人目の素数さん:2006/11/04(土) 14:21:18
>>309の前科式解けますよメガワロス
ぷぷぷ…
幾何的にも解けるし普通に解いてもできるからwww
とりあえず>>330は数学かじっただけなので帰ってよろしい。

332 名前:132人目の素数さん:2006/11/04(土) 14:24:06
こいつ2乗の位置まちがってるよ。

333 名前:132人目の素数さん:2006/11/04(土) 14:39:15
>>332間違ってねぇよwwwwwww

334 名前:132人目の素数さん:2006/11/04(土) 14:40:53
a1=i
an=0(n≧2)

335 名前:132人目の素数さん:2006/11/04(土) 14:42:59
余裕で解ける

336 名前:132人目の素数さん:2006/11/04(土) 14:46:54
an+1=√{1+Σ[1,n](ak)^2}
ヒントをくれてやる
連分数に直せば収束値がでるから、あとは逆算するだけ

337 名前:132人目の素数さん:2006/11/04(土) 14:48:16
さらっと二乗の位置が変わってるな

338 名前:132人目の素数さん:2006/11/04(土) 14:49:29
(ak)^2なら簡単すぎるだろ

339 名前:132人目の素数さん:2006/11/04(土) 14:50:20
3乗でもよゆうでとけるだろ。。。早くといてみろよ

340 名前:132人目の素数さん:2006/11/04(土) 14:54:24
だから>>309は a1=i、an=0(n≧2)でいいじゃん

341 名前:132人目の素数さん:2006/11/04(土) 15:00:09
>>336
an+1 = √{1+Σ[1,n](ak)^2}
an = √{1+Σ[1,n](ak)^2} - 1
an = √(1 + n(ak)^2) - 1

ほい、解いた。

342 名前:132人目の素数さん:2006/11/04(土) 15:03:56
>>309初校は1だ!さぁとけ

343 名前:132人目の素数さん:2006/11/04(土) 15:04:38
>>332間違ってねぇよwwwwwww

344 名前:132人目の素数さん:2006/11/04(土) 15:07:59
>>341
正解!

345 名前:132人目の素数さん:2006/11/04(土) 15:40:19
おいおい普通一般項だすだろwさぁ一般項をだせ
a[n+1]=√{1+(Σ[1,n]ak)^2}a[1]=1

346 名前:132人目の素数さん:2006/11/04(土) 15:59:16
ねぇまだー?
早く溶いて解いて溶いて解いて溶いて

347 名前:132人目の素数さん:2006/11/04(土) 16:00:22
なんで、二乗の位置が書くたびに変わるんだ?

348 名前:132人目の素数さん:2006/11/04(土) 16:00:44
n≧2のとき、与式からa[n+1]=√{1+(Σ[1,n]ak)^2}a[1]=1
すなわちa[n+1]=1となるので、a[n]=1 (n∈N)となる。

349 名前:132人目の素数さん:2006/11/04(土) 16:02:48
>>348
絶対やると思ったww

350 名前:132人目の素数さん:2006/11/04(土) 16:10:06
>>348バロス
二乗の位置はこれが正しい。
a[n+1]=√{1+(Σ[1,n]ak)^2}

351 名前:132人目の素数さん:2006/11/04(土) 16:10:27
(1+n^2(n+1)^2/4)^.5
ぐらいだね。

352 名前:132人目の素数さん:2006/11/04(土) 16:12:24
a[n+1]=√{1+(Σ[1,n]a[k])^2}なのか

a[n+1]=√{1+(Σ[1,n]ak)^2}なのか

an+1=√{1+(Σ[1,n]ak)^2}なのか

出題者溶け溶け言ってないで書き方統一してくんないかな

答え全然違ってくるじゃん

353 名前:132人目の素数さん:2006/11/04(土) 16:17:16
a[n+1]=√{1+(Σ[1,n]a[k])^2}すまん>>350はおれだがこれが正しい。
勝手に書き換えるなよ虫ども。

354 名前:132人目の素数さん:2006/11/04(土) 16:29:21
a[1]=i
a[n]=0(n≧2)

355 名前:132人目の素数さん:2006/11/04(土) 16:57:39
>>354…ひっひぃ〜やめてくれwwww
早く解いてー

356 名前:132人目の素数さん:2006/11/04(土) 17:46:15
S[n]=a[1]+a[2]+…+a[n]とおくと、与式はS[n+1]=S[n]+√(1+S[n]^2)と
変形できる。実はS[n]=1/tan{π/2^(n+1)}と表せることが、数学的帰納法により
分かる。よって、a[n]=S[n]−S[n−1]が答え。

357 名前:132人目の素数さん:2006/11/04(土) 18:01:30
>>356正解。
a[n]=1/sin(π/2^n)
もしくは>>356が答

358 名前:132人目の素数さん:2006/11/04(土) 20:26:12
こいつ、ほんとは途中計算が知りたいだけなんだろう。。。


359 名前:132人目の素数さん:2006/11/04(土) 22:58:26
どうして>>356は良くて>>354がダメなのはなんで?


360 名前::2006/11/05(日) 02:23:02
(x-a)(x-b)(x-c)・・・(x-y)(x-z)=?
最近本で見た問題。

361 名前:132人目の素数さん:2006/11/05(日) 02:24:31
面白くないからそれ

362 名前::2006/11/05(日) 02:30:57
答えは?

363 名前:132人目の素数さん:2006/11/05(日) 02:34:11
0
あと問題にするならせめて∫[0,1](x-a)(x-b)(x-c)・・・(x-y)(x-z)dxを求めよ
とかにしないと

364 名前::2006/11/05(日) 02:44:29
すいませんでした。でも友人はほぼ解けなかった。

365 名前:132人目の素数さん:2006/11/05(日) 03:17:47
>>268
を見てみような 4日前に出題済みだ。

366 名前:132人目の素数さん:2006/11/05(日) 04:46:54
>>360
(x-x)(=0)を掛けているので0

367 名前:132人目の素数さん:2006/11/05(日) 06:56:22
1/tan(pi/8)−1=(1+(1/tan(pi/8))^2)^.5にならないだろ、いいかげんな言い逃れをしやがって
2乗の位置を間違えてる。。

368 名前:132人目の素数さん:2006/11/05(日) 07:05:43
>>367
その計算式が間違ってる。正しくは
1/tan(pi/16)−1/tan(pi/8)=(1+(1/tan(pi/8))^2)^0.5

369 名前:132人目の素数さん:2006/11/05(日) 07:17:58
1/tan(pi/8)−1=(1+(1/tan(pi/4))^2)^.5

370 名前:132人目の素数さん:2006/11/05(日) 08:01:59
α、b、cを3辺の長さとする三角形がある。
条件
 α3(b−c)+b3(c−α)+c3(αーb)=0
が成り立つとき、この三角形はどんな三角形か。



371 名前:132人目の素数さん:2006/11/05(日) 08:23:00
>>370
二等辺三角形
a^100(b−c)+b^100(c−α)+c^100(αーb)=0
では?

372 名前:132人目の素数さん:2006/11/05(日) 08:26:46
正三角形

373 名前:132人目の素数さん:2006/11/05(日) 08:27:51
 α^3(b−c)+b^3(c−α)+c^3(αーb)=0

374 名前:132人目の素数さん:2006/11/05(日) 08:33:23
直角三角形、鈍角三角形、鋭角三角形、ほかになにがある?

375 名前:132人目の素数さん:2006/11/05(日) 08:59:51
2n×2n個のます目をもつ碁盤を考える。碁盤の1つのます目(正方形)
の1辺の長さを1単位の長さとする。この碁盤の上に、直径が(2n−1)
の円を描く。円の中心は碁盤の中心と一致するものとする。次の図はn=2
の場合である。下の問い(問1〜問3)に答えよ。

問1
n=3のとき、円周は何個のます目を通過するか。

問2
一般のnに対して、円周は何個のます目を通過するか。

問3
一般のnの場合、円内に完全に含まれるます目の数をf(n)とするとき、

π(n−1/2)2ー8(n−1/2)≦ f(n)≦ π(n−1/2)2

となることを示せ。



376 名前:132人目の素数さん:2006/11/05(日) 22:59:22
一辺が4の正方形の内部または周上に、n個の点をとる(n≧2)。ただし、どの2点間の距離も
√2以上になるようにする。nの最大値を求め、その理由も説明せよ。

377 名前:132人目の素数さん:2006/11/06(月) 20:12:28
各アルファベットについている色のイメージ。

赤…a
青…p,q,s,w,z
黄…b,i,j,l,r,u,v,y
黒…e,k,x
白…c,h,o
灰…f,n
茶…m,t
?…d,g

みんなはどう?

378 名前:132人目の素数さん:2006/11/07(火) 00:37:17
>>375
1)
 2)を見よ

2)
 8(n-1/2)

3)
 f(n)は円の面積{π(n-1/2)^2} 未満。
 円周の通過する正方形の面積の合計は8(n-1/2)なのでf(n)+8(n-1/2)は円の面積より大きい。
  

379 名前:378:2006/11/07(火) 00:54:04
2) の概略。
全体を田の字に分割し、左上の部分だけを考える。

円周(1/4の円弧)は(0,0)のマスから(n,n)のマスまでを通る。

円弧は単調増加なので(グラフが引き返すようなことはないので)
このようなグラフは(0,0)から(n,n)までに右方向にn-1マス分、
上方向にn-1マス分の移動がある。つまり通過するマスは2(n-1)+1。

その例外はグラフが格子点を通るときであるが、
円弧の半径はn-1/2であることを考えると、それが格子点を通過することはない。
(もし格子点を通過するならば、三平方の定理より半径の2乗が整数である必要がある)


380 名前:132人目の素数さん:2006/11/07(火) 09:37:27
dを自然数とする。
自然数a1,a2,…,an及び自然数k1,k2,…,knがあって、各iに対して1≦ai≦kiを
満たしているとする。a1〜anの値をそれぞれ変化させるとき、Σ[i=1〜n]aiが
dの倍数になるのは何通りあるか。

381 名前:132人目の素数さん:2006/11/07(火) 21:43:57
>>243
>>250
ってどう解くの?
ログにしても大小わかんなくねw

382 名前:132人目の素数さん:2006/11/07(火) 21:53:58
>>381
それはお前の頭が悪いから

383 名前:132人目の素数さん:2006/11/07(火) 21:54:22
>>380 意味わからん。

384 名前:132人目の素数さん:2006/11/07(火) 22:07:41
>>380
条件少なすぎ。
問題写し間違えてないか?

385 名前:132人目の素数さん:2006/11/07(火) 22:16:29
>>382
おすえてください

386 名前:132人目の素数さん:2006/11/07(火) 22:47:36
e^π とπ^eの大小関係について。

0<x,yの時
x^y > y^x
ylog(x) > xlog(y)
log(x)/x > log(y)/x

従って、e^π とπ^eの大小関係を論じるためには
log(e)/e と log(π)/πの大小関係を論じればよい。

f(x)=log(x)/x と置いて、f'(x)を計算すれば
f'(x)=log(x)*(-1/(x^2)) + 1/(x^2)
=(1-log(x))/(x^2)
となり、x≧eの時、f(x)は単調減少関数。このため、
log(e)/e > log(π)/π

387 名前:132人目の素数さん:2006/11/07(火) 22:56:29
>>386
あったまいぃ〜(・∀・)!!
漏れがただ頭悪いだけか・・・
あんがとっ、スッキリしたよw39〜

388 名前:380:2006/11/08(水) 01:51:48
流石にキツイか(^ ^;元の問題を載せておきます。

自然数a1,a2,…,a10は1≦ai≦6 (i=1〜10)を満たしているとする。a1〜a10の値を
それぞれ変化させるとき、(−1)^Σ[i=1〜10]ai=1が成り立つのは何通りあるか。

これが元の問題。なんで「 (−1)^Σ[i=1〜10]ai=1 」という回りくどい表現を
とっているのかを考えたら、解法が見えました。

389 名前:132人目の素数さん:2006/11/08(水) 07:50:03
3*6^9か?

390 名前:132人目の素数さん:2006/11/08(水) 15:25:13
>>386
その問題いいね。詩的で。
よく知られている超越数を二つ使っているところが上手いのかな。

391 名前:132人目の素数さん:2006/11/08(水) 16:53:48
まあπはeより大きな数だったら本来何でもいいわけだが

392 名前:132人目の素数さん:2006/11/08(水) 16:55:00
>>390
言っておくが、大学受験レベルの常識だぞこれ。

393 名前:132人目の素数さん:2006/11/08(水) 16:58:31
よくある有名問題だな

394 名前:132人目の素数さん:2006/11/08(水) 20:06:26
教育的だね。
高校でもこういうの教えればいいのに。

395 名前:132人目の素数さん:2006/11/08(水) 20:20:03
高校で教わったのだが。

396 名前:132人目の素数さん:2006/11/08(水) 21:08:46
そりゃあ、対数の計算方法くらいは教わるんだろうけど。

397 名前:132人目の素数さん:2006/11/08(水) 21:57:58
だから、まんま この問題を高校でやったのだが。解答も>>386と同じ。

398 名前:132人目の素数さん:2006/11/08(水) 22:52:20
「やったのだが」って言われてもw 確かめようがないからなぁ。
まあ、こういう面白い問題を授業で紹介したのなら、いい先生ではあるな。

399 名前:132人目の素数さん:2006/11/08(水) 23:16:16
よく高校の数IIIの参考書とか問題集に載ってるよ

400 名前:132人目の素数さん:2006/11/09(木) 00:13:26
てかe知ってたらわかるだろ。

401 名前:132人目の素数さん:2006/11/09(木) 01:28:40
>>398
低脳バカ高校乙。

402 名前:132人目の素数さん:2006/11/09(木) 07:57:43
超有名問題だろ。何をいまさらって感じだが。
>>386の解答は間違ってるけどな。
0<x,yの時
x^y > y^x
⇔ylog(x) > xlog(y)
⇔log(x)/x > log(y)/x
こう書かないとダメだぞ。

403 名前:132人目の素数さん:2006/11/09(木) 08:23:52
>>397
やべ。まんこの問題を高校でやったのだが。に見えた。

404 名前:132人目の素数さん:2006/11/09(木) 09:12:35
>>402
もう1つ、>>386には誤植もあって正しくは最後の行
 ⇔log(x)/x > log(y)/y
にしといてくれ。

405 名前:132人目の素数さん:2006/11/09(木) 09:20:15
>>404
死ね

406 名前:132人目の素数さん:2006/11/09(木) 09:43:30
何だ?
ファビョる相手を間違えてないか?

407 名前:132人目の素数さん:2006/11/09(木) 09:49:50
煽り合いツマンネ

408 名前:132人目の素数さん:2006/11/09(木) 11:40:41
煽り合ってないないw

409 名前:132人目の素数さん:2006/11/09(木) 14:02:46
>>405
必死だな。 >>386さんよぉ!

( ゚∀゚)アハハ八八ノヽノヽノヽノ \ / \/ \

410 名前:132人目の素数さん:2006/11/09(木) 16:47:41
教科書では見た事無いなぁ・・・
補習かなんかで参考書でみたんじゃないの?

411 名前:132人目の素数さん:2006/11/09(木) 19:31:52
>>410
教科書だけしかやらない低脳バカ高校乙。

412 名前:132人目の素数さん:2006/11/09(木) 19:49:13
>>372
二等辺三角形だよ

413 名前:132人目の素数さん:2006/11/09(木) 21:58:49
>>410
ハァ? 教科書? ( ´,_ゝ`)プッ
m9(^Д^)プギャー

414 名前:132人目の素数さん:2006/11/10(金) 03:41:32
>411,413
言うねぇw自信たっぷりだねぇww

 さいころをn回ふるとき、n回までに少なくとも一回1の目が出る確率は
 1-(5/6)^nで表される

これ説明してくれない?
いまいち良く分からないから・・・

415 名前:132人目の素数さん:2006/11/10(金) 04:15:09
>>414
そんな問題は このスレには書かれていないはずだが?

こっち行け低脳。スレ違いだから。そして二度と戻って来るな。
ttp://science4.2ch.net/test/read.cgi/math/1162905141

416 名前:132人目の素数さん:2006/11/10(金) 08:30:58
sage

417 名前:132人目の素数さん:2006/11/10(金) 15:08:38
n個の都市x1〜xn∈R^2に対してサラリーマンが全ての都市を1度は通る最短の経路の道のりをA(x1〜xn)とする
max[x1〜xn∈S^2](A(x1〜xn))を求めよ

418 名前:132人目の素数さん:2006/11/10(金) 17:42:30
問題がおかしい

419 名前:132人目の素数さん:2006/11/10(金) 20:48:13
S^2ってなんだ。

420 名前:132人目の素数さん:2006/11/11(土) 03:45:28
普通は球のことだがこの問題ではなあ

421 名前:KingOfUniverse ◆667la1PjK2 :2006/11/11(土) 09:04:44
動点Pは座標(x,y)にあるとき、加速度は(sin(x)/(1+sin(x)^2),-sin(x)^2/(1+sin(x)^2))である。
時刻tにおける動点Pの座標を(x(t),y(t))とし、(x(0),y(0))=(0,1), 時刻0での速度を(v,0)とする。
動点Pの軌跡を求めよ。

422 名前:417:2006/11/11(土) 09:21:27
間違えた
S^2じゃなくてS^1やった
{(x,y)∈R^2|x^2+y^2=1}の事ね

423 名前:132人目の素数さん:2006/11/11(土) 09:56:20
じゃあただの
内接正n角形の1辺の長さ×(n-1)
じゃないの?

424 名前:132人目の素数さん:2006/11/11(土) 13:23:58
S^2だと内接正(n-2)角形の一辺の長さ×(n-3)+大円/2か?

425 名前:132人目の素数さん:2006/11/11(土) 13:26:07
いや違うか…?

426 名前:132人目の素数さん:2006/11/11(土) 17:05:01
S^2解けたらノーベル賞もの

427 名前:132人目の素数さん:2006/11/11(土) 17:58:15
ノーベル笑

428 名前:132人目の素数さん:2006/11/12(日) 06:49:54
Cのジョルダン閉曲線Γのうち、次の2つの条件を満たすものを考える。
(1)Γは有限個の格子点p1,p2,…,pmを結んだ線分から成る折れ線である。
(2)各線分は、x軸に平行であるか、またはy軸に平行である。
このとき、C−Γは2つの連結成分から成ることを示せ。

429 名前:132人目の素数さん:2006/11/12(日) 20:30:20
>>410
俺も見たことないなあ。
参考書の「研究課題」とかで、欄外で紹介されるコラム的な問題のような。

430 名前:132人目の素数さん:2006/11/12(日) 21:39:49
>>429
(゚Д゚)≡゚д゚)、カァー ペッ!!

431 名前:132人目の素数さん:2006/11/12(日) 23:18:01
どこがだよw
普通に例題として載ってるような問題だろ。

432 名前:132人目の素数さん:2006/11/12(日) 23:21:39
チャートに類題載ってるじゃん。
3^πとπ^3の大小比較。

433 名前:132人目の素数さん:2006/11/12(日) 23:23:57
>>428
あのな、分からない問題は質問スレにだせよ、馬鹿!

>>386
低レベルな受験数学の問題は受験板に行け、もしくは自身が逝け!

434 名前:132人目の素数さん:2006/11/12(日) 23:41:50
>>429
>>410
誰も教科書に載ってるなんて言ってないんだが
いや乗ってるのもあるかもしれんけど

435 名前:132人目の素数さん:2006/11/13(月) 00:11:23
>>433
何で、回答者に当たってるんだ。
受験数学とかじゃなくて、単に質問がうっとーしーから答えただけだろ。

436 名前:132人目の素数さん:2006/11/13(月) 00:20:03
自分にも分かる問題だから叩く方も調子に乗ってるみたいだねw

437 名前:132人目の素数さん:2006/11/13(月) 00:30:27
>>433
「分からない問題」じゃなくて、「面白い問題」なのだが。ジョルダン閉曲線定理の
簡易版。相変わらずこのスレは、ちょっと解析っぽい問題になるとすぐに宿題扱いだな。

438 名前:132人目の素数さん:2006/11/13(月) 01:58:51
>>429>>410

439 名前:132人目の素数さん:2006/11/13(月) 02:08:10
>>434
乗ってる?教科書の上に問題が乗っかってるのか?

脳味噌沸いてる馬鹿表現だな

440 名前:132人目の素数さん:2006/11/13(月) 02:10:51
その煽りは流石に下らなすぎる

441 名前:380:2006/11/13(月) 05:14:39
f(x)=Π[i=1〜n](x^1+x^2+…+x^ki)=Σ[a∈A]x^(a1+a2+…+an)とおく。ただし
A={a:{1,2,…,n}→N|1≦ai≦ki for i=1〜n}とおいた。これをさらに変形して、
Σ[a∈A]x^(a1+a2+…+an)=Σ[r=0〜d−1]Σ[a∈A, a1+…+an≡r (mod d)]x^(a1+a2+…+an)
=Σ[r=0〜d−1]Tr(x)とする。ただしTr(x)=Σ[a∈A, a1+…+an≡r (mod d)]x^(a1+a2+…+an)と
おいた。ω=e^(2πi/d)とするとき、k∈Zに対してTr(ω^k)=Σ[a∈A, a1+…+an≡r (mod d)]ω^{k(a1+a2+…+an)}
=Σ[a∈A, a1+…+an≡r (mod d)]ω^{kr}=ω^(kr)Pr となる。ただしPr=Σ[a∈A, a1+…+an≡r (mod d)]1
=「a1+…+an≡r (mod d)が成り立つa∈Aの個数」とおいた。このとき
f(ω^k)=Σ[r=0〜d−1]Tr(ω^k)=Σ[r=0〜d−1]ω^(kr)Pr …*
となるので、k=0,1,…,d−1を*に代入し、得られたd個の式を全て足し合わせることでΣ[k=0〜d−1]f(ω^k)=dP0と
なるので、P0=Σ[k=0〜d−1]f(ω^k)/dとなる。P0=「a1+…+an≡0 (mod d)が成り立つa∈Aの個数」であり、これが
求める個数であった。以上より、Σ[k=0〜d−1]f(ω^k)/d が答えとなる。

442 名前:132人目の素数さん:2006/11/13(月) 20:25:27
n人(n≧6)でジャンケンを1回するとき、次の3条件を全て満たす確率を求めよ。
・2人以上はグーを出す。
・2人以上はチョキを出す。
・2人以上はパーを出す。
ただし、どの人間についても、グー・チョキ・パーを出す確率は同様に確からしく1/3とする。

443 名前:KingOfUniverse:2006/11/14(火) 08:40:20
talk:>>439 何やってんだよ?

444 名前:KingOfUniverse ◆667la1PjK2 :2006/11/14(火) 10:14:11
talk:>>443 お前誰だよ?

445 名前:132人目の素数さん:2006/11/16(木) 04:32:45
>>441
>f(ω^k)=Σ[r=0〜d−1]Tr(ω^k)=Σ[r=0〜d−1]ω^(kr)Pr …*
>となるので、k=0,1,…,d−1を*に代入し、得られたd個の式を全て足し合わせることで
>Σ[k=0〜d−1]f(ω^k)=dP0となるので、

てくだりは
Σ[k=0〜d−1]Σ[r=0〜d−1]ω^(kr)P_r=dP_0
てこと?ここがようわからんです。

446 名前:132人目の素数さん:2006/11/16(木) 08:15:23
>>445
そこは計算を省いてしまいました。
Σ[k=0〜d−1]f(ω^k)
=Σ[k=0〜d−1]Σ[r=0〜d−1]ω^(kr)P_r
=Σ[r=0〜d−1]Σ[k=0〜d−1]ω^(kr)P_r (kとrを入れ替える)
=Σ[r=0〜d−1]{P_rΣ[k=0〜d−1]ω^(kr)}
=dP_0+Σ[r=1〜d−1]{P_rΣ[k=0〜d−1]ω^(kr)}
=dP_0+Σ[r=1〜d−1]{P_r*0} (∵1≦r≦d−1のときΣ[k=0〜d−1]ω^(kr)=0)
=dP_0
となります。

447 名前:KingOfUniverse ◆667la1PjK2 :2006/11/16(木) 16:26:24
(1)平面上に八点があり、どの三点も同一直線上にないものとする。
さて、直角に交わる二直線を適当に選ぶと、八点はどちらの直線についても線対称であるとする。
その八点を通る楕円は存在するか?
(2)平面上に八点があり、どの三点も同一直線上にないものとし、
直角に交わる二直線を適当に選ぶと、八点はどちらの直線上にも無く、
どちらの直線についても線対称であるとするとき、その八点を通る楕円が存在することを証明せよ。

448 名前:132人目の素数さん:2006/11/16(木) 19:40:00
(±1,±1),(±2,±3)。


449 名前:KingOfUniverse ◆667la1PjK2 :2006/11/17(金) 06:08:18
つまり、面白い問題。

450 名前:132人目の素数さん:2006/11/17(金) 06:26:57
talk:>>447 お前に何が分かるというのか?
talk:>>449 何やってんだよ?

451 名前:KingOfUniverse ◆667la1PjK2 :2006/11/17(金) 07:02:25
talk:>>450 お前の問題はどこだ?

452 名前:132人目の素数さん:2006/11/17(金) 07:09:42
talk:>>451 人の脳を読む能力を悪用する奴を潰す方法を述べよ

453 名前:132人目の素数さん:2006/11/17(金) 08:43:33
教科書では見た事無いなぁ・・・
補習かなんかで参考書でみたんじゃないの?

454 名前:132人目の素数さん:2006/11/17(金) 08:44:34
分母が2桁の整数である分数のうちπの値に最も近いものを求めよ。

455 名前:132人目の素数さん:2006/11/17(金) 13:00:01
10π/10

456 名前:132人目の素数さん:2006/11/17(金) 13:43:23
それも正解といいたいところだけど、整数比で表される分数ということでよろしく。

さらに一般化した問題。
任意の実数aと任意の正整数nが与えられたとき、分母がn以下の整数である分数のうちaに最も近いもの
を求める効率的な方法を求めよ。

457 名前:132人目の素数さん:2006/11/17(金) 22:55:22
>>456  邦書ではやっぱり高木貞治の「初等整数論講義」がとっても
良い、と思います。結構実用でも役にたったりする、連分数。手でグラフ書くときに便利。
(実験の授業担当してるので)

と、言う訳で数学愛好家の物理専門家なのですが、きちんと計算した訳ではなく有効数字3ケタ
レベルの観察で思いついた事なのでデタラメだったらご免なさい、以下問題。

cos[2 \theta_n +\phi_{n-1}]=sin[\theta_n]、\phi_n=\phi_{n-1}+\theta_{n-1}
で、\phi_0=0の場合、\thetaの答えは常に\piの有理数で与えられる。

458 名前:132人目の素数さん:2006/11/18(土) 18:16:09
(1)gcd(n,6)=1のとき、n^8−n^4+1のどんな素因数も24で割ると1余ることを示せ。
(2)24で割ると1余る素数が無限に存在することを示せ。

459 名前:132人目の素数さん:2006/11/18(土) 18:21:13
ありゃ?gcd(6,n)=1は必要ないか。

訂正
(1)gcd(n,6)=1のとき、n^8−n^4+1のどんな素因数も24で割ると1余ることを示せ。

(1)自然数nに対して、n^8−n^4+1のどんな素因数も24で割ると1余ることを示せ。

460 名前:132人目の素数さん:2006/11/18(土) 18:22:22
>>458
上はn≡±1

461 名前:132人目の素数さん:2006/11/19(日) 16:00:09
>454
1桁   22/7 = π + 1.26448926734968…×10^(-3),
2桁  311/99 = π - 1.78512175651679…×10^(-4),
3桁 355n/113n = π + 2.66764189404967…×10^(-7).

462 名前:132人目の素数さん:2006/11/19(日) 16:15:32
>461

祖沖之(429-500) は πの近似値として
 約率: 22/7
 蜜率: 355/113
を求めたらしい。『隋書』の「律暦志」による。

463 名前:132人目の素数さん:2006/11/19(日) 17:26:53
>>386
 e^x > 1+x (x≠0) を使う。
 a>0,a≠e のとき (a/e)-1=d とおくと
 e^(a/e) = e・e^d > e・e^d > e(1+d) = a,
 e^a > a^e.

>391
 e以外の正数なら…

464 名前:132人目の素数さん:2006/11/19(日) 17:33:12
>386

改造したくなるのが不等式ヲタの…
 ( ゚∀゚)つ 「e^e < 3^e < e^3 < π^e < e^π < 3^3 < π^3 < 3^π < π^π を示せ。」

不等式への招待2
 http://science4.2ch.net/test/read.cgi/math/1105911616/540-548,551-553

出題(不等式)
 http://messages.yahoo.co.jp/bbs?action=m&board=1835554&tid=bdpbja1jiteybc0a1k&sid=1835554&mid=377

465 名前:132人目の素数さん:2006/11/20(月) 12:37:29
両面が赤のカード:2枚
両面が青のカード:2枚
表が赤で裏が青のカード:3枚
これらを中の見えない袋に入れ、1枚取り出し片面だけを見る。
そして、裏が何色なのかを当てるゲーム。
見えた色が赤だったら、どっちに賭ける?

466 名前:132人目の素数さん:2006/11/20(月) 12:53:19
>>465
青。

467 名前:132人目の素数さん:2006/11/20(月) 13:10:28
>>466
俺と賭けをしないか?

468 名前:132人目の素数さん:2006/11/20(月) 13:45:28
>>467
私の計算が間違っているのか?

469 名前:132人目の素数さん:2006/11/20(月) 13:57:51
>>468
じゃあ、袋から1枚引くとき、裏表同じ色のカードを引くか、裏表が違う色のカードを引くかで賭けをしたらどっちに賭ける?

470 名前:132人目の素数さん:2006/11/20(月) 15:02:33
>>469
両面同じ色に決まってるだろうが、ダボが!

471 名前:KingOfUniverse ◆667la1PjK2 :2006/11/20(月) 15:29:28
talk:>>466 無作為に選ぶときは赤になる確率の方が高いから、ディーラーがそれを逆手にとって、こっそり両面が違うカードを選びやすくしているだろう、という推測からその答えになったのか?

472 名前:132人目の素数さん:2006/11/20(月) 15:29:32
(・∀・) ニヤニヤ…

473 名前:132人目の素数さん:2006/11/20(月) 15:40:32
>>470
じゃあ、もう一回>>465のゲームで見えた色が青だったら、どっちに賭ける?

474 名前:132人目の素数さん:2006/11/20(月) 16:13:15
>>473
君の意見を聞こうか?

475 名前:132人目の素数さん:2006/11/20(月) 16:19:38
           _______
           |    ̄ ̄ ̄ ̄   |
           {`   r'" ̄ ̄  ´}
           |  /       |  条件付期待値という言葉を知っているかね、オービーくん?
          _l―‐------- ―‐l  
     =ニ二 ̄   ̄ ̄ ̄ ̄ ̄ ̄ ̄  ̄ 二ニ=  
         ̄ フ テ6〒 '' ーrrrァ、''ミl ̄
         く  こ ノ  ミシ リミミ.ヽ       
            {{lll、     、シ`‐'lミミミ}
         __}⊇ミ.、  .、シ   >''´ _}_
          / (}.}.}.l.l.l.l.l、シ__ /  //   `ヽ
       /   〃 ̄/ ̄_____ / /      ヽ

476 名前:132人目の素数さん:2006/11/20(月) 16:23:58
    
|  |                       オービー デモ バービー デモナイッ!
|  |∧_∧         
|_|;´・ω・`)             ヽ(`Д´;)ノ   ダービー ダ! オボエテオケッ!
|茶| o  ⊃               (  )へ
| ̄|―u'                  く
""""""""""""""""""""""""""""""""""""

477 名前:132人目の素数さん:2006/11/20(月) 16:56:00
>>473
赤。


478 名前:132人目の素数さん:2006/11/20(月) 17:15:04
>>477
俺と賭けをしないか?

479 名前:132人目の素数さん:2006/11/20(月) 17:58:59
赤 478が477に百万円払う
青 477が478に百十円払う


480 名前:132人目の素数さん:2006/11/20(月) 18:13:52
>>478-479
ok

481 名前:132人目の素数さん:2006/11/20(月) 18:33:35
>>479
期待値に差があり過ぎw

482 名前:132人目の素数さん:2006/11/20(月) 19:25:08
マジレスでも頂点に立つ俺様が、この条件付確率を晒そう!
 赤 : 4/7
 青 : 3/7


 ―┼‐         ノ     /   |  --ヒ_/     /   \ヽヽ    ー―''7
   `」   ┼,   二Z二   レ   /  /´レ' \ ―7 ̄}  |  ー-、   /
 (__  (|フ)   (__ノ  _ノ  ∨`  ノ  /  /     _ノ    \_

    ─┼-        /   |   ‐┼-   |     ー|―
    ─┼─ |   \ レ  /   ̄Tー  /      ノ -─
   (二フヽ  \/    _ノ   (二フ\  ヽ_ノ   / 、__

     i';i
    /__Y
     ||真||                   /⌒彡
  _ ||露||         /⌒\     /冫、 ) ・・・・・・。
  \ ||  || ̄ ̄ ̄ ̄ ̄ ̄ ̄\ `./⌒ i `  /ゝ    _,,..,,,,_
  ||\`~~´  (キムチ)       \( >     ('\\  ./ ,' 3 `ヽーっ ・・・・・・。
  ||\|| ̄ ̄ ̄ ̄ ̄ ̄ ̄ ̄ ̄|| ̄\`つ    ⌒ _) l   ⊃ ⌒_つ
     .|| ̄ ̄ ̄ ̄ ̄ ̄ ̄ ̄ ̄||                `'ー---‐
( 'A) ・・・。 〃∩ ∧_∧        <⌒/ヽ___
/(ヘ)ヘ    ⊂⌒(  ・ω・) ・・・。  <_/____/ zzzz・・・
         `ヽ_っ⌒/⌒c

483 名前:132人目の素数さん:2006/11/20(月) 20:00:45
これは
赤ー赤 2/7
青ー青 2/7
赤ー青 3/7
で見えたのと違うほうを答えるのが有利というわけではないんですか

厨房レベルでは理解できません・・・

484 名前:132人目の素数さん:2006/11/20(月) 20:19:13
>>483
表が赤だったときの、裏も赤である確率=4/7
表が赤だったときの、裏は青である確率=3/7
よって赤に賭けた方がよい。


485 名前:132人目の素数さん:2006/11/20(月) 20:31:46
条件付確率もわからない人多いのね

486 名前:132人目の素数さん:2006/11/21(火) 01:09:24
以前質問スレでも発問したのですが、きちんとした回答が得られなかったので改めて…

1〜kまでの目が均等に出るサイコロをつかってスゴロクをする
ここでプレイヤーがスタート地点からnマス先のマスに
(通過せずに)ピッタリ止まる確率をP(n)とする
(例えば明らかにP(1)=1/k,P(2)=(1/k)+(1/k)P(1)・・・)

P(n)をnの式で表しP=lim[n→]P(n)を求めよ。

直感的にPは2/(k+1)と思うのですが、
(∵サイコロは平均して(k+1)/2の目が出るので)
これがなかなか厳密に示せません。
漸化式を書いてみたものの、手計算で解ける気がしません。
興味のある方是非挑戦してみてください。お願いします。

487 名前:132人目の素数さん:2006/11/21(火) 01:13:24
>>486
俺も、手計算で解ける漸化式じゃないと思うわ

488 名前:132人目の素数さん:2006/11/21(火) 04:18:00
別に漸化式を厳密に解く必要ないでしょ。
max{P(i) ; n≦i≦n+k-1}-min{P(i) ; n≦i≦n+k-1}をΔ(n)とかおくと
nが十分大きいとき、Δ(n+k)≦rΔ(n)、0 < r < 1は定数、
みたいな感じの評価が出来るはずなので
こういうことを使って示せばよい。

489 名前:132人目の素数さん:2006/11/21(火) 10:07:29
>>486
出来た。確かにP=2/(k+1)になる。でも解き方が下手糞すぎる(Pが存在することは、
P(n)の特性方程式の解の性質を調べることで証明し、Pの具体的な値は、P(n)の母関数を
使って変な極限を作って求めた)ので書かない。

490 名前:132人目の素数さん:2006/11/21(火) 17:10:12
Pの存在が分かれば
1-P(n)=Σ[i=1,k-1]((k-i)/k)P(n-i)
(nマス目を通過する確率から)
より両辺n→∞として
1-P=Σ[i=1,k-1]((k-i)/k)P
p=2/(k+1)
でどうでしょう?

491 名前:483:2006/11/21(火) 18:51:41
>>484

>表が赤だったときの、裏も赤である確率=4/7
これが感覚とずれるんですよね、裏が赤は2枚しかないので

要するに
裏と表が同じ確率 =4/7
裏と表が違う確率 =3/7
よって見えた色と同じものを答えたほうが有利、ということでしょうか

算数からやり直すかな

492 名前:132人目の素数さん:2006/11/21(火) 19:00:52
>>491
条件付確率を勉強しる

493 名前:132人目の素数さん:2006/11/22(水) 01:59:09
>>491
両面赤のカードを引いた場合、表が見えてる場合と裏が見えてる場合があるでしょ。
赤は全部で2*2+3=7面ある。

494 名前:483:2006/11/22(水) 15:20:51
なるほど、面ですか
ありがとうございます

495 名前:132人目の素数さん:2006/11/24(金) 00:00:50
>>428
・弧状連結成分が2個以下である事の証明
Γと点Aでのみ交わる線分BCを用意する (BA、CAは十分短くする)
これはA≠p1,...,pmとして、またΓに垂直になるようにBCを取ればよい
次に任意の点X∈R^2-Γに対しXから1番近いΓの点をYとする

X → Yに十分近い点 → Γに十分近い部分をΓに沿って通る
→ 線分BCにぶつかる → BかCの近い方へ
というルートを通る曲線aはΓと交わらない。よって任意の点XはBかCと弧上連結

・連結成分が2個以上である事の証明
R^2-Γ上の関数f(x,y)を次のように定義する
点(x,y)から左45度の方向へ伸ばした半直線bとΓの交点が偶数ならf=-1、奇数ならf=1と。
bの取り方によりbはΓと有限個の点のみを共有するからfは上手く定義出来る
f(x,y)が連続関数である事はbとΓの交点の個数nについての帰納法によって証明する
{(x,y)|f(x,y)=-1}も{(x,y)|f(x,y)=1}も空でない閉集合だから
連結成分が1個と言う事は無い

曲線aの取り方の詳細と帰納法の証明部分はまた今度書けばいいか…

496 名前:132人目の素数さん:2006/11/24(金) 00:21:43
>>486の一般化。

非負の実数列{an}はΣ[i=1〜∞]ai=1,Σ[i=1〜∞]iai<∞を満たすとする。i∈Nに対し、
iの目が出る確率がaiであるサイコロをサイコロをつかってスゴロクをする 。プレイヤーが
スタート地点からnマス先のマスにピッタリ止まる確率をp[n]とする。このとき
lim[n→∞]p[n]=1/(Σ[i=1〜∞]iai)となる。

…これが正しいか否かは分からない。

497 名前:132人目の素数さん:2006/11/24(金) 00:31:39
>>495
>bの取り方によりbはΓと有限個の点のみを共有するからfは上手く定義出来る
bの取り方によって交点の偶奇が変化しないことを言わなければwell-defined
とは言えないのでは?

498 名前:132人目の素数さん:2006/11/24(金) 00:32:33
訂正。
誤:交点の偶奇が変化しないことを言わなければ
正:交点の個数の偶奇が変化しないことを言わなければ

499 名前:132人目の素数さん:2006/11/24(金) 00:36:07
>>496
正の実数列の方がいい。非負の実数列じゃ明らかに成り立たない。

500 名前:132人目の素数さん:2006/11/24(金) 00:41:09
>>499
>非負の実数列じゃ明らかに成り立たない。
どのへんが明らかなの?

501 名前:132人目の素数さん:2006/11/24(金) 00:42:04
a2=1

502 名前:132人目の素数さん:2006/11/24(金) 00:58:14
>>497
「bの取り方を変化させる事により」じゃなくて
「"点(x,y)から左45度の方向へ伸ばす"というbの取り方により」って意味で書いた

503 名前:132人目の素数さん:2006/11/24(金) 01:06:13
>>501
あー…本当だ。じゃあ、こうしたらどうかな?

非負の実数列{an}はa1>0,Σ[i=1〜∞]ai=1,Σ[i=1〜∞]iai<∞を満たすとする。i∈Nに対し、
iの目が出る確率がaiであるサイコロをサイコロをつかってスゴロクをする 。プレイヤーが
スタート地点からnマス先のマスにピッタリ止まる確率をp[n]とする。このとき
lim[n→∞]p[n]=1/(Σ[i=1〜∞]iai)となる。

504 名前:132人目の素数さん:2006/11/24(金) 22:35:16
問題: 自然数全体 N から有理数の集合 Q∩[0, 1] への全単射 r
に対して、二進小数表示(aij ∈ {0, 1})を用いて次のように表す:

r1 = 0.a11 a12 a13...
r2 = 0.a21 a22 a23...
r3 = 0.a31 a32 a33...
...

(1) 対角線上の数列で表される 0.a11 a22 a33... は有理数と
なり得るか?

(2) 対角線の一段下の数列で表される 0.a21 a32...a[k+1,k]...
は有理数となり得るか?

505 名前:132人目の素数さん:2006/11/24(金) 23:16:54
>>504
(1) 多分なり得ない。a=0.a11 a22 a33... が有理数ならば 1-a も有理数だが、
これは対角集合にビット反転をかけた数なので、リスト中に出てこない。

(2) b=0.a21 a32...a[k+1,k]...としたとき、1-b=r1 となるように並べる場合に限り、可能なのでは。

スマソ、あまり自信ない。

506 名前:504:2006/11/24(金) 23:26:37
>>505
(1) はそんな感じでok。
(2) はもう少し検討の余地有りかな。

507 名前:132人目の素数さん:2006/11/25(土) 01:25:51
一辺1の正方形を1個のマスとして、n^2個のマスからなる一辺nの正方形を考える。
それぞれのマスを赤or青or黄で隣り合ったマスの色とは異なるように塗り分けるとき、塗り方は何通りか

508 名前:132人目の素数さん:2006/11/25(土) 01:35:44
>>505
a=0.a11 a22 a33... = 0.10000... = 1/2
r2 = 0.10000... = 1/2
1-a = 0.01111... = 1/2

ひとつの有理数に異なる2個の少数表示が存在することがある

509 名前:504:2006/11/25(土) 02:40:09
>>508
やばい。それ考えてなかったわ。出題した俺も答え分からなくなった。
すまん。とりあえず 0 以外は無限に 1 が登場する方の表現を採用しよう。
0.10000... ×
0.01111... ○

510 名前:132人目の素数さん:2006/11/25(土) 03:19:08
>>507
3^(4n)+6^2n(n‐1)
なわけない

511 名前:132人目の素数さん:2006/11/25(土) 03:19:56
>>509
r1 = 0, r2 = 1/2,
k≧3 について a[k,k] = 1 とできる。(証明略)

このとき
0.a11 a22 a33... = 0.01111... = 1/2
は有理数。

512 名前:132人目の素数さん:2006/11/25(土) 08:47:04
>>488
具体的にどうやんの?

513 名前:504:2006/11/25(土) 16:07:36
2 進数に対しては対角線論法を使えないのかorz
俺としては対角線論法を使って、対角部分が有理数に
なり得ないことを示そうとしたんだけど、3 進数以上
ではそうなるよね?

>>511
ミイラ取りがミイラになってしまった。その証明、考え中。

514 名前:132人目の素数さん:2006/11/25(土) 20:33:16
           

515 名前:504:2006/11/25(土) 23:57:44
>>511
ギブアップ。良かったら証明の概略教えて下さい。

516 名前:132人目の素数さん:2006/11/26(日) 13:10:39
n 桁目に 1 が現れる数が無限にあるので、
back and forth method を使うのだろうね。

517 名前:132人目の素数さん:2006/11/26(日) 15:41:35
単純に、上から見ていって条件を満たさないものがあったら
条件を満たすようなものを探してきて順に入れ替えれていけばいいような。

back and forth argumentとかって名前は聞いたことあるんだけど
どういう議論のことを言うのかは知らない。

518 名前:504:2006/11/26(日) 18:13:20
>>517
直感的には、それで行けそうな気がする。でも、疑問は残る。
互換 (p[i],q[i]): N → N (i ∈ N) を考える。このとき、
全単射 r: N → Q∩[0, 1] に置換

τ[n] = (p[n],q[n]) (p[n-1],q[n-1]) ... (p[1],q[1])

を施した写像 r(τ[n](・)): N → Q∩[0, 1] は、また全単射
になる。しかし、n → ∞ のとき、果たして極限写像もまた
全単射になるだろうか?

>>516
その back and forth method は全単射を保証するのかな?
不勉強なもので、調べてみます。

519 名前:132人目の素数さん:2006/11/26(日) 18:30:01
有理数を順に並べる。
r(1)=0,r(2)=1/2。
3≦nのときr(n)をr(i)(1≦i<n)以外で
n桁目が1になる最初の有理数とすればいい。


520 名前:132人目の素数さん:2006/11/26(日) 18:43:51
本調べたら往復論法は載ってたけど
前のほうから読まないといけなさそうなので読む気が起きん。。

「最初の有理数」というのは当然 i が一番小さいという意味だよね。

>>518
何にどういう文字を当ててるか分からんけど
この場合OKでしょ。

条件を満たさないものがあったら、条件を満たすようなもののうち
「一番上にあるもの」と入れ替えていくことにする。入れ替えは上から順に行っていく。
この操作を繰り返して得られる写像をs: N → Q∩[0, 1]とする。
操作をk回繰り返せばs_1からs_kまでは確定するから s は最初の r を
定めればきちんと定義されている。

全写なことと単写なことを別々に確かめればよい。
単写なのは定義から明らか。

全写なのは背理法で示す。sによってr_i∈Q∩[0, 1]に対応する自然数が無かったとしよう。
このようなr_iたち同士の順序(上にあるか下にあるか)は操作によって変わらない。
このような i のうちで最小のものxを取る。
r_xは十分多くの操作(N回とする)が行われた後には
N + 1番目(未定義のr_iたちのなかで一番上)に来ている。
これ以降のM回目(M>N)の操作では必ず第M桁目が0となっていて
「条件を満たさないもの」となっている。
つまりr_xの小数表示はN + 1桁目以降は全て0。矛盾。

521 名前:132人目の素数さん:2006/11/26(日) 18:50:54
>>154
>1,2,3,4・・・nと1からnまでの数字が書かれたカードが1枚ずつ計n枚入っている箱がk個ある。
>このk個の箱のそれぞれからカードを1枚、計k枚取り出す。
>取り出されたカードの数字の和がm以下である確率を求めよ。

求める確率を(n,k,m)とすると、

p(n,k,m)=(Σ[t=0,m-k]Σ[i=0,FLOOR(t/n)]{C(k,FLOOR(t/n)-i)*(-1)^(FLOOR(t/n)-i)*C(k-1+t-n*FLOOR(t/n)+n*i,k-1)})/(n^k).





522 名前:504:2006/11/26(日) 23:50:32
>>520
難しくて時間食ったわ。

>r_iたち同士の順序(上にあるか下にあるか)は操作によって変わらない。

各操作の段階で、順序は変わると思うのだが。
極限操作の結果生成された、はみ出し者(r_i)たちの順序は
r を用いてソートされたものなので、s の構成操作の段階で
決まる?(うごめく)順序とは無関係なはず。そうなると

>これ以降のM回目(M>N)の操作では必ず第M桁目が0となっていて

となる必要もないと思う。

523 名前:132人目の素数さん:2006/11/27(月) 08:01:13
書き忘れたけど>>504でr_iを上から下に並べてあるので上とか下とか書いてます。
つまり分かりにくく書くとr^(-1)とかs^(-1)でNに引き戻したときの順序。
「はみ出し者」はQ∩[0, 1] - s(N)の元という意味。

>>522
いや、はみ出し者r_i1とはみ出し者r_i2があったとしたら、
r_i1とri2の相対的な順序は変わらず、最初r_i1のほうがr_i2より上のほうにあったら
何回操作をしてもr_i1はr_i2より上にあるままで変わらない、ということ。

「条件を満たすようなもののうち一番上にあるもの」がr_1より上にある場合と、
r_2より下にある場合と、r_1とr_2の間にある場合に場合分けして考えてみたら良い。

というか522もそう書いてるような気がするけど…

524 名前:132人目の素数さん:2006/11/27(月) 09:45:26
往復論法は古くは Cantor による実数の順序構造の特徴づけの
証明の前半に、可算で端点のない自己稠密な全順序の一意性と
して現れる。(後半は順序完備化の一意性)
例としては、Q と Q(π) は順序同型になるが、この同型を具
体的に与えるのはかなり難しいと思う。



525 名前:132人目の素数さん:2006/11/30(木) 04:11:34
暇潰し問題 "By Albert Einstein (maybe)"だってさ
http://www.coudal.com/thefish.php
There are five houses in a row in different colors.
In each house lives a person with a different nationality. The five owners drink a different drink,
smoke a different brand of cigar and keep a different pet, one of which is a Walleye Pike.

The question is-- who owns the fish?


Hints:
1. The Brit lives in the red house.
2. The Swede keeps dogs as pets.
3. The Dane drinks tea.
4. The green house is on the left of the white house.
5. The green house owner drinks coffee.
6. The person who smokes Pall Malls keeps birds.
7. The owner of the yellow house smokes Dunhills.
8. The man living in the house right in the center drinks milk.
9. The man who smokes Blends lives next to the one who keeps cats.
10. The Norwegian lives in the first house.
11. The man who keeps horses lives next to the one who smokes Dunhills.
12. The owner who smokes Bluemasters drinks beer.
13. The German smokes Princes.
14. The Norwegian lives next to the blue house.
15. The man who smokes Blends has a neighbor who drinks water.

526 名前:132人目の素数さん:2006/11/30(木) 05:22:51
nを3以上の自然数とする。1辺の長さが2の正n角形Sと半径がrの円Oがある。
r>1/tan(π/n)のとき、OはSに含まれないことを示せ。(こんなの当たり
前のようだが、厳密に証明しようとすると……まあ、それなりに当たり前。)

527 名前:132人目の素数さん:2006/11/30(木) 07:36:24
>>526
含まれる、含まれないの意味がわからん。

528 名前:132人目の素数さん:2006/11/30(木) 10:27:37
はみ出ないように重ねられる→含まれる
どうずらして重ねてもはみ出る→含まれない
で、おけ?

529 名前:132人目の素数さん:2006/11/30(木) 10:52:14
>>528
そういうことです。

530 名前:132人目の素数さん:2006/12/10(日) 13:21:22
N*Nの格子に切られたマスに、M以下の自然数の書かれたコマをおく。
マス一つにコマ一つ、コマはいくつ置いても良い。同じ自然数のコマがあっても良い。
盤面を回転反転して重ねられる場合は同じとして、何通りのコマの置き方があるか。
N、Mを使って示せ。

まったくわかりません。

531 名前:132人目の素数さん:2006/12/10(日) 14:13:37
> マス一つにコマ一つ、コマはいくつ置いても良い。

ひとマスにはコマをひとつだけ置ける。
コマが置いていないマスがあってもよい。

…ってこと?


532 名前:132人目の素数さん:2006/12/10(日) 15:27:35
どの2点間の距離も有理数で,どの3点も一直線上にないように平面上に点はいくつおけるか?

533 名前:132人目の素数さん:2006/12/10(日) 17:14:49
>>532
4つおこうとして早くも挫折中w

534 名前:132人目の素数さん:2006/12/10(日) 17:42:25
>>533
2辺の長さが 3 と 4 の長方形。

535 名前:132人目の素数さん:2006/12/10(日) 18:06:16
長さが2cmの糸を輪にして、原点にとめて、コンパスの
間隔を1/ncmにして糸に引っ掛けて、あちこちまわしてやる。

536 名前:132人目の素数さん:2006/12/10(日) 18:23:53
>>531
マス一つにコマ一つまで、マス全体ににコマはいくつ置いても良い。かなあ・・・。
というか全部のマスに一つずつ置くと決めても、1を置いてない場合と読み替えにするとかで
似たような話に帰結しませんか。

537 名前:132人目の素数さん:2006/12/10(日) 18:49:41
>>532
四つが限界っぽいが……

538 名前:132人目の素数さん:2006/12/10(日) 18:53:12
>>534
んじゃ、その長方形の対角線の中心に次の1点をおいてみたら5点は出来そう?

539 名前:132人目の素数さん:2006/12/10(日) 18:54:30
あっ、1直線上はダメなのか。

540 名前:132人目の素数さん:2006/12/10(日) 22:00:52
確かいくつでも置けるんだよな。円周上にうまく配置していくんだったかな?

541 名前:132人目の素数さん:2006/12/10(日) 22:02:24
一直線上がいいなら等間隔に置けばいくらでも置けるし

542 名前:132人目の素数さん:2006/12/10(日) 22:15:15
>>536
「かなあ・・・。」じゃねぇよクズ!問題文の意味くらい正確に把握して来い!

543 名前:132人目の素数さん:2006/12/10(日) 22:59:37
>>532
与えられたnに対して所望の配置が存在する。
無限個を配置することは出来ない。

544 名前:132人目の素数さん:2006/12/10(日) 23:01:24
>>543
証明キボン

545 名前:132人目の素数さん:2006/12/10(日) 23:02:18
ナ・イ・シ・ョ

546 名前:132人目の素数さん:2006/12/10(日) 23:05:03
有名問題だから、どっかで見たことがある

確かピーター・フラン来るの本に載ってなかったか?

547 名前:132人目の素数さん:2006/12/10(日) 23:38:04
>>540
そうだよね。
トレミーの定理を使うと、点を追加する際すでにある 2 点からの距離が
有理数になりさえすればよいことがわかる。

548 名前:132人目の素数さん:2006/12/11(月) 00:17:23
地味に投下

(X−A)(X−B)(X−C)…(X−Z)
この式の答を求めよ。
※A〜Xは任意の数














答 0
理由 (X−A)…(X−X)…(X−Z)=(X−A)…0…(X−Z)=0

549 名前:132人目の素数さん:2006/12/11(月) 00:29:27
>>548
ガロア拡大と自己同型群を使え

550 名前:132人目の素数さん:2006/12/11(月) 08:02:51
>>542
いや最初のであってますけど、
言い換えないと理解できないのかと思って。

551 名前:132人目の素数さん:2006/12/11(月) 11:34:33
どうしてこの問題は定期的に現れるんだ?

552 名前:132人目の素数さん:2006/12/11(月) 15:09:30
A={(cos 2t, sin 2t) | cos t, sin t∈ Q}

553 名前:132人目の素数さん:2006/12/11(月) 21:44:11
面白い問題以外は持ってくるなよ。質問とか論外だ。

554 名前:132人目の素数さん:2006/12/11(月) 21:47:01
拾ってきました 高さ1mの電柱ってw

底面が半径1の円、高さが1mの電柱が地面に立っている。底面の電柱の中心から2m離れたところに高さ2mの街灯がある。
真夜中に街灯が作る電柱の影の体積Vを求めよ。ただし、障害物はないものとする。

555 名前:132人目の素数さん:2006/12/11(月) 22:50:56
影は平面なので体積はありません。

556 名前:132人目の素数さん:2006/12/12(火) 08:01:09

究極のアホ。

557 名前:132人目の素数さん:2006/12/13(水) 00:06:20
で、底面の半径は1kmなのかね?


558 名前:132人目の素数さん:2006/12/17(日) 08:14:19
1光年でおながいします

559 名前:132人目の素数さん:2006/12/19(火) 18:56:48
>>543
各々の距離が有理数なら>>552にもあるように二次曲線上に置けばいいべ
無限個置くのが無理なのは各々の距離が自然数の場合

560 名前:132人目の素数さん:2006/12/21(木) 23:43:27
小太郎君がふたつの玉をいじっています。
どうやら雛子お姉ちゃんと一緒に遊びたいようなのですが、さて、ここで問題です。
rを正の実数とする。xyz空間内の原点O(0,0,0)を中心とする半径1の玉をA、
点P(r,0,0)を中心とする半径1の玉をBとする。玉Aと玉Bの和集合の体積をVとする。
ただし、玉Aと玉Bの和集合とは、玉Aまたは玉Bの少なくとも一方に含まれる点全体よりなる立体のことである。
V=8になるときrの値はともかくとして二桁の数字で表す男女の営みがありますが、それはなんですか?

561 名前:132人目の素数さん:2006/12/21(木) 23:51:17
かいなし

562 名前:132人目の素数さん:2006/12/22(金) 00:23:48
age

563 名前:132人目の素数さん :2006/12/23(土) 02:39:47
n枚の百円玉と(n+k)枚の500玉を同時に投げたとき、表の出た100円玉の枚数より表の出た500玉の枚数の方が多い確率を求めよ。
大学入試で出たのがk=1の場合だったので一般化してみました。答えは知らぬ存ぜぬ


564 名前:132人目の素数さん:2006/12/23(土) 02:46:38
>>563
もうね、アホガドバナナと

565 名前:132人目の素数さん :2006/12/23(土) 02:52:02
時々おとん てか?

566 名前:132人目の素数さん:2006/12/23(土) 03:01:45
>>565
もっと詳しく!

567 名前:132人目の素数さん:2006/12/23(土) 12:09:40
無限集合から2値集合への写像全体の集合はもとの集合より大きいって問題。



568 名前:132人目の素数さん:2006/12/23(土) 12:16:06
馬Ca*n+kCb/2^2n+k a>b

569 名前:132人目の素数さん:2006/12/23(土) 20:55:33
>>568
このDQNに数式の書き方を叩き込んでいいですかね?

570 名前:素数マニア:2006/12/23(土) 22:15:16
こんなもんだいとける? 
 規則性の問題
10、24、66、336、( )
この数列の規則を説明し、( )の中にはいる数を求めよ。

571 名前:素数マニア:2006/12/23(土) 22:20:21
ミスしました。
336を136にしてください。

規則性の問題
10,24,66,136、( )
この数列の規則を説明し、( )の中にはいる数を求めよ。

572 名前:132人目の素数さん:2006/12/23(土) 22:34:31
10
10、24、66、136
が周期的に訪れる数列

573 名前:132人目の素数さん:2006/12/23(土) 23:26:14
偶数

574 名前:132人目の素数さん:2006/12/23(土) 23:28:41
どうせ等差か頭皮数列しかない、ストかステイックなやつは10年に
いちどぐらいしか出題されない

575 名前:132人目の素数さん:2006/12/23(土) 23:43:41
>>571
答えは、234かな?

規則は7*nで、nは4づつ増えてる。

576 名前:132人目の素数さん:2006/12/24(日) 00:06:44
要するにたかが階差数列が面白い問題であると

577 名前:132人目の素数さん:2006/12/24(日) 01:05:28
A____B
.|   |
.|   |
.|   |
.|   |
D ̄ ̄ ̄ ̄C

正方形ABCDがあります
Aから辺DCに線をひき好転をPとします
∠BAPの二等分線を引き辺BCとの交点を
Qとします(必ず辺BCと交わります)

このときのAP=DP+BQを説明して



578 名前:132人目の素数さん:2006/12/24(日) 01:07:34
× Aから辺DCに線をひき好転をPとします

○ Aから辺DCに線を引き交点をPとします


579 名前:132人目の素数さん:2006/12/24(日) 01:12:55
>>567
全射f:X→2^Xが存在するなら任意の関数g:X→Xは不動点を持つ
不動点を持たない関数g:X→Xを作ればいい

580 名前:132人目の素数さん:2006/12/24(日) 01:40:32
>>571
有限項の数列の一般項など、無数に作れるぞ!

581 名前:132人目の素数さん:2006/12/24(日) 12:20:22
>>577
三角関数使えば大した事はないが、あまり面白くないので敢えて封印する。

∠BAQ=θとする。問題の仮定から∠BAP=2θ。
AB//DCだから∠APD=∠BAP=2θ

DP=EP・・・(1) となる点Eを線分AP上にとる。
△PDEが二等辺三角形だから、
∠PDE=(180°-∠BAP)/2 = 90°-θ
∠ADE=∠ADC-∠PDE=90°-(90°-θ) = θ

DEの延長線とABの交点をFとすると、
∠ADF=∠BAQ=θ、ABCDが正方形だからAD=BA, ∠DAF=∠ABQ=90°
合同条件を満たすから△ADF≡△BAQ
したがって AF=BQ ・・・(2)

∠AFE = 180°-∠FAD - ∠ADE = 90°-θ
∠AEF = 180°-∠AFE - ∠BAP = 180°- (90°-θ) - 2θ = 90°-θ
つまり△AFEはAE=AFの二等辺三角形である。 ・・・(3)

(1)(2)(3)から、
AP = EP+AE = DP+BQ

582 名前:132人目の素数さん:2006/12/24(日) 13:14:14
>>581
俺と違う考えだから合ってるかわからないが

俺が用意した答え








辺DP辺BQが一直線上にくるように図を書く
    A
D'______B
.|  |  |
.|  |  |
.|  |  |
  ̄ ̄ ̄ ̄ ̄ ̄C
C'   B'D


そうすると二等辺三角形が出来るから同じ長さ

言ってることは>>581と同じなのかなぁ

583 名前:132人目の素数さん:2007/01/22(月) 00:49:59
一直線上に、OA=1、OP=a(≠0)を満たす三点O、A、Pがある。
コンパスと目盛りのない定木だけを用いて、長さがa^2となる線分を描け。

方べきの定理を使わずに、中学までの知識でやってください。

584 名前:132人目の素数さん:2007/01/22(月) 08:03:38
>>583
俺は方べきの定理を中学で習ったから、方べきの定理は「中学までの知識」だな。終了。

585 名前:132人目の素数さん:2007/01/22(月) 08:45:21
残念。

「方べきの定理を使わずに」 かつ 「中学までの知識」でやってください


586 名前:132人目の素数さん:2007/01/22(月) 12:24:22
辺の長さが1とaと適当な長さの三角形を作って、その三角形と相似比が1:aになるようにもう一つ三角形を作る

587 名前:132人目の素数さん:2007/01/22(月) 12:27:39
そうそう、それそれ

588 名前:583:2007/01/22(月) 13:08:44
>>586
その方法は思いつきませんでした。
自分の考えたやり方より簡潔で手数も少なくていいですね。

↓自分が考えたやり方
直線をx軸とし、Oを原点として直交するy軸を描く。
y=axとx=aを描くと交点のy座標がa^2となるから…

589 名前:132人目の素数さん:2007/01/22(月) 13:14:57
こういう場合はコンパスで円書いて直交する線を書けるのか?

590 名前:132人目の素数さん:2007/01/22(月) 13:24:42
>>589
書けるでしょ
二等辺三角形かければいいんだから


591 名前:132人目の素数さん:2007/01/22(月) 13:27:58
なるほど、円を二つ書くのか。
コンパスなんて使った記憶ないんだよな。

592 名前:小3♀w:2007/01/23(火) 22:45:44
塾で聞いた話ですが。。。

A子のことをB男とC男がスキだといった。

セリフ  

B「僕の方がCよりもA子を愛してる」
C「僕の方がBよりA子を愛している」

そんなことを言っているとA子が。。。


A「貴方達2人はどっちも私を愛してないわ」

といいました。

これを数学的に説明しなさい。

593 名前:132人目の素数さん:2007/01/23(火) 22:52:00
>>592
上島「じゃあ、僕がA子を愛します」
B,C「どうぞ、どうぞ。」

594 名前:132人目の素数さん:2007/01/24(水) 00:17:22
>>592
どれくらいかは別問題として。
B=100C
C=1000B
(BはCの100倍愛してるということ)
よってB=C=0

595 名前:132人目の素数さん:2007/01/24(水) 23:26:10
問題豆乳

平面上にn個の点からなる集合Aが与えられたとする。Aのどの2点の距離も1より小さければ、
Aを内部に含む半径(√3)/2の円があることを証明せよ。

596 名前:132人目の素数さん:2007/01/24(水) 23:43:47
座標を一つ決めてAの点を(xi, yi)たちとする。
max xi - min xi ≦1、max yi - min yi ≦1だから
Aの点は全てある辺の長さ1の正方形に含まれる。
つまりAを内部に含む半径√2/2の円が存在する。

と思ったけどどっか間違ってるかな。
なんかあまりにも簡単に拡張が証明出来ちゃったから不安。

597 名前:132人目の素数さん:2007/01/25(木) 00:36:15
>>594
なんで比で取る?

598 名前:132人目の素数さん:2007/01/25(木) 02:04:08
>>595
半径(√3)/3の円じゃない?

599 名前:132人目の素数さん:2007/01/25(木) 02:06:42
>>598
半径(√3)/3の円だと、n=4のとき成立しないことがあるから駄目だな。

600 名前:132人目の素数さん:2007/01/25(木) 13:16:18
Aを内部に含む円の半径の最小値も(√2)/2なのかな?←ちょっと表現が変だと思うけどわかってね。

601 名前:132人目の素数さん:2007/01/25(木) 16:52:26
>>600
明らか。

602 名前:132人目の素数さん:2007/01/25(木) 18:06:20
>>601
どして?

603 名前:132人目の素数さん:2007/01/25(木) 18:39:33
>>602
半径がr<(√2)/2のとき、一辺の長さがr√2<1である正方形の頂点の位置に
4点を配置し、これをAとすれば、Aは半径rの円の内部に含まれない。よって
r≧(√2)/2でなければならない。一方、r=(√2)/2のとき、>>596より、Aが
どのような集合であっても、Aは半径rの円の内部に含まれる。

604 名前:132人目の素数さん:2007/01/25(木) 18:43:18
…と書いてみて気づいた。対角線上にある2点は距離が1より大きくなることがあるから間違いだな。
>>601,603は撤回します。

605 名前:132人目の素数さん:2007/01/27(土) 00:22:46
>>599
>半径(√3)/3の円だと、n=4のとき成立しないことがある

例キボンヌ

606 名前:132人目の素数さん:2007/01/27(土) 01:26:24
>>595
半径1、中心角120°の扇形を考える
1辺が1の正三角形を2つくっつけた形のひし形を考える
このひし形は半径√3/2の円内に収まる

こんなのかな

607 名前:132人目の素数さん:2007/01/27(土) 14:27:58
ttp://web2.incl.ne.jp/yaoki/aptc.htm
ここら辺が近いか?

608 名前:132人目の素数さん:2007/01/27(土) 16:10:12
>Footmark
数学掲示板の癌

609 名前:132人目の素数さん:2007/01/28(日) 02:07:21
禿同

610 名前:132人目の素数さん:2007/01/28(日) 09:26:26
同じ半径の円を3個接したとき、真ん中の三角形の面積は?
楕円でもやってみて

611 名前:132人目の素数さん:2007/01/28(日) 10:32:20
三角形なんかないが?

612 名前:132人目の素数さん:2007/01/28(日) 13:36:33
誰か、610を和訳してくれ


613 名前:132人目の素数さん:2007/01/28(日) 14:19:16
.〇
〇〇

こういう配置で中心を結んだ三角形ということだろうか

614 名前:132人目の素数さん:2007/01/28(日) 15:38:06
おそらく
「同じ半径の円3つを互いに他の2つの円に接するように配置する。
3つの円に囲まれた部分(正三角形を円弧で削ったような図形)の面積は?」
という問題だと思う。

615 名前:132人目の素数さん:2007/01/28(日) 15:46:10
それじゃつまんないので、3つの円に囲まれた部分に入る三角形の面積の最大値は?
って問題かと思た。

616 名前:132人目の素数さん:2007/01/28(日) 15:51:40
三つの円の中心を結ぶと正三角形が出来る。
その中に同じ正三角形が4つでき、一辺の長さが半径に等しい。

617 名前:132人目の素数さん:2007/01/28(日) 19:39:52
赤い帽子が3つ、白い帽子が2つあります。それを一直線に並んだA君、B君、C君にランダムにかぶせ、残りは隠しました。3人とも自分より前にいる人の帽子は見えるが自分の帽子は見えません。
そこで1番後ろのC君に自分がかぶっている帽子ね色が分かるかと聞くと「分からない」真ん中のB君に聞くと「分からない」1番前のA君に聞くと「分かった」という。
さぁ、A君のかぶっている帽子の色は赤白どちらか?

618 名前:132人目の素数さん:2007/01/28(日) 19:45:20
>>617
激しく有名問題

619 名前:132人目の素数さん:2007/01/28(日) 19:47:12
第二問。

A君が時速2キロメートルで歩き出しました。その1時間後、B君が時速4キロメートルでA君を追いかけました。
B君が歩き出すのと同時にC君が時速10キロメートルでA君を追いかけ、追いつくと後戻りしてB君のもとへ、B君のもとに戻るとまたA君のもとへと走ります。
これをB君がA君に追いつくまで繰り返しました。結果C君は何キロ走ったことになるでしょうか?

620 名前:132人目の素数さん:2007/01/28(日) 20:25:31
ちなみにこれ小学生でも5秒で解けます

621 名前:132人目の素数さん:2007/01/28(日) 20:27:14
無限級数を使えば暗算で求まる by von Neumann

622 名前:132人目の素数さん:2007/01/28(日) 22:05:51
10km。5秒じゃ問題読み終わらねえ。

623 名前:132人目の素数さん:2007/01/28(日) 22:52:47
じゃあお前は小学生以下だな

624 名前:132人目の素数さん:2007/01/29(月) 01:26:16
以下ってのは、等しい場合も含むので、小学生未満?

625 名前:132人目の素数さん:2007/01/29(月) 10:47:00
数学では等しい場合を含むが、日本語としては含む場合も含まない場合もある。
「もう、これ以上食べられません。」は、含むとすると矛盾してしまう。
小学生以下は難しい。「小学生以下は無料です。」は含むと思われるが、
>>623のような場合は含まないと思われる。

626 名前:132人目の素数さん:2007/01/29(月) 12:21:13
そのばあい
小学生以下に小学生を含もうが含まなかろうが
小学生以下にはお前が含まれます。
よって、623は真です。

627 名前:132人目の素数さん:2007/01/29(月) 13:28:56
>>625
> 数学では等しい場合を含むが、日本語としては含む場合も含まない場合もある。
> 「もう、これ以上食べられません。」は、含むとすると矛盾してしまう。
> 小学生以下は難しい。「小学生以下は無料です。」は含むと思われるが、
> >>623のような場合は含まないと思われる。

アホですか?
「もう、これ以上は…」 のこれは何を指すのかな?ぼうや!

628 名前:132人目の素数さん:2007/01/29(月) 14:08:12
「お年玉ちょうだい」
「お年玉?1000円以上は出せんな。」

629 名前:132人目の素数さん:2007/01/29(月) 14:13:28
>>627

630 名前:132人目の素数さん:2007/01/29(月) 16:49:10
goo辞書によれば、「以上」の意味は

数量・程度などを表す名詞の下に付けて、それより多いこと、また、
優れていることを表す。数量を表す用法では、その基準点を含む。

とある。つまり、「数量」なら含み、「程度」なら含まない。

631 名前:132人目の素数さん:2007/01/29(月) 18:40:36
>>627
頑張れよ

632 名前:132人目の素数さん:2007/01/29(月) 22:45:27
そもそも小学生と言っても複数名居るんで、
入学式に出たばかりの一年生も小学生なら
中学入試を終えてあとは卒業式を残すばかりの六年生も小学生なわけで
どちらも同じ「小学生レベル」だけど、だからと言って両者が同じ水準と言うわけでもなし

633 名前:132人目の素数さん:2007/01/30(火) 01:35:11
>>632
頑張れよ

634 名前:132人目の素数さん:2007/01/30(火) 17:46:48
分からないスレから改変引用

1〜63の自然数から異なる7個を選んでBとする。
このとき、どんなBに対しても、Bの空でない部分集合C,Dで、
以下を満たすものが取れることを示せ。

・ C∩D=φ
・ Cの要素の総和=Dの要素の総和

635 名前:132人目の素数さん:2007/01/30(火) 21:09:30
出来たっぽい。今証明を書いてる。

636 名前:635:2007/01/30(火) 22:38:10
詰まった(´・ω・`)

n≧3のとき、次が成り立つことを、数学的帰納法で示す。

B⊂{1,2,…,2^(n−1)−1},#B=nを満たす任意のBに対して、
Bの空でない部分集合C,Dで、以下の*を満たすものが取れる。
C∩D=φ,Cの要素の総和=Dの要素の総和 …*

…とかやっていたのだが、途中で行き詰まった。検証してみたら、n=4のとき
そもそも上の主張は成り立たない( B={3,5,6,7} )。でもn=3のときは成り立つ。
もしかしたら、元の場合(n=7の場合)も実は成り立たないのかな?あるいは、nが
偶数のときは成り立たなくて、nが奇数だと成り立つとか。ワカラン。

637 名前:132人目の素数さん:2007/01/30(火) 23:09:12
>>634
この問題は成り立たない
反例
B={63,63-1,63-2,63-4,63-8,63-16,63-32}

これが反例になっていることの説明
要素の数が異なっていれば、個数の多いほうが
和が大きくなる。(マイナスは全部あわせても-63だから)
個数が同じときは、2進数の考え方で
和は等しくなりえないことがわかる。



638 名前:634:2007/01/30(火) 23:58:29
正直すまんかった。

引用元
http://science5.2ch.net/test/read.cgi/math/1169473367/821

639 名前:132人目の素数さん:2007/01/31(水) 01:09:26
>>634-638
分からないスレで質問したものです。
ご迷惑おかけしました&ありがとうございます。


640 名前:132人目の素数さん:2007/01/31(水) 12:32:47
問題投下

1〜24の自然数から異なる7個を選んでBとする。
このとき、どんなBに対しても、Bの空でない部分集合C,Dで、
以下を満たすものが取れることを示せ。

・ C∩D=φ
・ Cの要素の総和=Dの要素の総和

641 名前:132人目の素数さん:2007/01/31(水) 12:57:57
>>640
Bの最小の要素をmとする。
Bの(空でない)部分集合の和としてとりうる値を考える。
最小はm
最大は117+m (24+23+..+19+m)
だから高々118種類の値しか取れない。

さて、Bの(空でない)部分集合は2^7-1=127
よって鳩ノ巣の原理で要素の和が等しい部分集合が
少なくとも2つ存在する。

これらが共通の要素を持つ場合は、これを取り去れば
求めるC,Dが得られる。

642 名前:132人目の素数さん:2007/01/31(水) 14:44:09
>>641
24+23+22+21+20+19=129では?
117ってどこからでてくるんでしょう。

643 名前:639:2007/01/31(水) 14:53:14
自分で最初の質問をしておいてなんですが、
Bの要素のうち任意の4個以下の整数を選びその和は
7C1+7C2+7C3+7C4=108通りある。
最大値は24+23+22+21=90
で後鳩ノ巣、でどうでしょう。


644 名前:132人目の素数さん:2007/01/31(水) 17:39:05
nは3以上の自然数とする。B⊂{i∈N|2≦i≦n^2+n−1},#B=n^2を満たすBについて、以下の問いに答えよ。
(1)「Bの異なる3元a,b,cでab=cを満たすものが存在する」が成り立たないBを1つ求めよ。
(2)n^2+n−1がBに含まれなければ、必ず「Bの異なる3元a,b,cでab=cを満たすものが存在する」ことを示せ。

645 名前:132人目の素数さん:2007/02/01(木) 04:57:18
>>634

>>637 と同じようにして
B = {46, 46-1, 46-2, 46-4, 46-7, 46-13, 46-24}
という反例が作れるから、「1〜46の自然数〜」まで成立しない
「1〜45の自然数〜」から真偽不明

646 名前:132人目の素数さん:2007/02/03(土) 21:30:26
(1)A,B⊂{1,2,…,n}は、|A|+|B|≧nを満たすとする。このとき、
∃a∈A∪{0},∃b∈B∪{0} s,t n=a+b が成り立つことを示せ。
ただし、|A|は集合Aの元の個数を表す。

(2)自然数列{an}は、limsup[n→∞]an/n ≦2 が成り立つとする。
A={ak|k∈N}とおくとき、∃M∈N,∀n>M,∃x,y∈A∪{0} s,t n=x+y が
成り立つことを示せ。

647 名前:132人目の素数さん:2007/02/03(土) 21:31:48
訂正。

誤:自然数列{an}は、limsup[n→∞]an/n ≦2 が成り立つとする。
正:自然数列{an}は、limsup[n→∞]an/n <2 が成り立つとする。

648 名前:647:2007/02/03(土) 21:38:38
つ∀`) アチャー。どうしようもないな。

誤:自然数列{an}は、limsup[n→∞]an/n <2 が成り立つとする。
正:狭義単調増加する自然数列{an}は、limsup[n→∞]an/n <2 が成り立つとする。

649 名前:132人目の素数さん:2007/02/04(日) 01:19:26
http://up2.viploader.net/upphp/src/vlphp012414.bmp

これ高校生に出せるように少し改変したんだが面白くない・・??
あ、面白くないよね・・・。

650 名前:132人目の素数さん:2007/02/04(日) 01:45:32
jpgかpngでうp汁

651 名前:132人目の素数さん:2007/02/04(日) 01:51:35
VIPロダのMin制限でムリだった
ほかのロダ使うのマンドクセ

652 名前:132人目の素数さん:2007/02/04(日) 02:43:37
>>649
目の覚めるような方法があるんじゃないかと期待

653 名前:132人目の素数さん:2007/02/04(日) 03:02:55
>>652
目の覚めるような方法は無いが・・・・
ただ、途中でいろんな道具を使うので面白いと言えるんじゃないかと
勝手に思ってるだけ・・。

654 名前:132人目の素数さん:2007/02/04(日) 06:40:17
>>649
数V微積分を習いたての者に丁度よい問題だな。
解法が見え見えなので、数学板のクズ(俺のこと)には面白くはないがな。

まず根号内を平方完成し そこを tan に変換後、加法定理などで整理して積分
sin(π/12)が出てきたので、半角公式でも使って計算。

高校生向けの良問だと思うよ。

655 名前:132人目の素数さん:2007/02/05(月) 18:40:18
507

656 名前:132人目の素数さん:2007/02/09(金) 23:54:40
>>649
流れてしまったか……
再うpキボン

657 名前:132人目の素数さん:2007/02/10(土) 00:04:52
>>656
でも断るッ!

658 名前:132人目の素数さん:2007/02/10(土) 00:44:52
>>656
∫[((√3)-1)/2→1]dx/(x√(x^2+x+1))

659 名前:132人目の素数さん:2007/02/10(土) 03:07:10
s、tを実数とする。
初期値sで一般項がsの有理関数f(s)で表される数列{a(s)_n}がn→∞でtに収束し、
かつ数列{a(t)_n}がn→∞でsに収束する。
s、tを求めよ。

660 名前:132人目の素数さん:2007/02/10(土) 03:56:05
>>659
問題文がわかりにくすぎる

661 名前:132人目の素数さん:2007/02/10(土) 04:02:05
空間内に、2定点 A, B と定直線g上を動く点Pがある。
直線ABとgはねじれの位置にあるとする。
AP+PBが最小となる点Pの位置を説明せよ。

662 名前:132人目の素数さん:2007/02/10(土) 11:27:23
nは2以上の自然数とする。A⊂Z/nZ (単なる集合として)
が#A>(n+1)/2を満たすとき、∃a,b,c∈A s,t a+b=c が
成り立つことを示せ。

663 名前:132人目の素数さん:2007/02/10(土) 11:51:25
>>659
>sの有理関数f(s)で表される数列{a(s)_n}
f(s) と {a(s)_n} の関係がわからん。

664 名前:132人目の素数さん:2007/02/10(土) 12:38:22
すいません、f(s)じゃなくてa(s)です。

665 名前:132人目の素数さん:2007/02/10(土) 12:42:42
それでも意味不明

「一般項が有理関数で表される有理関数列 { a_n } を取る。
 各項に s を代入して得られる実数列は t に収束し、
 各項に t を代入して得られる実数列は s に収束する。
 s と t を求めよ」

ということ?

666 名前:132人目の素数さん:2007/02/10(土) 13:00:07
まあまあ、DQNの言うことは置いといて、次いこう!

667 名前:132人目の素数さん:2007/02/10(土) 13:02:27
>>664
こんどは a(s) と {a(s)_n} の関係がわからん。
F(x)=lim[n→∞]a_n(x)として
F(s)=t, F(t)=s を解けってことか?
こんなもん F(x) が分からないと都県だろ。
たとえば F(x)=x とかだったらどんな s,t でもいいぞ。

668 名前:667:2007/02/10(土) 13:05:07
訂正
誤:たとえば F(x)=x とかだったらどんな s,t でもいいぞ。
正:たとえば F(x)=x とかだったら s=t ならなんでもいいぞ。

669 名前:132人目の素数さん:2007/02/10(土) 13:30:34
>>661
Bをgを軸にして回転させてもPBの距離は変わらない
それを利用して最短距離を求めるにはBをどこに移せばよいか?

670 名前:>659:2007/02/11(日) 01:23:57
すいません。
意味不明なものになってましたね。

改めて…

有理関数f(x)、g(x)が以下を満たすときf(x)、g(x)を求めよ。
x→+∞でf(x)→g(0)かつg(x)→f(0)

671 名前:132人目の素数さん:2007/02/11(日) 03:43:27
>>670
たとえば f(x)=g(x)=0 とかでもいいのか?


672 名前:132人目の素数さん:2007/02/11(日) 06:22:15
>>670
もういい。糞食って寝ろ!

673 名前:132人目の素数さん:2007/02/11(日) 07:11:34
>>670
たくさんありすぎる。死ね。

674 名前:132人目の素数さん:2007/02/11(日) 08:40:47
>>670
ほじくった鼻糞食べて寝ろ!

675 名前:132人目の素数さん:2007/02/11(日) 09:41:07
>>670
とりあえず、君はその問題の、どんなところを
面白いと感じたんだ?感じたのか?

676 名前:132人目の素数さん:2007/02/11(日) 10:00:21
>>670
とりあえずウンコ食って寝ろ!

677 名前:132人目の素数さん:2007/02/11(日) 14:07:04
毎朝=マイアス
朝日=アスヒー


678 名前:132人目の素数さん:2007/02/11(日) 14:53:32
有理関数f(x)、g(x)が以下を満たすときf(x)、g(x)を求めよ。
x→+∞でf(x)→g(0)かつg(x)→f(0)

f=g(0)f(0)/g
g=g(0)f(0)/f

679 名前:>670:2007/02/11(日) 16:51:03
例えば定数関数は題意を満たしますが、
他の場合はあるのかな?って思い、出題しました。

680 名前:132人目の素数さん:2007/02/11(日) 17:12:11
この問題で、関数 f, g を拘束する条件は有理関数である以外に、
原点と無限遠方における 2 値の指定しかないわけだよね。そんな
関数は無数に取れるじゃなの。例えば

f(x) = (1 - x)/(1 + x)
g(x) = -(1 - x)/(1 + x)

681 名前:132人目の素数さん:2007/02/11(日) 19:01:41
{p(x)/q(x) + a|p,q∈R[x]、a∈R、deg p<deg q、p(0)=0、q(0)≠0}

682 名前:132人目の素数さん:2007/02/11(日) 19:09:01
>>679
そんなものくだらんスレに書け。

683 名前:132人目の素数さん:2007/02/11(日) 20:09:06
>>679
ここは質問スレじゃねーんだぞ!
面白い問題を出題するっつーレベルじゃねーぞ!

二度とくるな! (゚Д゚)≡゚д゚)、カァー ペッ!! ペッ!! ペッ!! ペッ!! ペッ!! >>670

684 名前:132人目の素数さん:2007/02/11(日) 20:41:28
まあまあ。そういきり立たずに。
面白くなかったらスルーすれば宜し。

685 名前:132人目の素数さん:2007/02/11(日) 21:15:20
>>679
> 他の場合はあるのかな?って思い、出題しました。

分からんのに出題するって…
それは質問するっていうんじゃないのかね?
「お ・ し ・ え ・ て ・ く ・ だ ・ さ ・ い」 ぐらい書いて質問スレに書けよ!

686 名前:132人目の素数さん:2007/02/11(日) 21:20:38
トーラスに楕円体は最大何個つめられるか。

687 名前:132人目の素数さん:2007/02/11(日) 21:54:46
>>686
無限にちっちゃい楕円体を詰めていけば、無限個詰められる?

688 名前:132人目の素数さん:2007/02/11(日) 22:06:46
トーラスに内接する最大体積の楕円体は最大何個つめられるか。

689 名前:132人目の素数さん:2007/02/11(日) 22:12:16
ウインナー状態だと?

690 名前:132人目の素数さん:2007/02/11(日) 22:34:20
トーラスの大きさにもよるな。

691 名前:132人目の素数さん:2007/02/11(日) 22:59:50
一年ほど前に確率スレッドで出題され、正式な回答が出ないままお蔵入りとなった問題です。

3つの連なった部屋A,B,Cがある。
部屋Aには200人の囚人がいて、それぞれ1〜200までの囚人番号が割り当てられている。
部屋Bにはそれぞれ1〜200までの番号が書かれた200枚のカードが、一列にふせて置かれている。
囚人たちは囚人番号1番から一人ずつ呼び出されて部屋Aから部屋Bにうつる。
ここで部屋Bに呼ばれた囚人は、200枚のカードのうち100枚を表にしてよい。
表にしたカードに自分の囚人番号が含まれていれば、その囚人は部屋Cにうつされる。
その後、カードはそのまま裏返されて、次の囚人が呼ばれ、同じことを繰り返す。
自分の囚人番号が含まれていなければ、すべての囚人は処刑される。
このようにして200人すべての囚人が部屋Cにうつることが出来たら、囚人達は解放されるとする。
囚人達が解放される確率を1/12以上にしたい。どうすればよいか?

*部屋Aにいる囚人同士は互いに相談できるが、部屋が違う囚人同士は、一切情報交換できない。
*最初のカードの並び方はランダムである。


当然、何の策略もなく挑めば生還率(1/2)^200ですが
例のスレッドではかなり確率を高めることに成功しました。
ただし出題者が行方不明となってしまい正式な回答は得られませんでした…
それでも結構面白い問題だと思うので是非挑戦してみてください。

692 名前:132人目の素数さん:2007/02/12(月) 00:00:11
天和が出る確率

693 名前:132人目の素数さん:2007/02/12(月) 01:11:38
問題

19XX年夏の高校野球大会に出場する高校数は予選から4131校出場する。
この年のルールではコールドはなく何があっても決着がつくまで試合が続けられる。
県予選,甲子園共にトーナメント方式。
各県代表校は1校。

この年に県予選,甲子園など公式戦の総試合数は全部合わせて[   ]試合である。


694 名前:132人目の素数さん:2007/02/12(月) 01:42:43
2^n=4131

695 名前:132人目の素数さん:2007/02/12(月) 01:51:19
>>694
これは答えですか?
全然違います

696 名前:132人目の素数さん:2007/02/12(月) 02:03:07
1試合で1チームが負ける(勝つ)。
最終的に1チームが残るのだから、計4130試合じゃないの?

697 名前:132人目の素数さん:2007/02/12(月) 02:16:27
>>693
既出ネタを貼るなよ。
帰れ! (゚Д゚)≡゚д゚)、カァー ペッ!! ペッ!! ペッ!! ペッ!! >>693

698 名前:132人目の素数さん:2007/02/12(月) 02:33:16
問8
http://www.whatisgoingon.net/glat.html

699 名前:132人目の素数さん:2007/02/12(月) 10:22:13
正三角形を切り刻んで正方形にするとき最低何ピースに切ればいいか。

700 名前:132人目の素数さん:2007/02/12(月) 10:23:11
切り刻んだピースは全部使うんだよ

701 名前:132人目の素数さん:2007/02/12(月) 10:27:13
トーラスの表面は何ピースで

702 名前:132人目の素数さん:2007/02/12(月) 11:32:46
4ピース

703 名前:132人目の素数さん:2007/02/12(月) 13:16:56
{F_n}をフィボナッチ数列とし、m、nを非負整数とする
mがnで割り切れるならばF_mはF_nで割り切れることを示せ
また、mがnで割り切れるとき、F_mをF_nで割った商を{F_n}を用いて表せ

704 名前:132人目の素数さん:2007/02/12(月) 14:57:21
>>703前半
F_0=0, F_1=1 とする。数列全体をmod Nで考えたとき「F_n≡0 ならば F_(kn)≡0」を示せばよい。
F_n≡0, F_(n+1)≡x と仮定すると、これは初項が0とxで生成されるフィボであり、
0と1から始まるフィボ全体をx倍したのと同じなので、F_(n+i)≡xF_i が成り立つ。
よって、たとえば F_(3n)≡F(n+2n)≡xF(2n)≡xF(n+n)≡(x^2)F(n)≡0。
一般のF_(kn)も、F_(kn)≡xF((k-1)n)≡‥‥≡(x^(k-1))F(n)≡0。

具体例:mod 5で考えると
0, 1, 1, 2, 3, 0, 3, 3,‥‥(5番目が0、次が3だから、その後は)
↓(3倍)  ~~~~
0, 3, 3, 1, 4, 0, 4, 4,‥‥(全体を3倍したのと同じになる。だから5の倍数番目は全部0)

705 名前:132人目の素数さん:2007/02/12(月) 15:30:01
〔問.479〕
 x,y,z は自然数で、 1/x + 1/y = 1/z, xとzは互いに素とする。
 このとき x+y, x-z, y-z はすべて平方数であることを示せ。

http://www.cms.math.ca/Competitions/MOCP/2007/prob_jan.pdf
例 (x,y,z) = (x, x(x-1), x-1)

706 名前:132人目の素数さん:2007/02/12(月) 16:07:28
>>705
1/x + 1/y = 1/z より y=(x+y)z/x
xとzは互いに素でyは自然数だからx|(x+y)
したがってある自然数mが存在してx+y=mx となる
よってy=(m-1)x=mzとなるのでz=(m-1)x/m
m-1とmは互いに素でzは自然数だからm|x
したがってある自然数nが存在してx=mn
これよりz=n(m-1)を得るが、xとzは互いに素なのでn=1
よってx=m、y=m(m-1)、z=m-1を得る
このときx+y=m^2、x-z=1、y-z=(m-1)^2なので
たしかにx+y、x-z、y-z はすべて平方数となる

707 名前:703:2007/02/12(月) 16:43:44
>>704
エレガントな証明ですね
私が考えていたのは、F_(m+n) = F_(m+1) * F_n + F_m  * F_(n+1)
を利用して、帰納法で示すものでした
後半は表示が一意じゃないと思います

708 名前:132人目の素数さん:2007/02/12(月) 16:46:31
馬鹿馬鹿馬鹿馬鹿馬鹿馬鹿馬鹿馬鹿馬鹿馬鹿馬鹿馬鹿お前らがどんなに勉強しても天才には勝てな
お前ら凡人が千人集まっても天才には勝てない
もう勉強やめろ

709 名前:にょにょ ◆yxpks8XH5Y :2007/02/12(月) 16:53:41
天才に勝つために勉強してるんじゃないぜよ。

710 名前:132人目の素数さん:2007/02/12(月) 17:19:31
天才は馬鹿から生まれる

711 名前:132人目の素数さん:2007/02/12(月) 18:14:58
のび太は馬鹿だがドラえもんをつくった

712 名前:132人目の素数さん:2007/02/12(月) 20:22:42
x,y,z は自然数で、 1/x + 1/y = 1/z, xとzは互いに素とする。
このとき x+y, x-z, y-z はすべて平方数であることを示せ。

x+y=xy/z=mx->y=(m-1)x=mz->x=mz/(m-1)->z=(m-1)->x=m->x+y=m^2
x-z=m-(m-1)=1^2
y-z=mz-z=m^2-m-m+1=(m-1)^2


713 名前:132人目の素数さん:2007/02/12(月) 20:28:34
x+y=m^2
x-z=n^2
y-z=p^2
y+z=m^2-n^2
y=(p^2+m2-n^2)/2
z=(p^2-n^2-p^2)/2
x=m^2-y^2=(m^2-p^2+n^2)/2
...


714 名前:132人目の素数さん:2007/02/12(月) 21:15:44
円周上に異なる8個の点を取り、全ての点を線で結ぶ。

(1)、線分は全部で何本できるか。
(2)、この線分の中から3本を取って選ぶとき、選ばれた3本の
線分の端点が全て異なる確率を求めよ。

715 名前:132人目の素数さん:2007/02/12(月) 22:28:27
>>714
面白くないんだが

716 名前:132人目の素数さん:2007/02/13(火) 00:34:39
中学生の宿題かよ・・・

717 名前:132人目の素数さん:2007/02/13(火) 01:09:46
>>691
意味分からん
カードを並べ替えられるとかならともかく、
そのまま裏返したら確率 2^(-200) にしかならなくない?

718 名前:132人目の素数さん:2007/02/13(火) 01:16:00
>>714
質問は質問スレに行けよ、餓鬼が! (゚Д゚)≡゚д゚)、カァー ペッ!!

719 名前:132人目の素数さん:2007/02/13(火) 05:08:19
>>706,712
どうもです

〔問.476〕
 p >0, |x_0| ≦ 2p とし,
 x_n = 3x_(n-1) -(1/p^2){x_(n-1)}^3   (n≧1)
と定義する。
 x_n を n と x_0 の函数として表わせ。


〔問.478〕
  √{2+√[2+√(2+x)]} + (√3)√{2-√[2+√(2+x)]} = 2x,
  x ≧ 0.
を解け。

http://www.cms.math.ca/Competitions/MOCP/2006/prob_dec.pdf

720 名前:132人目の素数さん:2007/02/13(火) 05:29:47
>>708
天才のひらめきは千人万人の凡人の地を這うような研究の結果を元に起こるものなのだよ。

721 名前:132人目の素数さん:2007/02/13(火) 05:37:58
>>717
200人ではなくふたりの場合を考えみたんだが。
(カードは2枚中1枚をめくる)

打ち合わせ無しの場合 → ふたりが助かる確率は1/4

事前に以下の打ち合わせした場合 → 助かる確率は1/2
「1番は右の、二番は左のカードをめくろう」
(同じカードをめくらないようにしよう)

てな感じで、助かる確率を上げられそうだ。
200人の場合も何か方法があるかもしれん。

すくなくとも200人が全く同じカードをめくる場合助かる確率は0だもの。
直感的には、どのカードもちょうど半分のひとにめくられるように
戦略を立てるのがよいような気がする。

722 名前:Queen ◆xeS.CIM.Jk :2007/02/16(金) 11:02:08
定理:
n∈Nとする。
任意のnに対して平面上に以下を満たすn個の点が存在する。
・どの三点も同一直線上にない。
・どの二点間の距離も整数。

この定理を導く公理は何か。

723 名前:132人目の素数さん:2007/02/16(金) 12:14:13
>>719
[問476]
θ∈[0,2π)はsinθ=x0/(2p)を満たすとする。|x0|≦2pより、このようなθは存在する。
このとき、xn=(2p)sin(θ*3^n) と表せることが(数学的帰納法により)分かる。

724 名前:132人目の素数さん:2007/02/16(金) 12:41:19
[問478]
与式を変形して(√3)√[2−√{2+√(2+x)}]=2x−√[2+√{2+√(2+x)}]となる。
右辺は実数だから、左辺も実数となる。もし2−√{2+√(2+x)}<0だとすると、
左辺は(0でない)純虚数となってしまい、矛盾する。よって、2−√{2+√(2+x)}≧0となる。
これを解いてx≦2を得る。よって、x=2cosy,y∈[0,π/2]と表せる。これを代入すると、
与式 ⇔ 2cos(y/8)+(√3)2sin(y/8)=2cosy ⇔ sin(y/8+π/6)=cosy
⇔ sin(y/8+π/6)=sin(π/2−y)
となる。π/2−y,y/8+π/6∈[0,π/2]であるから、y/8+π/6=π/2−yとなる。
よってy=8π/27 となり、x=2cos(8π/27)が求める解である。

725 名前:132人目の素数さん:2007/02/17(土) 11:20:13
>723-724
 どうもです


[問472]
 (4-x)^(4-x) + (5-x)^(5-x) + 10 = 4^x + 5^x
を満たす整数x

[問474]
 {2^log_5(x) +3}^log_5(2) = x -3
を満たすx>0.

726 名前:132人目の素数さん:2007/02/18(日) 02:49:16
>>717
カードの並び替えOKだと1/12どころか、それよりはるかに高い確率が実現出来ますが
実はカードを並び替えなくても、十分高い確率がだせます。
>>721
まあそんな感じです。

「これが最高確率だ!」というような答えはありませんが、おそらくそれに近いであろう物は用意してあります。

727 名前:132人目の素数さん:2007/02/19(月) 22:56:51
(1)一辺の長さが1の正四角形の周上に全ての頂点を持つ正三角形の辺の長さの範囲を求めよ。
(2)一辺の長さが1の正五角形の周上に全ての頂点を持つ正四角形の辺の長さの範囲を求めよ。

次のは自分では解いてないです

(3)一辺の長さが1の正n+1角形の周上に全ての頂点を持つ正n角形の辺の長さの範囲を求めよ。

728 名前:132人目の素数さん:2007/02/20(火) 00:33:34
an+1=f(an)でfが多項式のときのクックの仕方はどうするのですか?
馬鹿教授が漸化式はむずいの一言で済ませて逝ってしまったので・・・とほほ

729 名前:132人目の素数さん:2007/02/20(火) 00:35:04
たぶんパスカルの三角形みたいな小技で済ませばいいとか?

730 名前:132人目の素数さん:2007/02/20(火) 01:36:35
カードを並べ替えることにどれだけ意味があるの?
先に打ち合わせをしておいたら同じだと思うのだが…
打ち合わせについてなにか誤解してるかな?俺…?

731 名前:132人目の素数さん:2007/02/20(火) 05:01:18
http://science5.2ch.net/test/read.cgi/math/1171912698/

732 名前:132人目の素数さん:2007/02/20(火) 05:50:01
番号の位置に並び替えることにすれば1が終わった時点で
1234,1243,1324,1432のどれかになる。


733 名前:132人目の素数さん:2007/02/20(火) 05:55:00
>>732
4人で2枚表にする場合。


734 名前:132人目の素数さん:2007/02/20(火) 06:04:02
>>730
例えば、
一人目が左端から100枚めくって番号順に並べ替える事にする。
このとき一人目がCの部屋に行ける確率は1/2。
二人目は右端から100枚めくって、全てのカードが
左端から順番に並ぶように並べ替える。
このとき二人目がCの部屋に行ける確率は1/2。
三人目以降は自分の番号のカードがどこに有るか
分かるので確実にCの部屋に行ける。

従って全員が生き残る確率は1/4。

735 名前:132人目の素数さん:2007/02/20(火) 06:13:46
>>726
ところで部屋Bで自分のカードをめくらなかった
囚人は部屋Aに戻るの?
でもさ、たった一人でも囚人が戻って来たら
一人も釈放されないんだから
まだ部屋Bにも行ってない囚人は
やる気をなくしてしまうだろうね。

736 名前:132人目の素数さん:2007/02/20(火) 11:36:03
>>734
二人目は右端から 99 枚めくり、その中にあれば、
右端から 100 枚目をめくって左から昇順に整列させる。
その中にないときは、左から 2 番目のカードをめくり、
最初の 99 枚は右端から昇順に整列させる。
二人目がCの部屋に行ける確率は 198/199.
三人目以降は、右端をめくれば全体が整列されているかどうかがわかる。
整列されていないときは、二人目が選んだ 99 枚から二分探索で探し、
その中になければ右から 100 枚目をめくる。
そこにもなければ、左から何枚目にあるかがわかっているはず。

ということで、確率は 99/199 にできると思う。

737 名前:132人目の素数さん:2007/02/20(火) 11:48:16
>736>734 は想定しているルールが違うようだ。

738 名前:132人目の素数さん:2007/02/20(火) 11:51:25
>>736
上手い!

でも元の問題ではカードは並べ替える事が出来ないから
関係ないかも。

>>737
そう。でも、>>730の疑問に答えているだけ。

739 名前:132人目の素数さん:2007/02/20(火) 14:27:29
なるほど、自分がめくって「いない」カードも並べ替えてよいのか。

740 名前:132人目の素数さん:2007/02/21(水) 03:21:35
test

741 名前:726:2007/02/21(水) 03:34:02
アク禁でレスが遅れてすみません。
>>735
自分のカードをめくれなかった囚人が一人でも発生したときは
その場ですべての部屋にいる囚人が処刑されます。
まあ題意とは関係のない設定ですが(苦笑

99/199は、かなりいい線いってますね。
並び替えOKのルールの中では、最高レベルに高い数字だと思います。
私は問題製作者でないので1/12という数字がどこから出てきたのか知りませんが
実際の答えはもっと高いので、あまり深い意味はないようです。

742 名前:132人目の素数さん:2007/02/21(水) 04:50:52
>>741
問題が >>691 のとおりだとすると、
囚人達にできることは、
「一番最初に部屋 A で、(>>721 のように)
各囚人がどのカードをめくるかをあらかじめ打ち合わせて置く」
ことしかできないと思うんだが、
それだと4人目の囚人のところで生存確率 1/12 を切ってしまう。

問題の解釈間違ってる?

743 名前:132人目の素数さん:2007/02/21(水) 08:21:03
そうなんだよね。n+1 人目が成功する確率は、どんなにがんばっても
100/(200-n) より高くならないように思えるのだけど。

744 名前:132人目の素数さん:2007/02/21(水) 11:08:22
だなあ。並べ替えが出来ないとすると、
2番目の囚人にわかることは、1番目の囚人がめくったカードの中に1番のカードがあったということだけな気がする。

1番目の囚人がパスする確率は1/2。これはどうしようもないと思う。とすると、残りの199人が1/6以上の確率でパスしなければならない。
2番目の囚人が最も高い確率でパスするのは、1番目の囚人がめくらなかった100枚をめくることだが、それが100/199。←違う?
なので、2人目までがパスする最も高い確率は50/199。
すると、残りの198人は199/300以上の確率でパスしなければならない。
3番目だけですら、そんなに高い確率でパスする方法はなさそうに思える。

さっぱりわからん。

745 名前:132人目の素数さん:2007/02/21(水) 11:22:15
成功した場合は自分の番号の付いたカードを表のままにし、
それ以外のカードを裏に戻すとルールを変更したとする。
このとき、n+1 人目にとって前の n 人がどのカードを
めくったかの情報は無意味だから(だよね?)、n+1 人目が
成功する確率は 100/(200-n).
本来のルールではこれより確率は高くならない。

746 名前:132人目の素数さん:2007/02/21(水) 20:55:13
ネタ投下します。

(√26+5)^n=a_nとする。a_nは小数点の後n個9か0が続く事を示せ。

α、βは1/α+1/β=1であるような無理数である。A={[nα]|n=1,2,3,・・・}
B={[nβ]|n=1,2,3,...}とする([x]はxの整数部分を表す。)。この時A∩B={0}であり、A∪B=Nである事を証明せよ。
Nは自然数の集合である。

既出ならすいません。

747 名前:=726=691:2007/02/21(水) 22:29:34
>>742
「一番最初に部屋 A で、
各囚人がどのカードをめくるかをあらかじめ打ち合わせておく」
その趣旨で間違いありません。

では以下ヒントを。
「並べ替えOKルール」の場合、実は全員が成功する確率を「1/2」にすることが出来ます。

まず囚人1号は無作為に100枚のカードをめくります。
そしてめくったカードのうち、k番と書かれたカードが右からk番目の位置に来るように
それぞれを並び替えます。
次に、それ以降の囚人は、
最初に、右から数えて自分の囚人番号の箇所にあるカードをめくります。
もし囚人1号がめくったカードの中に、自分の囚人番号が含まれていれば
最初の一枚で、自分の番号を引き当てられるので、その囚人はC部屋にうつれます。
では、そうでない囚人は、どのような「どのような規則で」それ以降のカードをめくればよいか?
実はこれがそのまま「並び替えNG」の場合における正解になるのですが、
ここではまだふせておきます。

748 名前:132人目の素数さん:2007/02/21(水) 23:44:13
>743 は、まず 100 枚選んだ後に同時に表にするという手順を仮定している。
一枚選ぶ毎に表にし、何が出たかによって次に選ぶカードを変化させるという
戦略にしないと >745 の推論が有効になるということか。

749 名前:132人目の素数さん:2007/02/22(木) 00:39:21
>>734>>736も何が出たかでどのカードを開くか決めている


750 名前:132人目の素数さん:2007/02/22(木) 00:57:08
>>746前半
(5+√26)^n+(5-√26)^n ∈Z であることが帰納法で証明されるので、
10^-(n+1) < |5-√26|^n < 10^-n を言えばよい。
⇔ 10^n < 1/|5-√26|^n < 10^(n+1)
⇔ 10^n < |5+√26|^n < 10^(n+1)
⇔ n < nlog|5+√26| < n+1
⇔ 1 < log|5+√26| < 1+1/n を示せば終了‥‥

‥‥と思いきや、これだと n>234 で主張が成立しないことになる。
というわけで、詰まった。

751 名前:132人目の素数さん:2007/02/22(木) 01:03:54
>>750
√26-5<0.1がすぐ言えるでしょ

752 名前:132人目の素数さん:2007/02/22(木) 01:06:48
>>747 のヒントで方針はわかった気がする。

まず、1番目の囚人が (右から) 1番目のカードをめくる。
出た数を n_1 として、それが1でないなら次に n_1番目のカードをめくる。
以後、 k番目にはk-1番目に出た値 n_(k-1) 番目のカードをめくり、
出た値 n_k を元に次をめくる。これを1が出るまで行う。
(必ず1に行き着く説明は省略)
1をめくった後は、ルールに従うと 1番目のカードに戻るので、
このルールでカードを選んでいくとループする。

さて、そのような同じループに属するカードごとに、
200枚のカードをグループ分けできる。(各グループに重複はない)
最初にm番目を開いたとき、mを見つけるまでにカードをめくる回数は、
m番目のカードが属するグループのサイズに等しい。

よって、200枚のカードを分割するいくつかのグループの
いずれのサイズも100以下であれば、囚人たちはこの方針でクリアできる。

ただ、そうなる確率の算出方法がわからない...。

753 名前:750:2007/02/22(木) 01:22:57
もしかして>>746の題意は「少なくともn個、9か0が続く事を示せ。」なのかな?
だとしたら>>751で解決か。「ちょうどn個」と解釈していた。

考えてみれば、+1乗するごとに、桁数がきっかり1桁ずつ増えていくなんて、
10以外では起こりえないのは当然かも。

754 名前:132人目の素数さん:2007/02/22(木) 02:42:16
>>752
だいたい 1-log 2 ぐらいだね。
正確には 1-(1/101+1/102+...+1/200)=0.3093...

755 名前:=726=691:2007/02/22(木) 15:25:33
>>752
正解です!

実は私も最初算出方法が分からなかったのですが、
>>754のいう1-log2≒「0.31」がそれのようです。
実際にプログラムを組んで実験させても、近い数字が出ます。

なかなか驚くべき答えだと思うのですがいかがでしょう?

756 名前:132人目の素数さん:2007/02/22(木) 16:25:41
長さ 101 以上のサイクルは 1 つしか存在しない。
長さを k の順列の個数は 200!/(200-k)!
長さを k のサイクルの個数はその k 分の 1.
残りの 200-k 個の順列は (200-k)!

したがって、失敗する組合せは
Σ[k=101,200] {(200!/((200-k)!k) (200-k)!}
=200!Σ[k=101,200] 1/k

757 名前:132人目の素数さん:2007/02/22(木) 16:43:18
nが十分でかいときはそれが最適解になるのかな?

758 名前:132人目の素数さん:2007/02/22(木) 17:25:32
>>752
なるほどなあ。
しかし、処刑する側がそのことを読んでいて、200枚でループするように置いていたらアウトだなあ。

759 名前:132人目の素数さん:2007/02/22(木) 17:40:00
位置をランダムにすればいい。


760 名前:132人目の素数さん:2007/02/22(木) 17:54:39
>>746後半

(A∪B=Nであること)
任意のk∈Nに対し、k∈Aまたはk∈Bが成り立つことを示せばよい。k∈Aのときは それでよいから、
k∈/Aのときを考える。iα<kを満たすi∈Nのうち最大のものをnとすれば、nα<k,k+1≦(n+1)α
が成り立つ(k+1>(n+1)αならばk∈Aとなってしまう)。この不等式をαで割ってn<k/α,
(k+1)/α≦n+1となる。1/α+1/β=1より、n<k(1−1/β),(k+1)(1−1/β)≦n+1 となる。
これを変形してk<(k−n)β≦(k+1) となり、βが無理数であることからk<(k−n)β<(k+1)
となり、よって[(k−n)β]=kとなる。すなわちk∈Bとなる(α>1なのでk−n>0であることに注意)。

(A∩B=φであること)
k=[nα]=[mβ]とすると、k<nα<k+1,k<mβ<k+1 が成り立つ(α,βは無理数だから等号は入らない)。
よってk/α<n<(k+1)/α,k/β<m<(k+1)/β が成り立つ。片々足してk<n+m<k+1となるが、k,n+m,k+1は
全て自然数だから矛盾。

761 名前:132人目の素数さん:2007/02/22(木) 18:04:19
>>746後半は
RayleighとVinogradovが証明したんだっけ。
なんかRayleighでぐぐってもVinogradovでぐぐっても出てこないw

762 名前:132人目の素数さん:2007/02/22(木) 18:29:30
>>755
二人目からが最初にめくるカードはどこにすればよいのでしょうか?

763 名前:132人目の素数さん:2007/02/22(木) 18:38:42
自分の番号番目のカード

764 名前:746:2007/02/22(木) 20:23:22
家にあった数オリの本にあった問題なので、レベル低かったかも。
後問題に書き間違いしていた事をお詫びします。(A∩B={0}のあたり。)

>>753
「ちょうど」です。>>751さんの不等式がキーです。

>>760
お疲れ様です。一応、本に書いてある解法を書いておきます。

Nを自然数とする。 Nより小さい数の中で、A∪Bに含まれるような数が何個あるかを調べる。
[nα]<Nならnα<Nである。なので、Nより小さい数はAには[N/α]だけ含まれる。
同じように、Bには[N/β]だけ含まれる。なので、Nより小さい数はA∪Bには[N/α]+[N/β]=kだけある。
αとβは無理数なので、k<N/α+N/β=N。[x]>x-1なので、k>N/α-1+N/β-1=N-2。
kは自然数なので、k=N-1である。これはA∪BにはNより小さい数はN-1個だけあるという意味である。
これは全ての自然数において成り立つため、同じ数は二回出てこない。
なので、A∪B=Nであり、同時にA∩B=φである事も証明された。

>>761
本には問題13としか書いてないので、全然分かりません。定理なんですか?これ。

765 名前:132人目の素数さん:2007/02/22(木) 23:07:28
なんかおかしいよな

766 名前:761:2007/02/23(金) 01:07:08
>>764
そう。たしか一松信「石とりゲームの数理」とか
中村滋「フィボナッチ数の小宇宙」に載ってるはず。

767 名前:132人目の素数さん:2007/02/24(土) 00:52:22
>>764
その証明は間違ってる。

>なので、Nより小さい数はA∪Bには[N/α]+[N/β]=kだけある。
A∩B=φであることを示さなければ、これは言えない。AとBに重複する
元があったら、Nより小さい自然数はA∪Bに[N/α]+[N/β]=kより少ない
個数しか無い。

>同時にA∩B=φである事も証明された。
先にA∩B=φを証明しておかなければならないから、これも間違い。

768 名前:132人目の素数さん:2007/02/24(土) 06:47:53
>>746=764
数オリっていつの問題だよ。
外国のサイトでもいいから貼ってくれないかい?

769 名前:746:2007/02/26(月) 00:00:47
>>766
面白そうな本ですね。Amazonで調べてみます。

>>767
Nを1から無限まで動かすと、A∩B=φが証明されると思います。
N=2として1が1個ある事が証明され、N=3としてN=2の時1が1個あるのが証明されたので,もうひとつは2、・・・って感じで。


>>768
数オリに出てた問題ではなく、数オリに出るためへの練習問題みたいな感じで書いてありました。
1問目は数オリっぽい問題ですが、何時の問題かとかは書いてありませんでした。


1問目の解答を書いたほうがいいかな?

770 名前:132人目の素数さん:2007/02/26(月) 00:10:25
1問目は書かんでも分かるだろ

771 名前:132人目の素数さん:2007/02/27(火) 20:12:49
>>746
の二問目は数蝉のエレ解で見たことがあるな。

772 名前:132人目の素数さん:2007/02/28(水) 00:39:16
>>746 の二問目

題意より α>1, β>1.
 M = {1,2,…,m} ⊂ N
とおく。
 A∩M = { [nα] | n∈M }, #(A∩M) = [m/α],
 B∩M = { [nβ] | n∈M }, #(B∩M) = [m/β],
 m/α + m/β = m
と無理数性より
 [m/α] + [m/β] = m-1,
mについての帰納法で…

773 名前:132人目の素数さん:2007/03/04(日) 00:55:19
わはははは。>>747のヒントでオレは分かったぞ。
以下、並び替える前提でスマートな回答。

1. 1番目の囚人は適当に100枚ひっくり返す。セーフの確率は1/2。
2. セーフだったとして、表返したカードを左(右)端から数えて
 カード自身の番号となるように「横向きに」並び替える。
 伏せているカードは間を埋めるように「縦向きのまま」おいておく。
3. 2番目の囚人は自身の番号(2番目)に該当するところを見る。
 2番目のカードが横向きならそれを表替えせばクリア。
 2番目のカードが縦向きならどこにあるかは不明なので、
 縦向きのまま残っている100枚をひっくり返せばどれかが当たり。
4. あとは並び替えるまでもなく2番目と同様に繰り返せばよい。

これで絶対確率は1/2じゃうひょぉおおおおおお!!!!!

・・・・・・カードが正方形とか丸だったらどうしようorz。

774 名前:132人目の素数さん:2007/03/04(日) 11:16:25
>>773
こんなアホウは久しぶりに見た

775 名前:132人目の素数さん:2007/03/04(日) 12:23:56
俺はむしろ柔軟な発想に感心したけどな

776 名前:132人目の素数さん:2007/03/04(日) 13:07:35
いや、既出なんだよ。
亀レスなのに、既出を見てないんだもの。

777 名前:772:2007/03/04(日) 14:47:54
>772 の訂正、スマソ

 A∩M = { [nα] ≦ m | n∈N },
 B∩M = { [nβ] ≦ m | n∈N },


778 名前:132人目の素数さん:2007/03/04(日) 15:34:10
>>772
帰納法使うくらいなら、A∩B=φを直接示した方が早くて分かりやすいな。

779 名前:772:2007/03/04(日) 18:40:12
>778

背理法による。k ∈ (A∩B) だったと仮定する。
 k < n1・α < k+1  … (1),
 k < n2・β < k+1  … (2),
(1)/α + (2)/β より
 k < n1 + n2 < k+1 … (3).
これは n1, n2 が自然数であることと矛盾する。

780 名前:132人目の素数さん:2007/03/05(月) 03:04:32
>>779
>>760

781 名前:132人目の素数さん:2007/03/08(木) 16:41:02
http://game11.2ch.net/test/read.cgi/handygame/1173022847/99

答えは↑スレ全部表示するとわかる

782 名前:132人目の素数さん:2007/03/11(日) 12:05:06
>>781
見た瞬間、ありきたりの問題かと思ったが、
意外に面白かった。

783 名前:132人目の素数さん:2007/03/11(日) 18:21:36
age

784 名前:132人目の素数さん:2007/03/11(日) 22:03:52
「モジュライ空間」を小学生にもわかるように説明せよ

785 名前:132人目の素数さん:2007/03/12(月) 00:17:40
喪男の7月空間

786 名前:132人目の素数さん:2007/03/12(月) 04:04:16
>>785
6月だろうが馬ー鹿

と書こうとしてjuly=7月だったと気づいた俺はもう逝きますさよなら

787 名前:132人目の素数さん:2007/03/13(火) 14:53:06
>>746の二問目はシェーンベルグの「数学点描」にも紹介されてますね。
起源は弦の振動数に関する問題だそうで。
http://www.amazon.co.jp/Theory-Sound-Classics-Science-Mathematics/dp/0486602923/ref=sr_1_1/249-7482077-6243506?ie=UTF8&s=english-books&qid=1173764534&sr=8-1

そこでは、それに関連する問題として第20回のIMOの問題が紹介されています(↓の3番)
http://imo.math.ca/Exams/1978imo.html

788 名前:132人目の素数さん:2007/03/22(木) 19:19:08
>>781
[予想]最低99人助けることが可能

便宜上 赤≡0 黄≡1 青≡2 (mod[3])とおき
前からi番目の小人がかぶった帽子の色をa[i]
前からi番目の小人が宣言する色をb[i]とする

ここで
 b[100]≡a[1]+a[2]+a[3]+…+a[99]
とし、以下
 b[99]≡b[100]-(a[1]+a[2]+・・・+a[98])
 b[98]≡b[100]-b[99]-(a[1]+a[2]+…+a[97])
 …
 b[k]≡b[100]-b[99]-…-b[k+1]-(a[1]+a[2]+・・・+a[k-1])
 …
 b[1]≡b[100]-b[99]-…-b[2]
とすれば、b[s]≡a[s](s=1,2,・・・,99)となるので
最後尾の小人以外は全員助かる。

どうよ?

789 名前:132人目の素数さん :2007/03/22(木) 22:55:32
10個の連続する2桁の正の整数がある。これらの10個の数字を小さい順に
並べた後、それぞれの2桁の2つの数字の和(十の位と一の位の和)を
求めると、n番目の和は必ずnの倍数になっていた。このような10個の
連続する2桁の整数のうち、最大の整数を求めよ。

790 名前:132人目の素数さん:2007/03/22(木) 23:18:55
勘で71から。
理由は10番目が10の倍数だから、他の整数は明らかに一桁目がnの数字になる。
あとは7の倍数で考えるのが一番早い、とおもう。

791 名前:132人目の素数さん:2007/03/22(木) 23:59:48
>>790
n=3のときちがうくない?

792 名前:132人目の素数さん:2007/03/23(金) 00:04:11
11〜20だとおもう

793 名前:132人目の素数さん:2007/03/23(金) 00:05:59
ぜんぜんちがった
てか11 21 31 41・・・って調べていったけどぜんぶちがった

794 名前:132人目の素数さん:2007/03/23(金) 00:12:09
連続する二桁で10の倍数なんてなくね・・・

795 名前:132人目の素数さん:2007/03/23(金) 00:18:22
10〜19と19〜28

796 名前:132人目の素数さん:2007/03/23(金) 00:38:32
ないってこと?

797 名前:132人目の素数さん:2007/03/23(金) 00:42:08
>>796
>>795

798 名前:132人目の素数さん:2007/03/23(金) 01:04:18
>>797
10の倍数つくれてないじゃん

799 名前:132人目の素数さん:2007/03/23(金) 01:43:37
10の倍数作れなんて問に書いてない。                   >>190          10番目の数は必ず10の倍数ではない。11〜21だって連続する2桁の正の整数

800 名前:132人目の素数さん:2007/03/23(金) 01:45:24
訂正11〜20

801 名前:132人目の素数さん:2007/03/23(金) 01:46:23
更に訂正12〜21

802 名前:132人目の素数さん:2007/03/23(金) 01:48:19
上のメチャクチャ

803 名前:132人目の素数さん:2007/03/23(金) 12:48:44
ここにいるのは問が理解できていない。

804 名前:132人目の素数さん:2007/03/23(金) 19:28:07
すまん、おれが>>790でよく読んでないこと言ったから。
37〜46っぽいね。
方法は91から下に考えていった。

805 名前:132人目の素数さん:2007/03/23(金) 19:33:37
>>804

3+8=11・・・2の倍数ではありません

806 名前:132人目の素数さん:2007/03/23(金) 21:24:13
ごめんなさい、控えるわ。
>>795か・・・。

807 名前:132人目の素数さん:2007/03/23(金) 21:59:09
19〜28

808 名前:132人目の素数さん:2007/03/23(金) 22:36:18
28

809 名前:132人目の素数さん:2007/03/29(木) 02:45:39
1

810 名前:132人目の素数さん:2007/03/31(土) 07:37:51
実数αについてα=βであることもα≠βであることも証明出来ない実数βが存在することを示せ。

811 名前:132人目の素数さん:2007/03/31(土) 08:07:48
「実数αについてα=βであることもα≠βであることも証明出来ない実数βが存在する」
これを論理式で書き下そうとしてみればわかるが、
内容が少しあいまいだ。

812 名前:132人目の素数さん:2007/03/31(土) 10:50:35
>>810
(R,≦)は全順序なので、任意の2つの実数x,yについて
x<y x=y x>y
のうちどれか1つが必ず成り立つ。よって、そのような実数βは存在しない。

813 名前:132人目の素数さん:2007/03/31(土) 11:12:29

究極のアホ

814 名前:132人目の素数さん:2007/03/31(土) 11:17:47
究極って事は無いだろ
ありがちって感じ

815 名前:812:2007/03/31(土) 12:55:54
意味が分かった/(^o^)\
実際に「そうである」ことと、それが「証明可能である」ことは違うんだったな。

816 名前:812:2007/03/31(土) 13:20:19
>>812は「α=β」の証明でもなければ「α≠β」の証明でも
なく、「α=βまたはα≠β」の証明ってわけか/(^o^)\

817 名前:132人目の素数さん:2007/03/31(土) 15:52:17
有限の文字列から成る証明は可算個しか存在しないから、とかそんな感じか?

818 名前:132人目の素数さん:2007/03/31(土) 22:41:49
論理体系は指定されてないんだから任意だとしてよいんだよな。
だとしたら定数記号として実数が全て含まれていて
任意のs,tに対してs=tかs≠tのどちらかが公理に含まれているような場合は
>>810は成り立たないな

819 名前:132人目の素数さん:2007/04/01(日) 02:18:51
定数記号って有限個じゃなくてもいいのか?

820 名前:132人目の素数さん:2007/04/01(日) 02:33:18
いちおう証明論的には。
もちろん我々がそういう理論をきちんと理解できるか、とか
現実的に計算機で証明をチェックできるかとかそういう問題はあるけど
有限の言語Lとか書いてなきゃ無限でも良いと考えるのが普通かと。

>>818の「論理体系」はもっと精確に言えば言語Lだね。

821 名前:132人目の素数さん:2007/04/01(日) 23:20:38
>>813

究極のアホ de キモオタ


822 名前:132人目の素数さん:2007/04/07(土) 10:58:59
>811
これまた1=0.9999・・・のスレで流用できそうなネタだな。

823 名前:132人目の素数さん:2007/04/07(土) 12:45:26
証明論とか、基礎論とか全く知らないんだが、
証明可能性ってどうやって、議論すればいいんだ?

824 名前:132人目の素数さん:2007/04/11(水) 04:29:42
αを無理数、nを正の整数とする
このとき実変数実数値関数f(x)で
f(0)はαの整数部分
f(n)はαの小数第n位の数
f(x)は実数全体で可微分
となるものは存在するか?

825 名前:132人目の素数さん:2007/04/11(水) 08:34:50
>>824
する

826 名前:132人目の素数さん:2007/04/11(水) 09:10:59
>>824
関数 sinc(x):= sin(πx)/(πx) を用いれば、関数
g(x):= 納n:0,∞] f(n) sinc(x - n)
は条件を満たすはず。厳密な証明は分からないけど。

827 名前:132人目の素数さん:2007/04/11(水) 16:23:19
つか、そのαとか意味あんのか?
f:N→Nでいいじゃん。

828 名前:132人目の素数さん:2007/04/11(水) 18:32:54
>>827
>f(x)は実数全体で可微分

829 名前:132人目の素数さん:2007/04/11(水) 21:58:13
>>828
f:N→NはR上の可微分関数に延長できないの?

830 名前:132人目の素数さん:2007/04/12(木) 12:56:56
多様体論で使用される、1の分割を使ってよいなら、存在は簡単に言える。

831 名前:132人目の素数さん:2007/04/12(木) 20:02:45
(1)関数f:R→Rは次の条件を満たすとする。
・f(0)=0
・fは微分可能で、x∈(-1,1)のとき|f '(x)|<1/2
このとき、|a0|<1,a[n+1]=f(an) (n=1,2,…)で定義される数列anの極限値を求めよ。

(2)関数f:R→Rは次の条件を満たすとする。
・f(0)=0
・fは微分可能で、x∈(-1,1)のとき|f '(x)|<1
このとき、|a0|<1,a[n+1]=f(an) (n=1,2,…)で定義される数列anの極限値を求めよ。

832 名前:132人目の素数さん:2007/04/13(金) 01:39:19
f(x)=x^x^x^x^x^・・・について
(1)xが実数のとき、f(x)が存在するxの条件を求めよ
(2)xが複素数のとき、f(x)が存在するxの条件を求めよ
ただし主値のみを考えるとする

833 名前:132人目の素数さん:2007/04/13(金) 15:52:34
f[n](x)=x^f[n-1](x)
f[1](x)=x
f(x)=lim[n→∞]f[n](x)
つーことですか?

834 名前:132人目の素数さん:2007/04/13(金) 17:56:47
そう

835 名前:132人目の素数さん:2007/04/14(土) 00:27:48
そうですか

836 名前:132人目の素数さん:2007/04/14(土) 16:52:48
pを素数とする。n=(p^p)+2が素数となるとき
(1)最小のnを求めよ
(2)このように表されるnは無数にあることを示せ

(1)はすぐ分かったのですが、(2)の証明がどうしても分かりませんでした
どなたか教えて下さい

837 名前:132人目の素数さん:2007/04/14(土) 17:23:39
>>836
マルチ

838 名前:132人目の素数さん:2007/04/14(土) 18:19:45
マルちゃん

839 名前:132人目の素数さん:2007/04/14(土) 19:44:24
(2)このように表されるnは無数にあることを示せ ー>素数は無数にあるから
問題の日本語はおかしい。

840 名前:132人目の素数さん:2007/04/14(土) 19:49:38
>>839
あげんなボケ

841 名前:132人目の素数さん:2007/04/15(日) 00:29:12
すげ〜・・・。ここの住人超頭良い。
俺は馬鹿だから、理数系に強い人は本当に好きだね。
憧れてしまうなぁ。

842 名前:132人目の素数さん:2007/04/16(月) 18:26:00
>>841
黙れ

843 名前:132人目の素数さん:2007/04/16(月) 21:46:19
(x-a)(x-b)(x-c)(x-d)・・・・(x-z)=?

844 名前:132人目の素数さん:2007/04/16(月) 21:48:46
>>841
コテつけろ!
そうすれば、貴様の糞レスを読まずに済むからな!

845 名前:132人目の素数さん:2007/04/16(月) 22:14:53
>>843
それ面白いか?

846 名前:132人目の素数さん:2007/04/17(火) 02:00:58
わからないスレから改変転載。
http://science6.2ch.net/test/read.cgi/math/1175764597/723

pを素数、Zp={1,2,‥‥,p-1}、*をZp上の通常の乗算とする。

a∈Zp に対し、Zpの元のp-1項組 (1*a,2*a,3*a,‥‥,(p-1)*a) を
(1,2,‥‥,p-1) の置換とみなしたとき、
それが偶置換となるためのaの条件を求めよ。

847 名前:132人目の素数さん:2007/04/17(火) 07:00:01
pが奇素数とする。
aが原始根のとき奇置換。
aが原始根の偶数乗のとき偶置換。
aが原始根の奇数乗のとき奇置換。


848 名前:132人目の素数さん:2007/04/17(火) 10:54:18
>>847
これって (a/p)=1 と同値?

849 名前:132人目の素数さん:2007/04/17(火) 12:22:24
>>848
そうだね。Z/pZ の単元群は巡回群だから、結局 Z/(p-1)Zでの足し算による移動(ずらし)の
隅奇性を考えればいいだけだな。

850 名前:132人目の素数さん:2007/04/17(火) 22:12:47
別のところにも、書いてしまったんだけど、10桁の足し算を一瞬で解ける公式を教えてください。

851 名前:132人目の素数さん:2007/04/17(火) 23:04:30
>>850
ただの足し算に公式なんてものがあると思うか?

852 名前:132人目の素数さん:2007/04/17(火) 23:54:22
>>836
お願いしまーす

853 名前:132人目の素数さん:2007/04/18(水) 00:06:33
>>852
作問者に聞いてこい

854 名前:マーティン:2007/04/18(水) 00:08:50
誰か教えてください!! ━━━━━━━━━━━━B分のA÷D分のC=B分のA×C分のD の証明
━━━━━━━━━━━━の仕方を教えてください!! 今まで結論だけわかって使ってるんですけど証明となると……。

855 名前:132人目の素数さん:2007/04/18(水) 00:30:08
>>854
スレ違い

856 名前:マーティン:2007/04/18(水) 00:47:14
そう??でも結構この問題おもしろくない??

857 名前:132人目の素数さん:2007/04/18(水) 02:40:29
つまらない。死ね。

858 名前:マーティン:2007/04/18(水) 03:01:52
今、死にました。次の命令を下さい。

859 名前:132人目の素数さん:2007/04/18(水) 04:11:56
二度とこのスレに書き込むな。

860 名前:マーティン:2007/04/18(水) 13:01:26
じゃぁ死んどきます

861 名前:132人目の素数さん:2007/04/18(水) 13:48:10
>>860
二度とくるな馬鹿め

862 名前:132人目の素数さん:2007/04/18(水) 14:54:31
マーチンテラバカスwwww

863 名前:マーティン:2007/04/18(水) 16:32:05
やっぱせめて答えだけでも

864 名前:132人目の素数さん:2007/04/19(木) 14:10:57
>>863
死ね

865 名前:132人目の素数さん:2007/04/19(木) 17:00:36
死ね?!俺はやり方をきいてるんです。      できない人はいいです。

866 名前:132人目の素数さん:2007/04/19(木) 17:26:57
質問は適切なスレでしてくれないかな。
別のスレで聞いたのなら答えても良いけどこのスレでは俺は答えない。

そう面白いとは思わないからね。

867 名前:132人目の素数さん:2007/04/19(木) 19:01:42
まぁね俺もこだわりすぎたケドね この問題ってそんなに簡単??

868 名前:132人目の素数さん:2007/04/19(木) 19:43:36
簡単。くだらない。低脳は死ね。

869 名前:132人目の素数さん:2007/04/19(木) 20:04:46
でもこの問題に反応してくれたのはこのスレの人たちだけだし…… だから頼む!!教えてくれヽ(´Д`ヽ ミ ノ´Д`)ノ

870 名前:132人目の素数さん:2007/04/19(木) 20:20:32
>でもこの問題に反応してくれたのはこのスレの人たちだけだし
「くだらない」「死ね」という反応しかないがなwww

871 名前:132人目の素数さん:2007/04/19(木) 20:44:12
いやいや、こういう会話もまぁまぁ楽しいし 会話になってるかどうかはわからんけど

872 名前:132人目の素数さん:2007/04/20(金) 10:44:35
>糞ルーチン
>教えてくれ
はぁ?自分よりはるかに頭がいい人たちに対してその口の聞方は何?
俺なら例えネット上でもとても出来ない

873 名前:132人目の素数さん:2007/04/20(金) 10:51:09
じゃぁもう諦めます。四つやり方はわかったんですけど…あと一つは他でききます。迷惑かけてすみませんでした。

874 名前:132人目の素数さん:2007/04/21(土) 10:33:15
マーチンワロスwww


875 名前:132人目の素数さん:2007/04/21(土) 16:50:03
自作問題。

f,L:(a,b)→Rは次の2つの条件を満たすとする。
・Lは各点で微分可能(C^1級とは限らない)
・∀x∈(a,b),∀ε>0,∃δ>0 s,t y∈(a,b),0<|y−x|<δ → {f(y)−f(x)}/(y−x)≦L'(x)
このとき、次が成り立つことを示せ。
・a<x≦y<b → f(y)−f(x)≦L(y)−L(x)

876 名前:KingOfUniverse ◆667la1PjK2 :2007/04/22(日) 10:42:39
n_1, n_2 を整数とし、 d_1, d_2 を0より大きい整数とする。
さらにn_1/d_1, n_2/d_2 が既約分数表示になっているとする。
n_1/d_1=n_2/d_2ならば、n_1=n_2 かつ d_1=d_2 であることを証明せよ。

877 名前:KingOfUniverse ◆667la1PjK2 :2007/04/22(日) 10:47:40
n_1, n_2 を整数とし、 d_1, d_2 を0より大きい整数とする。
さらにn_1/d_1, n_2/d_2 が既約分数表示になっているとする。
n_1/d_1=n_2/d_2ならば、(n_1=n_2 かつ d_1=d_2) または n_1=0 であることを証明せよ。

878 名前:KingOfUniverse ◆667la1PjK2 :2007/04/22(日) 10:54:26
[>>876]でいいのか。

879 名前:132人目の素数さん:2007/04/22(日) 11:13:37
n_1=p*n_2
d_1=q*d_2

880 名前:132人目の素数さん:2007/04/23(月) 00:38:03
>876,878

題意より
 n_1*d_2 = n_2*d_1,
 gcd(n_1,d_1) = 1,
 gcd(n_2,d_2) = 1.
任意の素数p,qと自然数j,kについて
 p^j | n_1 ⇔ p^j | n_2
 q^k | d_1 ⇔ q^k | d_2
よって
 n_1=n_2, d_1=d_2.

881 名前:132人目の素数さん:2007/04/23(月) 00:59:30
開区間(a,b)に対し、|(a,b)|=b−aと定義する。また、空集合φに対し、|φ|=0と定義する。
{On}(n=1,2,…)は開区間の列(φも開区間とする)とし、(0,1)⊂∪[i=1〜∞]Oiが成り立って
いるとする。以下の問いに答えよ。
(1)Σ[i=1〜∞]|Oi|≧1が成り立つことを示せ。
(2)Σ[i=1〜∞]|Oi|=1が成り立つ{On}(n=1,2,…)を1つ求めよ。
(3)|Oi|<1 (i=1,2,…)が成り立つとき、Σ[i=1〜∞]|Oi|>1 が成り立つことを示せ。

882 名前:KingOfUniverse ◆667la1PjK2 :2007/04/23(月) 08:01:00
talk:>>880 素因数分解でできるのか。n_1/d_1=n_2/d_2かつ、d_2>d_1ならば、n_2/d_2=(n_2-n_1)/(d_2-d_1)が成り立つことを利用する方法もある。

883 名前:132人目の素数さん:2007/04/24(火) 17:09:33
Zagier's problems
http://www-groups.dcs.st-and.ac.uk/~john/Zagier/Problems.html

884 名前:132人目の素数さん:2007/04/24(火) 18:03:03
n人(n≧3)のグループから、任意の3人を選ぶ。
3人の誕生日の月と日が同じであるような確率 Pn を求めよ。
1年は365日とし、うるう年は考えない。


885 名前:132人目の素数さん:2007/04/24(火) 23:11:41
n人じゃなくて3人でいいの?
「月と日が同じ」ってのは?完全に一致するってこと??
なんにせよ、意図がよーわからん

886 名前:132人目の素数さん:2007/04/24(火) 23:36:21
すまん出題が悪かった。

n人(n≧3)のグループにおいて、誕生日が同じ3人組が存在する確率Pnを求めよ。
1年は365日とし、うるう年は考えない。


887 名前:KingOfUniverse ◆667la1PjK2 :2007/04/25(水) 07:25:19
pを0でない実数とし、qを実数とする。O=(0,0),A=(1,0),B=(p,q)のとき三角形OABの五心を求めよ。

888 名前:132人目の素数さん:2007/04/25(水) 07:26:16
>>884
誕生日のパラドクス【Part 8】
http://science6.2ch.net/test/read.cgi/math/1126016995/

参考スレ

889 名前:132人目の素数さん:2007/04/25(水) 16:28:01
>>886
>n人(n≧3)のグループにおいて、誕生日が同じ3人組が存在する確率Pnを求めよ。

p(n)=1-((n!*(1/2)^365)*納k=(2*n+1-(-1)^n)/4,365]C(365,k)*C(2*k,2*k-n))/(365^n).

計算例
p(67)=0.275082173722958739776582350023661578986456826158565230293…,
p(90)=0.534195571499801513117864155312496780467198156847508353184…,
p(159)=0.98083145864996116932607047331416046602116980905361451973….

890 名前: ◆BhMath2chk :2007/04/27(金) 00:00:00
>>875
g(x)=f(x)−L(x)とおくと
limsup_{y−>x}((g(y)−g(x))/(y−x))≦0なので
a<x≦y<bのときg(x)≧g(y)。


891 名前:132人目の素数さん:2007/04/27(金) 03:16:26
>>888
すまん。 発見が遅れた。

>>884
20%くらいじゃないの?

892 名前:132人目の素数さん:2007/04/28(土) 16:04:03
>>846
> pを素数、Zp={1,2,‥‥,p-1}、*をZp上の通常の乗算とする。
> a∈Zp に対し、Zpの元のp-1項組 (1*a,2*a,3*a,‥‥,(p-1)*a) を
> (1,2,‥‥,p-1) の置換とみなしたとき、

この置換をaに対応させるとZpの置換表現になっているのは自明ですか?

893 名前:132人目の素数さん:2007/04/30(月) 20:40:34
自明じゃないけど、このレベルの問題なら、
<Zp,*>が乗法群になることくらいは既知扱いでもいいと思われ。

894 名前:132人目の素数さん:2007/05/01(火) 09:28:33
>>893
> pを素数、Zp={1,2,‥‥,p-1}、*をZp上の通常の乗算とする。
> a∈Zp に対し、Zpの元のp-1項組 (1*a,2*a,3*a,‥‥,(p-1)*a) を
> (1,2,‥‥,p-1) の置換σ_aとみなしたとき

σ_a*σ_b=σ_(a*b )   (左辺は置換の積)

が成立するか?ってことです。

895 名前:132人目の素数さん:2007/05/01(火) 10:01:18
>>894
失敬。自明だった

896 名前:132人目の素数さん:2007/05/06(日) 23:12:31
>>890
平均値の定理?


897 名前:132人目の素数さん:2007/05/07(月) 21:38:46
tri

898 名前:132人目の素数さん:2007/05/11(金) 00:12:46
転載。
http://science6.2ch.net/test/read.cgi/math/1174230352/506

正の整数が「交代的」であるとは、その整数を十進法表示したときに、
どの隣接する2つの桁の数字に対してもそれらの偶奇が異なることをいう。
交代的な倍数をもつような正の整数をすべて決定せよ。

899 名前:132人目の素数さん:2007/05/11(金) 21:39:22
>>898
これ数オリの問題

900 名前:132人目の素数さん:2007/05/12(土) 18:05:01
>>882
 このスレは無限降下する…

900げとー

901 名前:132人目の素数さん:2007/05/14(月) 23:56:56
自然数mがm=(p[1]^a[1])*(p[2]^a[2])*・・・*(p[n]^a[n])と素因数分解されたとする。
rをmの約数の総数、sをmの約数の逆数の総和とするとき、r/s≧nとなることを示せ。
また、等号が成り立つのはmが完全数であるときに限ることを示せ。

902 名前:132人目の素数さん:2007/05/15(火) 00:27:47
>901
題意より
 r = Π[k=1,n] {1 + p[k]^1 + p[k]^2 + …… + p[k]^a[k]},
 s = Π[k=1,n] {1 + p[k]^(-1) + p[k]^(-2) + …… + p[k]^(-a[k])},
辺々割って
 r/s = Π[k=1,n] p[k]^a[k] = m.
等号成立は m=n=1 ?

903 名前:132人目の素数さん:2007/05/15(火) 00:33:19
>>901
完全数28=2^2*7のとき
s=2、r=6、n=2
で成立してないような。 

rをmの約数の総数(1と自分自身を除く)ってことか? 

904 名前:132人目の素数さん:2007/05/15(火) 00:53:21
>903
そうすると、題意より
 r = Π[k=1,n] {1 + p[k]^1 + p[k]^2 + …… + p[k]^a[k]} -1 -m,
 s = Π[k=1,n] {1 + p[k]^(-1) + p[k]^(-2) + …… + p[k]^(-a[k])} -1 -(1/m),
辺々割って
 r/s = m.


905 名前:132人目の素数さん:2007/05/15(火) 01:01:00
総和≠総数

906 名前:132人目の素数さん:2007/05/15(火) 01:34:25
完全数(偶数、奇数を問わない)の約数の逆数の総和は常に2である。
ってのはあるけど

(mの約数の逆数の総和)×m = (mの約数の総和)

(mの約数の総和)=2mなので、
(mの約数の逆数の総和)=2
 

907 名前:901:2007/05/15(火) 02:07:23
題意が間違ってた
不等式の中のnは間違いで、a[1]+a[2]+・・・+a[n]だった
本当に申し訳ない

908 名前:901:2007/05/15(火) 02:39:51
申し訳ない。二つの問題が混ざってた。
>>901 の二行目までは正しい
三行目は次のように読み替えてください。

mが完全数のときはr/s=a[1]+a[2]+・・・+a[n]となることを示せ。

909 名前:132人目の素数さん:2007/05/15(火) 15:00:28
>>901
r/s はmの約数の調和平均Hになってるみたい。
また、約数の相乗平均G=√m も証明できるので

H=r/s≦√m ってのは証明できた。 下限はまだ

910 名前:132人目の素数さん:2007/05/16(水) 02:28:32
>>909
相乗平均=√m、綺麗で感動した
証明は帰納法でごり押ししてみた

911 名前:132人目の素数さん:2007/05/16(水) 04:50:32
r/s≧2^n/(n+2)を示せるな
もちろん2^n/(n+2)≧n (等号はn=2のみ)
k番目の素数が>klogkなことを使って改良すると
例えばn≧3で、r/s>(5log3/12)2^n/lognとか

912 名前:132人目の素数さん:2007/05/16(水) 04:52:57
r/s≧(2^(n+1))/(n+2)
だ、ミスった

913 名前:132人目の素数さん:2007/05/16(水) 12:59:08
n≧3で、r/s>(5/12)log(7/2)*(2^n)/log(n+1/2)に修正
これも大雑把だけど

914 名前:132人目の素数さん:2007/05/17(木) 02:20:26
下からの評価をするとき、結局a[k]≧1を使うから荒くなるんだよね
しかもm→∞のときn→∞となるとは限らないから、もうなんかダメポ

915 名前:132人目の素数さん:2007/05/17(木) 02:26:49
いや、nだけの式を作るならa[k]≧1を使うのは当然でしょう
じっさい自由なんだから

916 名前:132人目の素数さん:2007/05/17(木) 03:11:45
目的がnによる評価だから仰るとおりです
気分的にm→∞のときr/s→∞と限らないのが嫌で
でも>>913のように、nに関する評価を精密にするのは大切だと思う


917 名前:132人目の素数さん:2007/05/17(木) 03:17:06
>>916
たとえばこんなのはどう?
q_kをk番目の素数として、
r/s≧Π[k=1,n](2p_k/(p_k+1))≧Π[k=1,n](2q_k/(q_k+1))
1番右はnだけの関数でf(n)とおいとく
さらに適当なmをとってそこから先を近似して
f(n)≧{Π[k=1,m](q_k/(q_k+1))}(2^n)(f(m)/f(n)) (n≧m)
たとえばf(x-1/2)=(logx)^(1+1/logx-loglogx/logx)とすると、
(m=15としたときn=1000でのf(n)との誤差は10%以下
これはコンピュータで計算しただけで誤差評価ではない)

918 名前:132人目の素数さん:2007/05/17(木) 03:18:31
あ、mって使ったらまずかったな、このmはただの数字ってことで・・・

919 名前:132人目の素数さん:2007/05/17(木) 03:25:19
f(n)≧{Π[k=1,j](q_k/(q_k+1))}(2^n)(g(m)/g(n)) (n≧j)
たとえばg(x-1/2)=(logx)^(1+1/logx-loglogx/logx)とすると、
j=15としたときn=1000での右辺とf(n)との誤差は10%以下

どうも調子悪い

920 名前:132人目の素数さん:2007/05/20(日) 20:17:04
順列(1,2,3,...n)を辞書式順序でn!個並べると
、、、(偶置換、偶置換)、(奇置換、奇置換)、(偶置換、偶置換)、、、
となることを示せ

921 名前:132人目の素数さん:2007/05/20(日) 20:30:11
>>920
問題の意味が…

922 名前:132人目の素数さん:2007/05/20(日) 20:49:18
すいません。

例えば n=3 のとき
(1,2,3) (1,3,2) (2,1,3) (2,3,1) (3,1,2) (3,2,1) が辞書式順序で並べた数列で

数列に対応する置換σを(σ(1)σ(2)σ(3))とするということです。
上記の場合 恒等置換(偶), 互換(奇)(2,3)、(1,2), 巡回置換(偶)(1,2,3), (1,3,,2),
互換(奇)(1,3) と最初と最後を除き奇奇偶偶となるということです。

簡単ならごめんなさい。

923 名前:132人目の素数さん:2007/05/20(日) 21:20:03
(1,5,4,2,3),(1,5,4,3,2),(2,1,3,4,5)。


924 名前:132人目の素数さん:2007/05/20(日) 22:59:55
>>923 ありゃ間違ってましたか。 実はn=4のときまでしか確かめてなかった、、
  
0=偶置換、1=奇置換、n’をnの偶奇を反転したものとすると

n=2のとき 2=01  2’=10
n=3のとき 3=22’2=011001
n=4のとき 4=33’33’=011001 100110 011001 100110
n=5のとき 5=44’44’4=011001100110011001100110 1001100、、、

となるようです(多分)。 また 最初と最後の偶奇は 

n≡0、1(mod 4) のとき00
n≡2、3(mod 4) のとき01 

925 名前:132人目の素数さん:2007/05/21(月) 10:17:41
いいや面白い予想を書くスレじゃないから

926 名前:132人目の素数さん:2007/05/22(火) 06:11:35
t(n) := (n を 10進数で書いたときの各位の和)
とする。
このとき、Σ_{i=1 〜 ∞} t(n)/{(n)(n+1)} を求めよ。

927 名前:926:2007/05/22(火) 06:13:44
nじゃなくてiじゃないか。

Σ_{i=1 〜 ∞} t(i)/{(i)(i+1)}

928 名前:132人目の素数さん:2007/05/22(火) 13:41:14
>>926
∞ 調和級数で発散

929 名前:132人目の素数さん:2007/05/23(水) 01:18:20
>927

t(i) ≦ 9*log(i+1) ≦ c√i < 2c/{1/√i + 1/√(i+1)} = 2ci(i+1){1/√i - 1/√(i+1)},
c = (9/2)*log(5) = 3.145365…


930 名前:927:2007/05/24(木) 03:14:55
ちゃんとした数に収束するよ。

自分で用意した証明は、大学1、2年程度の知識が必要だけど。

931 名前:132人目の素数さん:2007/05/24(木) 14:06:06
>>927
良問。
オリジナル?

932 名前:132人目の素数さん:2007/05/24(木) 18:52:34
>>931
Digit Sum -- from Wolfram MathWorld
http://mathworld.wolfram.com/DigitSum.html

933 名前:132人目の素数さん:2007/05/24(木) 23:44:26
2進法展開の場合を計算したらlog4になった。計算の方針は、
f(x)=Σ[k=1〜∞]t(k)x^(k−1) (0≦x<1)
とおき、これを別の計算によって簡単な形にする。その結果は
f(x)=1/(1−x^2)+{1/(1−x)}Σ[k=1〜∞]{x^(2^k−1)}/{1+x^(2^k)}
となる(計算は略)。この式から、
Σ[k=1〜∞]t(k)/{k(k+1)}=∫[0,1]∫[0,t]f(x)dxdt=∫[0,1](1−x)f(x)dx
=∫[0,1]1/(1+x)+Σ[k=1〜∞]{x^(2^k−1)}/{1+x^(2^k)}dx
=log2+Σ[k=1〜∞](log2)/2^k
=log4
になる。積分とΣの順序交換についても確認が必要だが、面倒くさいのでここでは書かない。
10進法の場合も似たような計算かな?

934 名前:132人目の素数さん:2007/05/25(金) 00:00:17
マテよ、Σ[k=1〜∞]t(k)/{k(k+1)}=∫[0,1]∫[0,t]f(x)dxdt=… という形で
計算するより、Σ[k=1〜∞]t(k)/{k(k+1)}=lim[y↑1]∫[0,y](1−x)f(x)dx=…
の形で計算した方が安全だな。

935 名前:132人目の素数さん:2007/05/31(木) 22:56:35
自然数l,m,nに対し、m*m行列Aをa_ij=C[l+n,n+i-j]、n*n行列Bをb_ij=C[l+m,m+i-j]で定めるとき、detA=detBを示せ。

936 名前:132人目の素数さん:2007/06/01(金) 00:03:53
>>935
p<qに対してC[p,q]はどう定義して?

937 名前:132人目の素数さん:2007/06/01(金) 09:04:04
0だろ

938 名前:132人目の素数さん:2007/06/01(金) 22:29:25
>>936
ごめん書いてなかった
937の言うとおりでok

939 名前:132人目の素数さん:2007/06/03(日) 12:55:32
自作問題。

f,g:[0,+∞) → [0,+∞)は単調減少関数で、lim[x→+∞]f(x)=0,lim[x→+∞]g(x)=0
であるとする。このとき、次を示せ。
・任意のε≧0に対して、広義積分∫[0,∞]f(x)g(εx)e^(ix)dxが存在する。
・lim[ε↓0]∫[0,∞]f(x)g(εx)e^(ix)dx=g(0)∫[0,∞]f(x)e^(ix)dxが成り立つ。

940 名前:132人目の素数さん:2007/06/12(火) 00:54:02
殆ど、初めて勉強した事を得意になって開陳する厨房状態ですが。ナッシュの埋め込み定理の
具体例に関して。
 2次元球面は十分に大きな空間においては好きなだけ小さな領域に等長に埋め込める。私は
めちゃくちゃおおざっぱな評価をしたのですが、このようにできる最小次元は何次元なんで
しょう?これじゃ問題と言うより質問になっちゃうが。4次元でできるかな?以上shape operator
なるものは超曲面の場合しか具体計算したことの無い人間の戯言でした。

941 名前:132人目の素数さん:2007/06/12(火) 03:36:46
>>940
うせろ!

942 名前:132人目の素数さん:2007/06/14(木) 21:51:21
1,2,3, ...n から二数 x、yを適当に選んで消してf(x、y)を付け加える。
この操作を続け最終的に残る数が二数を選ぶ順番によらないようなf(x、y)をすべて求めよ。

943 名前:132人目の素数さん:2007/06/14(木) 22:51:32
定数関数

944 名前:132人目の素数さん:2007/06/15(金) 01:12:56
交換法則と結合法則を満たす二項演算全部か?

945 名前:132人目の素数さん:2007/06/22(金) 22:02:47
初等数学で。次の等式を満たす整数x,y,z (x≦y)を全て求めてください。
(1) 2^x+2^y=2^z
(2) 3^x+3^y=3^z
(3) 2^x+2^y=3^z

946 名前:132人目の素数さん:2007/06/22(金) 23:00:02
>>945
つ…いや、何でもない。

947 名前:132人目の素数さん:2007/06/22(金) 23:20:52
>>935
に挑戦してるんだが、だめだなあ。
lで帰納法かなあ。うまくいかん。

948 名前:132人目の素数さん:2007/06/24(日) 17:34:07
>935, >947
det(A) = Π[k=0,m-1] (l+n+k)!k!/[(l+k)!(n+k)!],
 det(B) = Π[k=0,n-1] (l+m+k)!k!/[(l+k)!(m+k)!].

949 名前:132人目の素数さん:2007/06/24(日) 17:40:51
>945
 (1) x=y=z-1
 (2) なし
 (3) (x,y,z)=(-1,-1,0), (0,1,1), (0,3,2)

950 名前:948:2007/06/24(日) 20:59:57
>935, >947

>948
 (l+n+k)!/(n+k)! = Π[p=0,l-1] (p+n+k+1),
 k!/(l+k)! = Π[p=0,l-1] 1/(p+k+1),
 (l+m+k)!/(m+k)! = Π[q=0,l-1] (q+m+k+1),
 k!/(l+k)! = Π[q=0,l-1] 1/(q+k+1),
を代入すると いづれもl項の積の形になり
 det(A) = Π[p=0,l-1] {(p+m+n)!/(p+n)!} * {p!/(p+m)!},
 det(B) = Π[q=0,l-1] {(q+m+n)!/(q+m)!} * {q!/(q+n)!},
だから、
 det(A) = det(B).

951 名前:132人目の素数さん:2007/06/24(日) 22:26:54
>>948>>950
やるぅー。
ところで>>948式は俺の頭ではどうにも導けそうにないんだが 。
教えて下せえ。ところどころ端折ってもいいんで。

952 名前:132人目の素数さん:2007/06/30(土) 13:08:11
今、1億円の財産を持った一人と1000万円の借金がある10人が
輪になって座っています。 ここで、借金がある人は両隣の二人から
借金と同額のお布施を受けることが出来るといいます。

さて、このときこのお布施を続けていくと全員が平等に無一文になってしまう
ことを証明してください。

953 名前:132人目の素数さん:2007/06/30(土) 16:35:38
>>952
> 両隣の二人から借金と同額のお布施を受けることが出来るといいます。

n円の借金がある人は、両隣からそれぞれn円? それともn/2円ずつ?

借金がある人の隣の人は借金してでも隣にお布施をするの? それとも借金のない人だけ?


954 名前:132人目の素数さん:2007/06/30(土) 16:46:04
>>953
両隣からそれぞれn円です。

借金がある人はそれが財産に化けるというむちゃくちゃな話ですが、、 

955 名前:132人目の素数さん:2007/06/30(土) 17:29:41
>>954
なるほどわかった。

できればもう一つの質問にも答えていただきたい。

956 名前:132人目の素数さん:2007/06/30(土) 17:42:43
>>955
借金がある人も借金を増やして隣の借金のある人にお布施をします。

ちなみにこの問題を一般的に考えると最先端の数学(表現論)ともつながっているそうです。

957 名前:132人目の素数さん:2007/06/30(土) 17:45:02
ちょっと試してみたが、1億とマイナス一千万が10人では全員がゼロになるが
合計がゼロならかならず全員がゼロになるというわけではないのだな‥

958 名前:132人目の素数さん:2007/06/30(土) 19:00:49
>>957
すいません、問題に不備がありました。
合計が0という設定はまずかったです。

初期値の合計が0より大きいときいつかは全員の財産が
0以上になるようになるっていうふうに訂正します。

ヒントは不変量 

959 名前:132人目の素数さん:2007/06/30(土) 21:36:14
金のやり取りの操作が線型である以上
収束先が固有値で決まるのは自明だろう

960 名前:132人目の素数さん:2007/06/30(土) 21:48:12
線型かどうかは重要なのかなあ。
問題の中では明言されてないけど、それぞれが財産を出来るだけ増やそうとしていたらどうなんだろ。

961 名前:132人目の素数さん:2007/07/01(日) 01:55:02
これ、IMOの過去問で見た事あるぞ

962 名前:132人目の素数さん:2007/07/01(日) 02:05:49
>>959
必ず収束するのか?

963 名前:132人目の素数さん:2007/07/01(日) 04:58:46
問題。
20桁(20桁に満たない数は先頭ゼロ埋めで20桁にする)の非負整数の中で
以下の条件を満たすものはいくつあるか?
上位10桁と下位10桁をそれぞれ10桁の数とみなして
この二つを足して自乗すると、元の数になる。


964 名前:132人目の素数さん:2007/07/01(日) 05:12:21
>952の問題の意味がよくわからんのだけど
借金がある人はお布施を受けることができるっていうのは
借金がある人全員が一斉に受け取る操作を繰り返すの?
つまり
初期段階:
 一人が100000000 のこり十人が-10000000
時刻1:
 100000000円の人は80000000に
 その両隣の人は0
 残り八人は-10000000
・・・みたいな感じで時刻を十分に進めるとどうなるかを考えるの?

それとも「十一人のうちの誰か一人がお布施を要求する」という現象が
十分多い有限の回数起こると借金持ちが必ず消えるっていうのを示すの?

965 名前:132人目の素数さん:2007/07/01(日) 10:18:57
>>964
初期値の合計は0より大きくしないといけないので借金1000万円の人数は9人にしてください。

借金持ちを任意に一人選んで、その両隣からお金を奪ってプラスにするってこと

y<0 、(x、y、z)→(x+y、-y、z+y) なる変換を負の値を適当に選んで
繰り返していくといつか全部0以上になるっていうことです。

上の変換で変化するある量を計算して背理法を使うと簡単に示せます。

966 名前: ◆BhMath2chk :2007/07/03(火) 17:00:01
x(k+n)=x(k)。

Σ_{0≦i<n,0<k<n}((Σ_{0≦j<n}(x(i+j)))^2)。


967 名前:132人目の素数さん:2007/07/03(火) 20:28:47
lim[x↑1](1−x)Σ[k=1〜∞]kx^(k^2)=1/2を示せ。

968 名前:132人目の素数さん:2007/07/04(水) 00:08:35
>963
今ちょっと考えたけど
0と1、それに99999999980000000001 があてはまるね
これは20桁以外でも常に同様のことが言える

また4桁の場合で同じ問題を考えると
まず上に挙げたように0、1、9801 が条件を満たす
もし他に条件を満たすものがなかったら
「任意の偶数桁でこの三つだけが条件を満たす」とか予想したくなるけど
実際には
2025,3025
がさらに存在していて、どうやら桁数2nのとき、nの値固有の形の解とかが色々出て来そう


969 名前:132人目の素数さん:2007/07/04(水) 00:55:36
10^10以下の平方数が幾つあるか、
またそれらを二数の和にする方法が何通りあるか、
ということだと思うよ。

970 名前:969:2007/07/04(水) 00:58:50
違うね、しばらくROMるわorz

971 名前:132人目の素数さん:2007/07/04(水) 01:13:32
よし、俺もROMるわ

972 名前:W.K.B.:2007/07/04(水) 03:53:06
>967

極大点(鞍点?)のまわりで放物線近似する。つまり正規分布近似だな。
 log{kx^(k^2)} = -|log(x)|(k^2) + log(k) ≒ -(1/2)log(2e|log(x)|) - {k√(2|log(x)|) -1}^2,
-∞<k<∞ で和分する

973 名前:132人目の素数さん:2007/07/04(水) 11:00:00
三百日四時間。


974 名前:132人目の素数さん:2007/07/04(水) 13:08:46
>>972
正規分布近似なんて知らないお( ^ω^)

an=√n (nは平方数), 0 (それ以外)
と定義すると、この数列はチェザロのC-1総和法で1/2に収束する。すなわち、
lim[n→∞](a1+a2+…+an)/n=1/2 が成り立つ。よって特に、アーベル総和法
でも1/2に収束する。すなわち、lim[x↑1](1−x)Σ[k=1〜∞]akx^k=1/2が
成り立つ。この式はlim[x↑1](1−x)Σ[k=1〜∞]kx^(k^2)=1/2を意味する。

975 名前:132人目の素数さん:2007/07/05(木) 00:18:48
>>968
(x+y)(x+y-1)=999999999x

976 名前:132人目の素数さん:2007/07/05(木) 02:34:35
>972

σ = μ/√2 = 1/{2√(Log|x|)} とおくと
k・x^(k^2) ≒ {√(2/e)}σ・exp{-[(k-μ)^2/(2σ^2)]}
0<k<∞ で積分して
 Σ[k=1,∞) k・x^(k^2) ≒ (2/√e)σ^2・∫[-1,∞) exp(-t^2)dt
 = (3.2661…/√e)σ^2
 = 1.9810…σ^2
 = 0.49525… / Log|x|,
 Lim[x→1-0] (1-x)Σ k・x^(k^2) → 0.49525…
微妙にズレてまつね…

977 名前:132人目の素数さん:2007/07/05(木) 09:32:37
>>963
なかなか面白い

x=10^10a+b (0≦a,b<10^10)とする
(a+b)^2=10^10a+b<10^20より、0≦a+b<10^10である。
(a+b){10^10-(a+b)}=(10^10-1)b、さらにちょっと変形して
(a+b){(10^10-1)-(a+b-1)}=(10^10-1)b
よって(a+b)(a+b-1)≡0 (mod. 10^10-1)
10^10-1=3^2*11*41*271*9091と素因数分解されるが、
a+b,a+b-1は互いに素なので、9,11,41,271,9091を振り分けることになる
これらを振り分けた積をそれぞれp,qとすれば、
p,q≠1のとき、中国剰余定理より
a+b≡0 (p), a+b≡1 (q) となるa+bがmod. 10^10-1で丁度1つある
このとき、a+b≠0だから、a+bは1つとなる
q=1とすればa+b=0,10^10-1であり、p=1とすればa+b=1となる
a+b≧1のとき、
(a+b){(10^10-1)-(a+b-1)}-(10^10-1)(a+b)=-(a+b)(a+b-1)≦0
であるから、実際に(a,b)をとることが可能である
a+b=0なら、a=b=0でOK
以上から、2^5+1=33(通り)

978 名前:132人目の素数さん:2007/07/05(木) 09:38:12
最後a+b=0分ける必要なかったな、a+b≧0で整数だから)-(a+b)(a+b-1)≦0
で良かった

979 名前:132人目の素数さん:2007/07/05(木) 18:47:39
5a+3b (a,bは0以上の整数) で表すことのできない最大の数は
なに?

980 名前:132人目の素数さん:2007/07/05(木) 18:59:08
最大の整数なら7

981 名前:132人目の素数さん:2007/07/06(金) 07:00:00
三百二日。


982 名前:132人目の素数さん:2007/07/06(金) 09:10:00
面白い問題おしえて〜な 十三問目
http://science6.2ch.net/test/read.cgi/math/1183680000/


983 名前:132人目の素数さん:2007/07/06(金) 10:22:16
>>982
乙枯れ

984 名前:132人目の素数さん:2007/07/06(金) 16:50:15
埋めたる

985 名前:132人目の素数さん:2007/07/07(土) 06:18:00
0707070707

986 名前:132人目の素数さん:2007/07/07(土) 06:24:41
>>985
さて、誰が取るか?

987 名前:132人目の素数さん:2007/07/07(土) 06:31:06
やはり、ここは華麗に1000を2007/07/07 07:07:07でとっておきたいものだが。
どうしたらよいものか。


988 名前:132人目の素数さん:2007/07/07(土) 06:34:52
過疎スレだから案外かなうんじゃないか?

989 名前:132人目の素数さん:2007/07/07(土) 06:42:49
別に私がとれなくとも良い。誰かが取れれば。
その場合、果たして999まで埋めてしまうのは得策だろうか。
数うちゃあたる戦法で出来るだけ残しておいたほうがよいだろうか。
しかし、現在このスレを見ている何人がそれを狙っているのか。
それがあまりに少ない場合、それは裏目になるはずだ。


990 名前:132人目の素数さん:2007/07/07(土) 06:44:19
同一IPで続けてかけるのは5レス程度らしいから
いまのままだと時刻までに埋まりきらない可能性もあるな

991 名前:132人目の素数さん:2007/07/07(土) 06:46:40
それはつまり>>995において我々の夢は儚く散るということか?

992 名前:132人目の素数さん:2007/07/07(土) 06:52:13
おいらも参加すれば叶うかもしれないお( ^ω^)

993 名前:132人目の素数さん:2007/07/07(土) 07:00:00
三百三日。


994 名前:132人目の素数さん:2007/07/07(土) 07:02:01
リハーサル
2007/07/07 07:07:07

995 名前:994:2007/07/07(土) 07:02:46
誤差1秒

996 名前:132人目の素数さん:2007/07/07(土) 07:03:00
三百三日三分。


997 名前:132人目の素数さん:2007/07/07(土) 07:06:00
三百三日六分。


998 名前:132人目の素数さん:2007/07/07(土) 07:06:22
なんと、ゴルゴを彷彿とさせる正確さ。ここは>>996,7に依頼すべきか。

999 名前:132人目の素数さん:2007/07/07(土) 07:09:01
三百三日九分。


1000 名前:132人目の素数さん:2007/07/07(土) 07:09:44
ゴルアアアア

1001 名前:1001:Over 1000 Thread
このスレッドは1000を超えました。
もう書けないので、新しいスレッドを立ててくださいです。。。


全部 最新50
DAT2HTML 0.34a Converted.